Hip replacmt; health promo Nclex

Réussis tes devoirs et examens dès maintenant avec Quizwiz!

food high in calcium

diary, cereal, bread

patient is taking spironolactone and K level is 6.0 what is priority Nursing intervention

give IV insulin , dextrose IV and calcium gluconate IV

a patient has renal damage because of a long history of diabetes. this becomes a problem of excretion and total body imbalances defines as which one

hyperkalmeia

A client who has had total hip replacement surgery asks the nurse when she will be able to use a regular-height toilet seat again. What is the nurse's best response?

As soon as you are able to walk without a limp

thirst, dry mucous membranes, decreased urine output requires what priority intervention

0.45% sodium chloride

pt diagnoses with hypercalcemia. priority

0.9% NS

The nurse is caring for a client who had right total knee replacement surgery 3 days ago. During the assessment, the nurse notes that the client's right lower leg is twice the size of the left. What is the nurse's priority intervention?

Assess the client's respiratory status.

The nurse is caring for a client who has had hip replacement surgery 2 days before. The client reports severe pain at the surgical site despite having received 2 Vicodin tablets 2 hours previously. The client is requesting IV pain medication. What is the nurse's primary intervention?

Assess the surgical site for signs of infection

Which of the following is significant data to gather from a client who has been diagnosed with pneumonia? Select all that apply. 1. Quality of breath sounds. 2. Presence of bowel sounds. 3. Occurence of chest pain. 4. Amount of peripheral edema. 5. Color of nail beds.

1, 3, 5. A respiratory assessment, which includes auscultating breath sounds and assessing the color of the nail beds, is a priority for clients with pneumonia. Assessing for the presence of chest pain is also an important respiratory assessment as chest pain can interfere with the client's ability to breathe deeply. Auscultating bowel sounds and assessing for peripheral edema may be appropriate assessments, but these are not priority assessments for the client with pneumonia.

The nurse administers two 325 mg aspirin every 4 hours to a client with pneumonia. The nurse should evaluate the outcome of administering the drug by assessing which of the following? Select all that apply. 1. Decreased pain when breathing. 2. Prolonged clotting time. 3. Decreased temperature. 4. Decreased respiratory rate. 5. Increased ability to expectorate secretions.

1, 3. Aspirin is administered to clients with pneumonia because it is an analgesic that helps control chest discomfort and an antipyretic that helps reduce fever. Aspirin has an anticoagulant effect, but that is not the reason for prescribing it for a client with pneumonia, and the use of the drug will be short term. Aspirin does not affect the respiratory rate, and does not facilitate expectoration of secretions.

Which of the following findings would most likely indicate the presence of a respiratory infection in a client with asthma? 1. Cough productive of yellow sputum. 2. Bilateral expiratory wheezing. 3. Chest tightness. 4. Respiratory rate of 30 breaths/ minute.

1. A cough productive of yellow sputum is the most likely indicator of a respiratory infection. The other signs and symptoms- wheezing, chest tightness, and increased respiratory rate- are all findings associated with an asthma attack and do not necessarily mean an infection is present.

When developing a discharge plan to manage the care of a client with chronic obstructive pulmonary disease (COPD), the nurse should advise the the client to expect to: 1. Develop respiratory infections easily. 2. Maintain current status. 3. Require less supplemental oxygen. 4. Show permanent improvement.

1. A client with COPD is at high risk for development of respiratory infections. COPD is slowly progressive; therefore, maintaining current status and establishing a goal that the client will require less supplemental oxygen are unrealistic expectations. Treatment may slow progression of the disease, but permanent improvement is highly unlikely.

The client with pneumonia develops mild constipation, and the nurse administers docusate sodium (Colace) as ordered. This drug works by: 1. Softening the stool. 2. Lubricating the stool. 3. Increasing stool bulk. 4. Stimulating peristalsis.

1. Docusate sodium (Colace) is a stool softener that allows fluid and fatty substances to enter the stool and soften it. Docusate sodium does not lubricate the stool, increase stool bulk, or stimulate peristalsis.

A client with deep vein thrombosis suddenly develops dyspnea, tachypnea, and chest discomfort. What should the nurse do first? 1. Elevate the head of the bed 30 to 45 degrees. 2. Encourage the client to cough and deep breathe. 3. Auscultate the lungs to detect abnormal breath sounds. 4. Contact the physician.

1. Elevating the head of the bed facilitates breathing because the lungs are able to expand as the diaphragm descends. Coughing and deep breathing do not alleviate the symptoms of a pulmonary embolus, nor does lung auscultation. The physician must be kept informed of changes in a client's status, but the priority in this case is alleviating the symptoms.

A client experiencing a severe asthma attack has the following arterial blood gas: pH 7.33; Pco2 48; Po2 58; HCO3 26. Which of the following orders should the nurse perform first? 1. Albuterol (Proventil) nebulizer. 2. Chest x-ray. 3. Ipratropium (Atrovent) inhaler. 4. Sputum culture.

1. The arterial blood gas reveals a respiratory acidosis with hypoxia. A quick-acting bronchodilator, albuterol, should be administered via nebulizer to improve gas exchange. Ipratropium is a maintenance treatment for bronchospasm that can be used with albuterol. A chest x-ray and sputum sample can be obtained once the client is stable.

The nurse is instructing a client with COPD how to do pursed-lip breathing. In which order should the nurse explain the steps to the client? 1. "Breathe in normally through your nose for 2 counts (while counting to yourself, one, two)." 2. "Relax your neck and shoulder muscles." 3. "Pucker your lips as if you were going to whistle." 4. "Breathe out slowly through pursed lips for 4 counts (while counting to yourself, one, two, three, four)."

2, 1, 3, 4. The nurse should instruct the client to first relax the neck and shoulders and then take several normal breaths. After taking a breath in, the client should pucker the lips, and finally breathe out through pursed lips.

Which of the following mental status changes may occur when a client with pneumonia is first experiencing hypoxia? 1. Coma. 2. Apathy. 3. Irritability. 4. Depression.

3. Clients who are experiencing hypoxia characteristically exhibit irritability, restlessness, or anxiety as initial mental status changes. As the hypoxia becomes more pronounced, the client may become confused and combative. Coma is a late clinical manifestation of hypoxia. Apathy and depression are not symptoms of hypoxia.

A client with pneumonia has a temperature of 102.6 ° F (39.2 ° C), is diaphoretic, and has a productive cough. The nurse should include which of the following measures in the plan of care? 1. Position changes every 4 hours. 2. Nasotracheal suctioning to clear secretions. 3. Frequent linen changes 4. Frequent offering of a bedpan.

3. Frequent linen changes are appropriate for this client because of the diaphoresis. Diaphoresis produces general discomfort. The client should be kept dry to promote comfort. Position changes need to be done every 2 hours. Nasotracheal suctioning is not indicated with the client's productive cough. Frequent offering of a bedpan is not indicated by the data provided in this scenario.

When caring for the client who is receiving an aminoglycoside antibiotic, the nurse should monitor which of the following laboratory values? 1. Serum sodium. 2. Serum potassium. 3. Serum creatinine. 4. Serum calcium.

3. It is essential to monitor serum creatinine in the client receiving an aminoglycoside antibiotic because of the potential of this type of drug to cause acute tubular necrosis. Aminoglycoside antibiotics do not affect serum sodium, potassium, or calcium levels.

The nurse teaches a client with chronic obstructive pulmonary disease (COPD) to assess for signs and symptoms of right-sided heart failure. Which of the following signs and symptoms should be included in the teaching plan? 1. Clubbing of nail beds. 2. Hypertension. 3. Peripheral edema. 4. Increased appetite.

3. Right-sided heart failure is a complication of COPD that occurs because of pulmonary hypertension. Signs and symptoms of right-sided heart failure include peripheral edema, jugular venous distention, hepatomegaly, and weight gain due to increased fluid volume. Clubbing of nail beds is associated with conditions of chronic hypoxemia. Hypertension is associated with left-sided heart failure. Clients with heart failure have decreased appetites.

A client with pneumonia is experiencing pleuritic chest pain. The nurse should assess the client for: 1. A mild but constant aching in the chest. 2. Severe midsternal pain. 3. Moderate pain that worsens on inspiration. 4. Muscle spasm pain that accompanies coughing.

3. Chest pain in pneumonia is generally caused by friction between the pleural layers. It is more severe on inspiration than on expiration, secondary to chest wall movement. Pleuritic chest pain is usually described as sharp, not mild or aching. Pleuritic chest pain is not localized to the sternum, and it is not the result of a muscle spasm.

A client with acute asthma is prescribed short-term corticosteroid therapy. Which is the expected outcome for the use of steroids in clients with asthma? 1. Promote bronchodilation. 2. Act as an expectorant. 3. Have an anti-inflammatory effect. 4. Prevent development of respiratory infections.

3. Corticosteroids have an anti-inflammatory effect and act to decrease edema in the bronchial airways and decrease mucus secretion. Corticosteroids do not have a bronchodilator effect, act as expectorants, or prevent respiratory infections.

What intervention is most important to teach the client at risk for hypercalcemia? A. "Avoid drinking coffee and other caffeinated beverages." B. "Be sure to drink at least 3 liters of fluids each day." C. "Do not eat or drink any dairy products." D. "Take at least one 2-hour nap per day.

B -"Be sure to drink adequate amounts of fluids each day." -Dehydration is the most common cause or contributing factor for hypercalcemia. Ingestion of reasonable amounts of dairy products does not increase the risk for hypercalcemia because high normal blood levels of calcium inhibit calcium absorption from the intestinal tract. Not allowing clients at risk for hypercalcemia to have any dairy products at all has not been proven effective for preventing hypercalcemia. Caffeine may decrease calcium levels, but this would not be an appropriate intervention.

Which question will the nurse ask the client to help determine the cause of hypokalemia? a. "Do you use sugar substitutes?" b. "Do you use diuretics or laxatives?" c. "Do you have any kidney disease?" d. "Have your bowel habits changed recently?"

B -"Do you use diuretics or laxatives?" -Misuse or overuse of diuretics, especially high-ceiling (loop) and thiazide diuretics, and laxatives are common causes of hypokalemia in older adults and clients with eating disorders.

Which of the following is an appropriate expected outcome for an adult client with well-controlled asthma? 1. Chest X-ray demonstrates minimal hyperinflation. 2. Temperature remains lower than 100 ° F (37. 8 ° C). 3. Arterial blood gas analysis demonstrates a decrease in PaO2. 4. Breath sounds are clear.

4. Between attacks, breath sounds should be clear on auscultation with good air flow present throughout lung fields. Chest X-rays should be normal. The client should remain afebrile. Arterial blood gases should be normal.

What action is most important for the nurse to teach the client who is at continued risk for complications from hypocalcemia? A. "Drink at least 3 L of fluids daily." B. "Use an electric shaver rather than a safety razor." C. "Wear gloves and stockings in cool or cold weather." D. "Remain in an upright position for at least 1 hour after a meal."

B -"Use an electric shaver rather than a safety razor." -Calcium is a cofactor at every step in the blood-clotting cascade. When hypocalcemia is present, blood takes longer to clot. Clients must be taught to avoid activities that can result in injury to prevent excessive bleeding or bruising.

Which subjective symptom will alert the nurse to the possibility of hypocalcemia? A. "I have a bowel movement only every 2 to 3 days." B. "Usually I wake up several times a night with painful cramps in my legs or feet." C. "My rings and shoes are much tighter fitting at night than they are in the morning." D. "I notice that my heart seems to pound whenever I climb steps or drink a cup of coffee."

B -"Usually I wake up several times a night with painful cramps in my legs." -Extracellular calcium is an excitable membrane stabilizer. Irritable skeletal muscles, as manifested by twitches and cramps, are an indication of mild hypocalcemia.

A client presents to the emergency department with severe dehydration and is ordered to receive 3 L of fluid over 6 hours. The nurse sets the intravenous pump at a rate of ______ mL/hr.

500

A client who is having a tunneled central venous catheter inserted begins to complain of chest pain and difficulty breathing. What action will the nurse take first?

B -Assists with insertion of a chest tube -An insertion-related complication of central venous catheters is a pneumothorax. Signs and symptoms of a pneumothorax include chest pain and dyspnea. The treatment includes removing the catheter, administering oxygen and placement of a chest tube. The pain is caused by the pneumothorax, which must be taken care of with a chest tube insertion. A sterile dressing and placement of the client in a Trendelenburg position are not indicated for the primary problem of a pneumothorax.

2. The client is taking a medication for an endocrine problem that inhibits aldosterone secretion and release. To what complications of this therapy should the nurse be alert? a. Dehydration, hypokalemia b. Dehydration, hyperkalemia c. Overhydration, hyponatremia d. Overhydration, hypernatremia

B -Dehydration, hyperkalemia -Aldosterone is a mineralocorticoid that increases the reabsorption of water and sodium in the kidney at the same time that it promotes excretion of potassium. Any drug or condition that disrupts aldosterone secretion or release increases the client's risk for excessive water loss and potassium reabsorption.

The client's urine specific gravity is 1.018. Which is the nurse's best action?

B -Documenting the finding as the only action -This specific gravity is within the normal range for urine

What is the initial action taken after completion of an intermittent medication administration through a Groshong catheter?

B -Flushing the line with saline -The Groshong catheter is not an implanted port. After intermittent use, the catheter is to be flushed with saline. The manufacturer's instructions state that the catheter should not be clamped to maintain the integrity of the catheter valve. If a heparin flush is ordered, it is given after the catheter has been flushed with saline. The access needle is used for implanted ports.

Which prevention strategy will the nurse teach the client with a risk for renal calculi? A. "Drink at least 3 to 4 L of fluid every day." B. "Avoid dairy products and other sources of calcium." C. "Avoid aspirin and aspirin-containing products." D. "Start taking antibiotics at the first sign of a stone."

A -"Drink at least 3 to 4 L of fluid every day." -Dehydration contributes to the precipitation of minerals to form a stone. Ingestion of calcium or aspirin does not cause a stone. Antibiotics neither prevent nor treat a stone.

After discontinuing a nontunneled, percutaneous central catheter, it is most important for the nurse to record which information? a. Application of a sterile dressing b. Length of the catheter c. Occurrence of venospasms d. Type of ointment used to seal the tract

B -Length of the catheter -After removal of a catheter, measure the catheter length and compare it with the length documented on insertion. If the entire length has not been removed, the nurse should contact the physician immediately because some of the catheter may still be in the client's vein.

Which condition would trigger the release of antidiuretic hormone (ADH)?

B -Plasma osmolarity increased secondary to dehydration -Antidiuretic hormone is triggered by a rising extracellular fluid (ECF) osmolarity, especially hypernatremia.

The nurse is caring for a client with HIV who has been prescribed didanosine (Videx EC). Which action by the nurse is most appropriate? a.Help the client plan specific meal and dosing times. b.Explain that the client will have frequent complete blood counts (CBCs) drawn. c.Advise the client to take Videx EC with milk or a small meal. d.Tell the client to take Tylenol (acetaminophen) for any abdominal pain.

A. Help the client plan specific meal and dosing times

Why is controlling blood glucose levels important? A. High blood glucose levels increase the risk for heart disease, strokes, blindness, and kidney failure. B. High blood glucose levels increase the risk for seizure disorders, arthritis, osteoporosis, and bone fractures. C. Low blood glucose levels increase the risk for peripheral neuropathy, Alzheimer's disease, and premature aging. D. Low blood glucose levels increase the risk for obesity, pancreatitis, dehydration, and certain types of cancer.

A. High blood glucose levels increase the risk for heart disease, strokes, blindness, and kidney failure. Rationale: Persistent high blood glucose levels cause major changes in blood vessels that lead to organ damage, serious health problems, and early death. The long-term complications of diabetes include heart attacks, strokes, and kidney failure. In addition, diabetes is the main cause of foot and leg amputations and new-onset blindness.

The nurse is caring for a young client who ass acquired immune deficiency syndrome (AIDS) and a very low CD4+ cell count. The nurse is teaching the client how to avoid infection at home. Which statement by the client indicates that additional teaching is needed? a."I will let my sister clean my pet iguana's cage from now on." b."My brother will change the kitty litter box from now on." c."It will seem funny but I'll run my toothbrush through the dishwasher." d."I will not drink juice that has been sitting out for longer than an hour."

A. I will let my sister clean my pet iguana's cage from now on.

15. What is the priority problem for a client experiencing chemotherapy-induced anemia? a. Risk for injury related to fatigue b. Fatigue related to decreased oxygenation c. Body image problems related to skin color changes d. Inadequate nutrition related to anorexia

ANS: A Safety is always a client priority. The client who is anemic will be fatigued and may need assistance with activity to prevent injury. The other problems may apply; however, they do not take priority over safety.

A client who is 2 days post-femoral vein cannulation begins to have difficulty with outflow of blood during dialysis. For which complication does the nurse assess? a. Hematoma at cannula insertion site b. Infection c. Oliguria d. Skin necrosis at cannula insertion site

ANS: A The puncture site of the femoral vein is prone to hematoma formation because positioning the extremity can cause movement of the cannula and subsequent bleeding at the site. The hematoma can compress the cannula, decreasing flow through it. The other complications would not diminish outflow.

A client has hypokalemia. Which question by the nurse obtains the most information on a possible cause? a. "Do you use sugar substitutes?" b. "Do you use diuretics or laxatives?" c. "Do you have any kidney disease?" d. "Have your bowel habits changed recently?"

ANS: B Misuse and overuse of diuretics, especially high-ceiling (loop) and thiazide diuretics, and laxatives are common causes of hypokalemia in older adults and in clients with eating disorders. Sugar substitutes and bowel habits are not related to hypokalemia. The client with kidney disease would be more likely to have hyperkalemia.

A client is receiving continuous arteriovenous hemofiltration (CAVH). Which laboratory value does the nurse monitor most closely? a. Hemoglobin b. Glomerular filtration rate c. Sodium d. White blood cells

ANS: C CAVH is used for clients who have fluid volume overload. It continuously removes large quantities of plasma, water, waste, and electrolytes, such as sodium. Fluid removal can also affect the serum sodium level.

A middle-aged client with diabetes mellitus is being treated for the third episode of acute pyelonephritis in the past year and asks what can be done to help prevent these infections. Which is the nurse's best response? a. "Test your urine daily for the presence of ketone bodies and proteins." b. "Use tampons rather than sanitary napkins during your menstrual period." c. "Drink more water and empty your bladder every 2 to 3 hours during the day." d. "Keep your hemoglobin A1c under 9% by keeping your blood sugar controlled."

ANS: C Clients with long-standing diabetes mellitus are at risk for pyelonephritis for many reasons. Chronically elevated blood glucose levels spill glucose into the urine, changing the pH and providing a favorable climate for bacterial growth. The neuropathy associated with diabetes reduces bladder tone and reduces the client's sensation of bladder fullness. Thus, even with large amounts of urine, the client voids less frequently, allowing stasis and overgrowth of microorganisms. Increasing fluid intake (specifically water) and voiding frequently prevent stasis and bacterial overgrowth. Testing urine and wearing tampons will not help prevent pyelonephritis. A hemoglobin A1c of 9% is too high.

The nurse is assessing a client's susceptibility to rejecting a transplanted kidney. Which result does the nurse recognize as increasing the client's chances of rejection? a. Decreased T-lymphocyte helper b. Decreased white blood cell count c. Increased cytotoxic-cytolytic T cell d. Increased neutrophil count

ANS: C Cytotoxic-cytolytic T cells function to attack and destroy non-self-cells, specifically virally infected cells and cells from transplanted grafts and organs. A high level of these cells would increase the chances of rejection. Decreased white blood cells would indicate immune suppression. Neutrophils are increased during an infection. pg 857

A client has end-stage kidney disease (ESKD). Which food selection by the client demonstrates understanding of a low-sodium, low-potassium diet? a. Bananas b. Ham c. Herbs and spices d. Salt substitutes

ANS: C Herbs and spices can be used in place of salt to enhance food flavor. Bananas are high in potassium. Ham is high in sodium. Many salt substitutes contain potassium chloride and should not be used.

Which staff member does the charge nurse assign to care for a client newly diagnosed with chronic kidney disease? a. Licensed practical nurse who usually works on the unit b. Registered nurse floated from the hemodialysis unit c. Registered nurse who has taken care of this client before d. Registered nurse with the most years of experience

ANS: C Provide continuity of care, whenever possible, by using a consistent nurse-client relationship to decrease anxiety and promote discussion of concerns.

A client was just admitted to the emergency department for new-onset confusion. As the nurse starts the IV line, the client says he just finished a hemodialysis session. The IV site is bleeding briskly. What action by the nurse takes priority? a. Assess for a bruit and thrill over the vascular access site. b. Draw blood for coagulation studies and white blood cell count. c. Prepare to administer protamine sulfate. d. Hold constant firm pressure with a gauze pad for 5 minutes.

ANS: C To prevent blood clots from forming within the dialyzer or blood tubing, anticoagulation is needed during hemodialysis treatment. The drug used is heparin, which makes the client at risk for hemorrhage for the next 4 to 6 hours. Protamine sulfate is the antidote to heparin, and the nurse should prepare to administer it. Pressure may help, and someone else can apply it while the nurse is getting the medication. Laboratory studies are not needed because the client is at known risk for bleeding from heparin. Assessing the vascular access device does nothing to help the situation.

A client with autosomal dominant polycystic kidney disease (ADPKD) asks whether his children could develop this disease. Which is the nurse's best response? a. "No genetic link is known, so your children are not at increased risk." b. "The disease is sex linked, so only your sons could be affected." c. "Both you and your wife must have the disease for your children to develop it." d. "Each of your children has a 50% risk of having ADPKD."

ANS: D ADPKD is transmitted as an autosomal dominant trait and therefore is not gender specific. Children whose parents have the autosomal dominant form of PKD have a 50% chance of inheriting the gene that causes the disease.

A client has a large renal calculus. Which assessment finding may indicate the development of a complication? a. Blood pressure of 178/94 mmHg b. Urine output of 5600 mL/24 hr c. Client reports of pain on urination d. Asymmetric, tender flank area

ANS: D Hydronephrosis, indicated by an asymmetric flank with tenderness, is commonly caused by obstruction such as a renal calculus. As the kidney continues to make urine, the volume of urine backs up into the kidney, increasing pressure, and the kidney is enlarged as a result. An asymmetric tender flank would be one manifestation of this condition. Polyuria, dysuria, and hypertension are not complications associated with renal calculi.

In planning care for a client with renal cell carcinoma, the nurse monitors for which electrolyte imbalance? a. Hyponatremia b. Hypernatremia c. Hypocalcemia d. Hypercalcemia

ANS: D Renal cell carcinoma tissues frequently produce ectopic hormones, including parathyroid hormone. Increased production of parathyroid hormone leads to decreased renal excretion of calcium and increased serum calcium concentration. The other electrolyte abnormalities typically do not occur.

The RN has assigned a client with a newly placed arteriovenous (AV) fistula in the right arm to an LPN. Which information about the care of this client is most important for the RN to provide to the LPN? a. "Avoid movement of the right extremity." b. "Place gentle pressure over the fistula site after blood draws." c. "Start any IV lines below the site of the fistula." d. "Take blood pressure in the left arm."

ANS: D Repeated compression of a fistula site can result in loss of vascular access. Therefore, avoid taking blood pressures and performing venipunctures or IV placement in the arm with the vascular access. The other statements are not appropriate.

The nurse is teaching a client who has AIDS how to avoid infection at home. Which statement indicates that additional teaching is needed? a."I will wash my hands whenever I get home from work." b."I will make sure to have my own tube of toothpaste at home." c."I will run my toothbrush through the dishwasher every evening." d."I will be sure to eat lots of fresh fruits and vegetables every day.

D. I will be sure to eat lots of fresh fruits and vegetables every day.

An HIV positive client verbalizes concerns about the high cost of antiretroviral medications. What is the nurse's best response? a."The medications are actually less expensive than they used to be." b."These medications are the best course of treatment for you." c."You should be glad the medications will help prolong your life." d."Let's talk to the social worker about getting financial assistance for you."

D. Let's talk to the socal worker about getting financial assistance for your

The nurse has been exposed to HIV through splashing of urine from a client who is HIV positive with a low viral load. The urine came into contact with the nurse's face. Which drug regimen does the nurse prepare to initiate? a.Retrovir (zidovudine) for 14 days b.Retrovir (zidovudine) for 28 days c.Retrovir (zidovudine) and Epivir (lamivudine) for14 days d.Retrovir (zidovudine) and Epivir (lamivudine) for 28 days

D. Rerovir (zidovudine) and Epivir (lamivudine) for 28 days

The nurse is caring for an HIV positive client. What assessment finding assists the nurse in confirming progression of the client's diagnosis to AIDS a.Generalized lymphadenopathy b.HIV-positive status for 8 years c.Low-grade fever for the last 10 days d.Thick white patches on the client's tongue

D. Thick white patches on the client's tongue

The nurse is working in a clinic when a young male client presents with reports of pain and urination. The client wants testing for sexually transmitted diseases. The nurse notes that the client's eyes are red and inflamed. What question by the nurse is most important?

Do you have any new joint pain

The nurse is working in a primary care clinic and sees a young male client. The client is athletic and is well over 6 feet tall, with size 14 shoes. What diagnostic test does the nurse facilitate for the client?

Echocardiography

The nurse is caring for a female client who has a history of chronic fatigue syndrome. Which finding is the nurse surprised to see in the client's record?

Hemoglobin, 7.2 g/dL

(Select All) The nurse is teaching a client with rheumatoid arthritis about joint protection principles. What information does the nurse include?

Hold objects with two hands, not one Use assistive adaptive devices Bend at your knees to lift objects

The nurse is instructing a client about the management of systemic sclerosis. Which statement indicates that the client requires additional teaching?

Ice packs will help relieve the aching pain in my hips and knees.

The nurse is caring for an older adult client who will be discharged after being hospitalized for a total hip replacement. Which statement indicates that arrangements may have to be made to have the client's medications supervised at home?

If my legs get swollen, I will take an extra Coumadin pill that day.

The nurse is caring for an older adult client who will be discharged home to live with an adult daughter. The client will be given prescriptions for four new medications for rheumatoid arthritis. How does the nurse ensure that the client will be able to take the medications correctly at home?

Include the clint's daughter when teaching the client about the medications.

(Select All) After hip replacement surgery, a client receives two doses of enoxaprain (Lovenox) during the day shift. What orders does the nurse anticipate for the client?

Laboratory draw for platelet count Order for protamine sulfate

Which statement by a client indicates that additional teaching is needed in the management of fibromyalgia?

Limiting my physical activity will reduce my fatigue

A client is suspected to have rheumatoid arthritis. Which manifestations does the nurse assess this client carefully for?

Low-grade fever, fatigue, anorexia with weight loss

The school nurse is working with a group of high school students who will be going on a field trip to a nature center. Which student is at highest risk for a tick bite?

Male student wearing a long-sleeved shirt and shorts.

A client with rheumatoid arthritis had abdominal surgery and has returned to the postoperative nursing unit. The client is unable to use the incentive spiromter correctly, demonstrating limited lung volume and fatiguing easily. What action by the nurse takes priority?

Notify the physician immediately

The home care nurse is making a follow-up visit to a client who had total hip replacement surgery 2 weeks ago. Which client statement indicates a need for clarification regarding postoperative routine?

Now tha my hip doesn't hurt, I can cross my legs like a lady again.

The nurse is instructing a client about management of discoid lupus erythematosus. Which statement indicates that the client requires additional teaching?

Steroids weaken the immune system, so I will wash my hands frequently.

A female client with rheumatoid arthritis has taken Rheumatrex (methotrexate) for the past year to control her symptoms. The client comes to the clinic and tells the nurse that a home pregnancy test was positive. What is the nurse's best response?

Stop taking Rheumatrex immediately. I'll tell the physician you are pregnant.

The nurse is caring for an older adult client who has had a hip replacement 2 days previously. Which assessment finding is the best indicator that the client does not need pain medication at this time?

The client states that she has no pain

A client with chronic gout take probenecid (Benemid) and comes to the clinic reporting frequent severe headaches and a new gout flare. The client is frustrated because the gout had been under good control Which question by the nurse is most helpful?

What do you take for your headaches?

Four clients are scheduled for surgery. Which client does the nurse determine is at highest risk for postsurgical complications? a. 89-year-old scheduled for a knee replacement b. 40-year-old requiring gallbladder surgery c. 19-year-old requiring a laparoscopy d. 10-year-old admitted for a tonsillectomy

a. 89-year-old scheduled for a knee replacement

The nurse is monitoring a client newly diagnosed with DM for signs of complications. Which sign would indicate hyperglycemia? a. Polyuria b. Diaphoresis c. Hypertension d. Increased pulse rate

a. Polyuria Rationale: Remember the 3 P's with Hyperglycemia - Polyuria, Polydipsia, Polyphagia

pt has a K 3.1 and digoxin. what is intervention

administer digbind

What recently learned information about a client who is scheduled to have surgery within the next 2 hours is the nurse certain to communicate to the surgical team? a. An allergy to cats b. Hearing problem c. Consumption of a glass of wine 12 hours ago d. Taking 2000 mg of vitamin C each day

b. Hearing problem

The home care nurse visits an older client with diabetes. For which nutritional problem does the nurse monitor this client? a. Obesity b. Malnutrition c. Alcoholism d. Hyperglycemia

b. Malnutrition Rationale: Older adults are more at risk for developing malnutrition as a result of multiple factors. Inadequate income, poor dentition, decreased cognition, decreased motor ability, depression, and lack of understanding about which foods constitute an adequate diet all contribute to an increased risk for malnutrition in all older adult clients, including those with diabetes mellitus.

A client with a history of diabetes mellitus has new onset of microalbuminuria. Which component of the diet must the client reduce? a. Percentage of total calories derived from carbohydrates b. Percentage of total calories derived from proteins c. Percentage of total calories derived from fats d. Total caloric intake

b. Percentage of total calories derived from proteins Rationale: Restriction of dietary protein to 0.8 g/kg body weight/day is recommended for clients with microalbuminuria to retard progression to renal failure. The other options would not be needed.

The nurse has given a client an injection of glucagon. Which action does the nurse take next? a. Apply pressure to the injection site. b. Position the client on his or her side. c. Have a padded tongue blade available. d. Elevate the head of the bed.

b. Position the client on his or her side. Rationale: Glucagon administration often induces vomiting, increasing the client's risk for aspiration. The other actions are not required.

A client receiving preoperative medication tells the nurse that she took all the following vitamins and herbs last night before going to bed. Which one does the nurse report to the surgical team as a priority? a. Valerian root b. St. John's wort c. Garlic d. Chamomile

c. Garlic

The nurse is caring for a client who will be undergoing emergency surgery as soon as possible. Which information is most important for the nurse to teach the client at this time? a. How the surgery will be performed b. Importance of early ambulation after surgery c. What to expect in the operating and recovery rooms d. Complications that may occur after surgery

c. What to expect in the operating and recovery rooms

nurse caring for client with hyperparathyroidism and notes clients valium level at 13. which med should nurse prepare to administer

calcitonin

a patient with the nursing diagnosis of fluid volume deficit is at risk for

falls

a patient with tachycardia, altered mental status and hypotension has this imbalance

fluid volume deficient

nursing caring for client who has a serum sodium level of 156. which should nurse anticipate

initiate seizure precautions

IV infusion therapy correct

order, rate, infusion and solution

nurse reviews a clients electrolyte results and notes K of 5.5. the nurse understands a K value at this level would be notes with which contain

traumatic burn

most appropriate intervention to prevent central line infection

using aseptic technique when providing care using the line

A client who is positive for HIB presents with confusion, fever, headache, blurred vision, nausea, and vomiting. What does the nurse do first? a.Assess the client's deep tendon reflexes. b.Ask the client to place his chin on his chest. c.Start an IV line with normal saline. d.Assess the client's pupil reaction

B. Ask the client to place his chin on his chest.

A client has selective immune globulin A (IgA) deficiency. The provider orders an infuson of immune globulin (IVIG). Which action by the nurse is best a.Start a second IV line for the client's antibiotics. b.Call the physician to clarify the order. c.Review the client's renal panel before administration. d.Obtain baseline vital signs and another set after 15 minutes.

B. Call the physician to clarify the order

An HIV positive client is taking lopinavir/ritonavir (Kaletra) and reports nausea, abdominal pain, and diarrhea. What orders does the nurse anticipate? a.Renal function studies b.Liver enzymes c.Blood glucose monitoring d.Albumin and prealbumin

B. Liver enzymes

The client at risk for continuing hyperkalemia states that she is upset because she cannot eat fruit every day. What is the nurse's best response? a. "You are correct. Fruit is usually very high in potassium." b. "If you cook the fruit first, that lowers the potassium." c. "Berries, cherries, apples, and peaches are low in potassium." d. "Fresh fruit is higher in potassium than dried fruit."

C -"Berries, cherries, apples, and peaches are low in potassium." -Not all fruit is potassium-rich. Fruits that are relatively low in potassium and can be included in the diet include apples, apricots, berries, cherries, grapefruit, peaches, and pineapples. Fruits high in potassium include bananas, kiwi, cantaloupe, oranges, and dried fruit.

The client with bladder cancer is scheduled to have intravesical chemotherapy. Which statement made by the client indicates correct understanding of this therapy?

C -"I will have few, if any, side effects from this type of chemotherapy." -Intravesical chemotherapy involves instilling the chemotherapy agents directly into the bladder. The side effects are local, not systemic.

The client is receiving 100 mL of dextrose 20% in water with 20 units of regular insulin. The client is slightly confused, has a weak hand grasp, and is pale and sweaty. What is the nurse's first intervention?

C -Assessing fingerstick blood glucose -Intravenous fluids containing insulin place the client at risk for hypoglycemia as well as hypokalemia. The client's clinical manifestations could indicate hypoglycemia and/or hypokalemia. Because hypoglycemia can be life-threatening, the nurse should intervene for this first. The nurse's first action will be to assess a fingerstick blood sugar. If the client's blood sugar is low, appropriate intervention must be taken. Stopping the current IV solution but maintaining IV access is critical. Although potassium may be given as a small infusion (100 to 150 mL) of IV fluid, it is not given by IV push.

The client has an elevated blood urea nitrogen (BUN) level and an increased ratio of blood urea nitrogen to creatinine. What is the nurse's interpretation of these laboratory results? a. The client probably has a urinary tract infection. b. The client may be overhydrated. c. The kidney may be hypoperfused. d. The kidney may be damaged.

C -The kidney may be hypoperfused. -When dehydration or renal hypoperfusion exist, the BUN level rises more rapidly than the serum creatinine level, causing the ratio to be increased, even when no renal dysfunction is present.

A client with HIV who is taking highly active antiretroviral therapy (HAAART) medications is in radiology waiting for a chest x-ray when medications are due. What action by the nurse is best a.Call radiology to see when the client will be brought back to the nursing unit. b.Send the nursing assistant to radiology to bring the client back to the nursing unit. c.Take the client's medications to radiology and administer them there if possible . d.Stagger the next dose of the medication if the current dose is given late.

C. Take the client's medications to radiology and administer them there if possible.

When obtaining a sexual history from a client in a clinic setting, the nurse notes that the client appears very uncomfortable and pauses for long periods before answering the nurse's questions. What is the nurse's best response? a."I am sorry that my questions are making you very uncomfortable." b."Don't worry. We'll be done with these questions in no time at all." c."Take your time. I realize that this is a very private topic to talk about." d."These questions are making you uncomfortable, so we'll finish next time."

C. Take your time. I realize that this is a very private topic to talk about.

The nurse is working with a client at a public health clinic. The client says to the nurse, The doctor said that my CD4+ count is 450. Is that good?" What is the nurse's best response? a."Your count is high so you can cut back on your medication." b."Your count is normal because your medications are working well." c."Your count is a bit low and you are susceptible to infection." d."Your count is very low and you actually now have AIDS."

C. Your count is a bit low and you are susceptible to infection.

A client had a total knee replacement earlier in the day and has a continuous femoral nerve blockade. When entering the room to assess the client, the nurse notes that the television volume is quite loud. The client explains that it is hard to hear with "all the ringing in my ears." What action by the nurse takes priority?

Call another nurse to notify the anesthesiologist immediately.

A client is admitted for a total hip replacement. Past medical history includes diabetes mellitus type2, a heart attack 5 years ago, and allergies to sulfa drugs. The client currently takes insulin on a sliding scale and celecoxib (Celebrex). Before administering the client's medications, which action by the nurse is most appropriate?

Call the physician to clarify the order

symptom of hypocalcemia

Chvostek

The nurse has taught a client with lupus about skin protection in the clinic. Later, the nurse sees the client at an outdoor music festival. Which observation by the nurse indicates that the client requires further instruction?

Client is discussing her new perm.

The nurse is working a t a clinic, where several clients are waiting to be seen. Which client does the nurse assess first?

Client with temporal arteritis with new onset of blurry double vision

A nurse is caring for a client who has had rheumatoid arthritis for 5 years. Which laboratory value requires the most immediate intervention by the nurse?

Creatinine, 3.2 mg/dL

Which assessment finding obtained while taking the history of an older adult client alerts the nurse that the client needs further assessment for fluid or electrolyte imbalance? a. "I am often cold and need to wear a sweater." b. "I seem to urinate more when I drink coffee." c. "In the summer, I feel thirsty more often." d. "My rings seem to be tighter this week."

D -"My rings are tighter this week." -A change in ring size over a relatively short period of time may indicate a change in body fluid amount or distribution rather than a change in body fat. The other statements are not indicators of a fluid or electrolyte imbalance.

The client is scheduled to have renography (kidney scan). She is concerned about discomfort during the procedure. Which is the nurse's best response?

D -"The only pain associated with this procedure is a small needle stick when you are given the radioisotope." -The test involves an intravenous injection of the radioisotope and the subsequent recording of the emission by a scintillator.

Which client is at greatest risk for a fungal urinary tract infection?

D -A middle-aged man with diabetes mellitus -Clients who are severely immunocompromised or who have diabetes mellitus are more prone to fungal urinary tract infections. The client with an enlarged prostate gland would not be at greater risk. Being sexually active does not place the client at greater risk for developing an infection.

Which client is at greatest risk for the development of hyperkalemia? a. Client with heart failure using a salt substitute b. Client taking a thiazide diuretic for hypertension c. Client taking nonsteroidal anti-inflammatory drugs daily d. Client with type 2 diabetes taking an oral antidiabetic agent

A -Man with heart failure using a salt substitute -Many salt substitutes are composed of potassium chloride. Heavy use can contribute to the development of hyperkalemia. The client should be taught to read labels and choose a salt substitute that does not contain potassium. Nonsteroidal anti-inflammatory drugs (NSAIDs) promote the retention of sodium but not potassium.

Which client is at greatest risk for developing hyponatremia? a. Client who is NPO receiving intravenous D5W b. Client taking a sulfonamide antibiotic c. Client taking ibuprofen (Motrin) d. Client taking digoxin (Lanoxin)

A -Middle-aged adult client who is NPO and receiving D5W as the mainstay of intravenous therapy -D5W contains no electrolytes. Because the client is not taking any food or fluids by mouth, normal sodium excretion can lead to hyponatremia. The antibiotic, Motrin, and digoxin will not put a client at risk for hyponatremia

Which action by the nurse is most effect to prevent becoming exposed to the human immune deficiency virus (HIV)? a.Always use Standard Precautions with all clients in the workplace. b.Place clients who are HIV positive in Contact Precautions. c.Wash hands before and after contact with clients who are HIV positive. d.Convert parenteral medications to an oral form for clients who are HIV positive.

A. Always use standard precautions with all clients in the workplace

The nurse is caring for a young woman at the primary health care clinic. Which assessment finding leads the nurse to question the client about risk factors for HIV? a.Six vaginal yeast infections in the last 12 months b.Unable to become pregnant for the last 2 years c.Severe cramping and irregular periods d.Very heavy periods and breakthrough bleeding

A. Six vaginal yeast infections in the last 12 months.

A client who is receiving highly active antiretroviral therapy (HAART) tells the nurse, "The doctor said that my viral load is reduced. What does this mean?" What is the nurse's best response? a."The HAART medications are working well right now." b."You are not as contagious as you were anymore." c."Your HIV infection is becoming resistant to your medications." d."You are developing an opportunistic infection."

A. The HAART medications are working well right now.

A nursing assistant asks the nurse if respiratory isolation is needed for a client with Pneumocystis jiroveci pneumonia. What is the nurse's best response? a."This type of pneumonia is an opportunistic infection, so the staff is not at risk." b."You should wear a mask and a gown to provide care." c."Yes, please institute respiratory isolation because this is very contagious." d."You are not at risk for this infection if you have had a vaccination.

A. This type of pneumonia is an opportunistic infection, so the staff is not at risk

A client who had kidney trauma required a nephrectomy. What does the nurse teach the client about this condition? a. "You need to avoid participating in contact sports like football." b. "You probably will end up on dialysis a few years from now." c. "You need medication to control your high blood pressure from the injury." d. "You will always be required to restrict your salt and fluid intake."

ANS: A Clients with one kidney need to avoid contact sports because the kidneys are easily injured. The client will not end up on dialysis, have new hypertension, or be required to restrict salt and fluids because of the nephrectomy.

Which response by a client indicates an understanding of measures to facilitate the flow of peritoneal dialysate fluid? a. "I will take my stool softeners every day." b. "I will keep the drainage bag at the level of my abdomen." c. "Flushing the catheter is needed with each exchange." d. "Warmed dialysate infusion increases the speed of flow."

ANS: A Constipation is the primary cause of inflow and outflow problems. To prevent constipation, clients are placed on a bowel regimen before placement of a peritoneal catheter. The drainage bag should be lower than the abdomen. Warming the fluid helps prevent discomfort during the procedure. Flushing the catheter will not facilitate the flow of dialysate.

A client presents with painful, inflamed fingers with small, hard, yellow nodules that have a sandy yellow drainage. Which medication does the nurse prepare to administer to the client?

Allopurinol (Zyloprim)

A client is being treated for dehydration. Which statement made by the client indicates understanding of this condition? a. "I must drink a quart of water or other liquid each day." b. "I will weigh myself each morning before I eat or drink." c. "I will use a salt substitute when making and eating my meals." d. "I will not drink liquids after 6 PM so I won't have to get up at night."

B -"I will weigh myself every morning before I eat or drink." -Because 1 L of water weighs 1 kg, change in body weight is a good measure of excess fluid loss or fluid retention. A weight loss of more than 0.5 pound daily is indicative of excessive fluid loss. The other statements are not indicative of practices that will prevent dehydration.

The client is an older woman who is receiving treatment with levofloxacin (Levaquin). Which is the highest priority instruction that the nurse can provide to this client?

B -How to assess her own radial pulse -The client should assess her own radial pulse at least twice daily because this class of drugs can induce serious cardiac dysrhythmias. Assessment of blood pressure and respirations will not allow the client to detect if she is experiencing cardiac side effects of the medication. She should not attempt to assess her carotid pulse because a syncopal episode could result.

The female client's urinalysis shows all the following results. Which should the nurse document as abnormal?

B -Ketone bodies present -Ketone bodies are byproducts of incomplete metabolism of fatty acids. Normally, there are no ketones in urine. Ketone bodies are produced when fat sources are used instead of glucose to provide cellular energy.

A client is to receive 10 days of antibiotic therapy for urosepsis. The nurse should select which intravenous catheter to place?

B -Midline catheter -Midline catheters are used for therapies lasting from 1 to 4 weeks. Short peripheral catheters can be inserted by the nurse to use for antibiotic therapy, but can only stay in for up to 96 hours. If the length of intravenous therapy is longer than 6 days, a midline catheter should be chosen. Nontunneled central catheters and Hickman catheters are inserted by a physician.

The nurse is teaching a client how to prevent transmitting HIV to his sexual partner. Which statement by the client indicates that additional teaching is needed a."I can throw the condoms in the trash after I have used them." b."I will store my condoms in my wallet so they are always handy." c."Water-based lubricants are best to prevent condom breakage." d."The condom needs to stay on until I withdraw my penis."

B. I will store my condoms in my wallet so they are always handy

The nurse is teaching a seminar about preventing the spread of HIV. Which statement by a student indicates tat additional teaching is required. a."A woman can still get pregnant if she is HIV positive." b."I won't get HIV if I only have oral sex with my partner." c."Showering after intercourse will not prevent HIV transmission." d."People with HIV are still contagious even if they take HAART drugs."

B. I won't get HIV if I only have oral sex with my partner

Two hours after a closed percutaneous kidney biopsy, the client reports a dramatic increase in pain. Which is the nurse's best first action? A. Repositioning the client on the operative side B. Administering prescribed opioid analgesic C. Assessing pulse rate and blood pressure D. Checking the Foley catheter for kinks

C -Assessing pulse rate and blood pressure -An increase in the intensity of pain after a percutaneous kidney biopsy is a symptom of internal hemorrhage.

The nurse assesses distended neck veins in a client sitting in a chair to eat. What intervention is the nurse's priority? a. Document the observation in the chart. b. Measure urine specific gravity and volume. c. Assess the pulse and blood pressure. d. Assess the client's deep tendon reflexes.

C -Assessing the pulse and blood pressure -Neck veins in the normovolemic person are full in the supine position and flat in the sitting position. Full neck veins in the sitting position are an indicator of overhydration. Checking the pulse and blood pressure can help determine if overhydration is present.

The nurse is working with a client who has AIDS related dementia and will soon be discharged to the care to family members. What teaching topic is best for the nurse to include in the discharge plans? a.Feed the client when he will not do it by himself. b.Make sure that a clock and a calendar are easily visible. c.Remove locks from bathroom and bedroom doors. d.Do not allow the client to smoke when he is alone

B. Make sure that a clock and a calendar are easily visible

The nursing supervisor is working with an HIV positive nurse who has open weeping blisters on her arms after being exposed to poison ivy. Which instructions should the nursing supervisor provide to the nurse before she starts her shift? a."You should reassure your clients that you are not contagious." b."You should work phone triage at the desk today rather than taking clients." c."You should wear a long-sleeved scrub jacket today while working with clients." d."You should not care for clients who are immune compromised or in isolation."

B. You should work phone triage at the desk today rather than taking clients

Which client is at greatest risk for development of hypocalcemia?

C -Older adult client who has alcoholism and malnutrition -Calcium is absorbed from the gastrointestinal tract under the influence of vitamin D. When a client is malnourished, not only is the dietary intake of calcium usually low, but the client is also vitamin-deficient.

Which client is not a candidate for intermittent self catheterization training? A. Older female client B. Middle-aged female client who is blind C. Older male client with dementia D. Middle-aged client who has paraplegia

C -Older male client with dementia -Clients of any age with a variety of impairments and disabilities can participate in intermittent self-catheterization. The two main requirements are that the client be cognitively intact and can reach the area.

Which client is at greatest risk for hypokalemia?

C -The client with Cushing's disease -In Cushing's disease, the person has an excess of glucocorticoids, especially cortisol. Cortisol has some action of aldosterone, resulting in an increased reabsorption of sodium and water while enhancing the excretion of potassium. Therefore, any client with cortisol excess, whether from Cushing's disease or from exogenous cortisol, is at high risk for the development of hypokalemia.

What action will the nurse take when preparing to infuse packed red blood cells through a Groshong catheter?

D -Use a pump to infuse the blood. -The length of the peripherally inserted central catheter (PICC) adds resistance and may prevent the blood from infusing within the 4-hour limitation. Therefore, a pump is needed to ensure adequate flow rates. Infusion of packed red blood cells is considerably slower through a PICC such as the Groshong catheter. Therefore, it would be difficult to infuse the red blood cells within 1 hour. Normal saline solution is used as the priming solution with red blood cell administration. IV tubing containing filters should always be used when administering red blood cells.

Which intervention is most likely to be effective in stimulating the initiation of voiding for the client with overflow incontinence? A. Stroking the medial aspect of the thigh B. Using intermittent catheterization C. Providing digital anal stimulation D. Using the Valsalva maneuver

D -Using the Valsalva maneuver -In clients with overflow incontinence, the voiding reflex arc is not intact. Mechanical pressure, such as that achieved through the Valsalva maneuver (holding the breath and bearing down as if to defecate) can initiate voiding.

A client is reeiving warfarin (Coumadin) daily following toal hip replacement surgery. Which laboratory value requires intervention by the nurse?

International normalized ratio (INR), 5.1

A client in the emergency department has been diagnosed with ketoacidosis. Which manifestation does the nurse correlate with this condition? a. Increased rate and depth of respiration b. Extremity tremors followed by seizure activity c. Oral temperature of 102° F (38.9° C) d. Severe orthostatic hypotension

a. Increased rate and depth of respiration Rationale: Ketoacidosis decreases the pH of the blood, stimulating the respiratory control areas of the brain to buffer the effects of increasing acidosis. The rate and depth of respiration are increased (Kussmaul respirations) in an attempt to excrete more acids by exhalation.

A client is receiving IV insulin for hyperglycemia. Which laboratory value requires immediate intervention by the nurse? a. Serum chloride level of 98 mmol/L b. Serum calcium level of 8.8 mg/dL c. Serum sodium level of 132 mmol/L d. Serum potassium level of 2.5 mmol/L

d. Serum potassium level of 2.5 mmol/L Rationale: Insulin activates the sodium-potassium ATPase pump, increasing the movement of potassium from the extracellular fluid into the intracellular fluid, resulting in hypokalemia. In hyperglycemia, hypokalemia can also result from excessive urine loss of potassium. The chloride level is normal. The calcium and sodium levels are slightly low, but this would not be related to hyperglycemia and insulin administration.

The nurse is teaching a client with diabetes about self-care. Which activity does the nurse teach that can decrease insulin needs? a. Reducing intake of liquids to 2 L/day b. Eating animal organ meats high in insulin c. Limiting carbohydrate intake to 100 g/day d. Walking 1 mile each day

d. Walking 1 mile each day Rationale: Moderate exercise, such as walking, helps regulate blood glucose levels on a daily basis and results in lowered insulin requirements for clients with type 1 diabetes. Restricting fluids and eating organ meats will not reduce insulin needs. People with diabetes need at least 130 grams of carbohydrates each day.

The client scheduled for intravenous urography informs the nurse of the following allergies. Which one should the nurse report to the physician immediately? A. Seafood B. Penicillin C. Bee stings D. Red food dye

A - Seafood - Clients with seafood allergies often have severe allergic reactions to the standard dyes used during intravenous urography.

A client with AIDS has been admitted with fever, night sweats, and weight loss of 6 pounds in 2 weeks. The client's purified protein derivative (PPD) test, placed 3 days ago in the clinic, is negative. Which action by the nurse is most appropriate? a.Place the client in Airborne Precautions. b.Facilitate the client's chest x-ray. c.Initiate a 3-day calorie count. d.Start an IV of normal saline

A. Place the client in Airborne precautions.

The nurse is caring for a client who is HIV positive. The client has become confused over the course of the shift, and the client's pupils are no longer reacting to light equally. The nurse anticipates an order for which medication? a.Prednisone (Deltazone) b.Trimethoprim/sulfamethoxazole (Bactrim) c.Pentamidine isethionate (Pentam) d.Ketoconazole (Nizoral)

A. Prednisone (Deltazone)

The visiting nurse has many clients who are African American. Which intervention is most important for the nurse to accomplish when seeing these clients? a. Weigh the clients and compare their weights. b. Assess the clients' blood pressure. c. Observe the clients for any signs of abuse. d. Ask the clients about their medications.

ANS: B All interventions are important for the visiting nurse to accomplish. However, African Americans have a high rate of hypertension leading to end-stage renal disease. Each encounter that the nurse has with an African-American client provides a chance to detect hypertension and treat it. If the client is already on antihypertensive medication, assessing blood pressure monitors therapy.

A client is scheduled to have dialysis in 30 minutes and is due for the following medications: vitamin C, B-complex vitamin, and cimetidine (Tagamet). Which action by the nurse is best? a. Give medications with a small sip of water. b. Hold all medications until after dialysis. c. Give the supplements, but hold the Tagamet. d. Give the Tagamet, but hold the supplements.

ANS: B All three medications are dialyzable, meaning that they will be removed by the dialysis. They should be given after the treatment is over.

A client with acute kidney failure and on dialysis asks how much fluid will be permitted each day. Which is the nurse's best response? a. "This is based on the amount of damage to your kidneys." b. "You can drink an amount equal to your urine output, plus 700 mL." c. "It is based on your body weight and changes daily." d. "You can drink approximately 2 liters of fluid each day."

ANS: B For clients on dialysis, fluid intake is generally calculated to equal the amount of urine excreted plus 500 to 700 mL.

The nurse is assessing a client with acute kidney injury and hears the following sound when auscultating the lungs. For what complication does the nurse plan care? Audio Clip a. Cardiac tamponade b. Pericarditis c. Pulmonary edema d. Myocardial Infarction

ANS: B The sound heard is a pericardial friction rub. This is heard in pericarditis because the pericardial sac becomes inflamed from uremic toxins. Other manifestations include low-grade fever, tachycardia, and chest pain. A tamponade would manifest as muffled heart tones. Pulmonary edema would manifest with crackles in the lungs. A myocardial infarction may or may not have abnormal chest sounds associated with it.

A client who is admitted to the hospital with a history of kidney disease begins to have difficulty breathing. Vital signs are as follows: blood pressure, 90/70 mm Hg; heart rate, difficult to feel peripheral pulses. His heart sounds are difficult to hear. Which intervention does the nurse prepare for? a. Administration of digoxin (Lanoxin) b. Draining of pericardial fluid with a needle c. Emergency hemodialysis d. Placement of a pacemaker

ANS: B These signs and symptoms are of cardiac tamponade, an emergency situation in which fluid accumulates in the pericardial sac, making it difficult for the heart to pump normally. Treatment includes a pericardiocentesis, or withdrawing the fluid with a needle or catheter. The other interventions are not appropriate in this situation.

The nurse is providing dietary teaching to a client who was just started on peritoneal dialysis (PD). Which instruction does the nurse provide to this client regarding protein intake? a. "Your protein needs will not change, but you may take more fluids." b. "You will need more protein now because some protein is lost by dialysis." c. "Your protein intake will be adjusted according to your predialysis weight." d. "You no longer need to be on protein restriction."

ANS: B When renal disease has progressed and requires treatment with dialysis, increased protein is required in the diet to compensate for protein losses through peritoneal dialysis. The other statements are inaccurate.

Which action does the nurse teach a client to reduce the risk for dehydration? a. Restricting sodium intake to no greater than 4 g/day b. Maintaining an oral intake of at least 1500 mL/day c. Maintaining a daily oral intake approximately equal to daily fluid loss d. Avoiding the use of glycerin suppositories to manage constipation

C -Maintaining a daily oral intake approximately equal to daily fluid loss -Although a fixed oral intake of 1500 mL daily is good, the key to prevention of dehydration is to match all fluid losses with the same volume for fluid intake. This is especially true in warm or dry environments or when conditions result in a greater than usual fluid loss through perspiration or ventilation.

Which is an appropriate dietary choice for the client with uric acid renal calculi? A. Chicken salad sandwich, potato chips B. Chef salad, low sodium crackers C. Mixed green salad, melba toast D. Baked fish, steamed vegetables

C -Mixed green salad, melba toast -The only diet selection that does not contain any type of meat is the mixed green salad and melba toast. To reduce the client's level of uric acid, he or she must avoid any food that contains purine. This is found primarily in organ meats, poultry, and fish. This means that the client must avoid the chicken salad sandwich, chef salad (contains meat), and baked fish.

The client who has undergone a nephrolithotomy procedure 24 hours ago now has a fever of 101° F (38.3° C). Which is the nurse's priority intervention?

C -Notifying the physician -The elevated temperature indicates a possible infection. Treatment must be initiated as soon as possible to prevent septic complications.

The client with bladder cancer has undergone a complete cystectomy with ileal conduit. Four hours after the surgery, the nurse observes the stoma to be cyanotic. Which is the nurse's priority action?

C -Notifying the surgeon -A pale or cyanotic stoma indicates impaired circulation to the stoma and must be treated to prevent necrosis.

What intervention is most important to teach the client about identifying the onset of dehydration? a. Measuring abdominal girth b. Converting ounces to milliliters c. Obtaining and charting daily weight d. Selecting food items with high water content

C -Obtaining and charting accurate weight -Because 1 L of water weighs 1 kg, change in body weight is a good measure of excess fluid loss or fluid retention. Obtaining and charting accurate weight is the most sensitive and cost-effective way of monitoring fluid balance in the home.

A client is on a potassium-restricted diet. Which protein choice by the client indicates a good understanding of the dietary regimen? a. 1% or 2% milk b. Grilled salmon c. Poached eggs d. Baked chicken

C -Poached eggs -Meat and fish have cells that contain large amounts of potassium. Eggs contain few cells and have one of the lowest potassium contents among high-protein foods. Broccoli is not considered a source of protein.

The nurse is seeing clients at a drop in primary health clinic. Which client does the nurse teach about the risks of acquiring HIV? a.Middle-aged woman with a new sexual partner b.Young male who has male sexual partners c.All clients who come to the clinic d.Young woman having her first gynecologic examination

C. All clients who come to the clinic

The school nurse removes a tick embedded in a student's scalp by the hairline. Which follow-up instruction is the nurse sure to provide to the mother?

Call your pediatrician right away if a fever or a red rash develops at the bite.

The client with severe bacterial cystitis is prescribed to take cefadroxil (Duricef) and phenazopyridine (Pyridium). What will the nurse teach this client regarding the drug regimen? A. "Do not take these drugs with food or milk." B. "Stop these drugs if you think you are pregnant." C. "Do not be alarmed by the discoloration of your urine." D. "Drink a liter of cranberry juice every day."

C -"Do not be alarmed by the discoloration of your urine." -Phenazopyridine discolors urine most commonly to a deep reddish orange. Many clients think they have blood in their urine when they see this. In addition, the urine can permanently stain clothing.

Which assessment finding will alert the nurse to a worsening of the client's hyponatremia?

C -Bowel sounds are hyperactive in all abdominal quadrants. -Clinical manifestations of hyponatremia are most evident in excitable tissues and include lethargy, decreased blood pressure, increased gastric motility, and diminished deep tendon reflexes.

An older adult client is scheduled for knee replacement surgery. Which statement by the client indicates a need for further preoperative instruction?

I need to keep my leg positioned away from my body.

The nurse is teaching a community health class about health promotion techniques. Which statement by a student indicates a strategy to help prevent the development of osteoarthritis?

I will keep my BMI under 24

The nurse is caring for a client who is 1 day post total hip replacement. The nurse is instructing the client about how to perform quadriceps-setting exercises correctly. Which direction does the nurse provide to the client?

Straighten your legs and push the back of your knees into the mattress

The nurse is working with a client who will be taking 20 mg of prednisone daily for rheumatoid arthritis. Which precautions does the nurse give the client about taking this medication?

Take calcium and vitamin D supplements daily

A client who has had bilateral total knee replacements is prescribed enoxaprin sodium (Lovenox) injections twice daily for the next 3 weeks. The client asks the nurse why she has to have the medication. What is the nurse's best response?

To prevent the formation of blood clots in your legs.

A client has long-standing diabetes mellitus. Which finding alerts the nurse to decreased kidney function in this client? a. Urine specific gravity of 1.033 b. Presence of glucose in the urine c. Presence of ketone bodies in the urine d. Sustained elevation in blood pressure

d. Sustained elevation in blood pressure Rationale: Hypertension is both a cause of renal dysfunction and a result of renal dysfunction. Renal dysfunction often occurs in the client with diabetes. Glucose and ketones in the urine are consistent with diabetes mellitus but are not specific to renal function. Specific gravity is elevated with dehydration.

The nurse applies antiembolism stockings to a client preoperatively. When the client says that they are uncomfortably tight, what is the nurse's best action? a. Remove the stockings for an hour to relieve the pressure. b. Pull the stockings down so that they are not constricting. c. Measure the client's calf to ensure that they are the correct size. d. Teach the client the purpose of wearing the stockings.

d. Teach the client the purpose of wearing the stockings.

The nurse is performing preoperative teaching with an older adult client who will be having colon resection surgery the following day. The surgeon has ordered bowel preparation the night before. Which action is a priority? a. Administer antibiotics with a sip of water. b. Encourage the client to drink plenty of juice. c. Teach the client to eat only low-fat foods the night before surgery. d. Tell the client not to get up and go to the bathroom alone.

d. Tell the client not to get up and go to the bathroom alone.

The nurse is interviewing a client with Type 2 DM. Which stmt by the client indicates an understanding of the treatment for this disorder? a. "I take oral insulin instead of shots" b. "By taking these medications, I am able to eat more" c. "When I become ill, I need to increase the number of pills I take" d. The medications I'm taking help release the insulin I already make"

d. The medications I'm taking help release the insulin I already make" Rationale: Clients with Type 2 DM have decreased or impaired insulin secretion. Oral hypoglycemic agents are given to facilitate glucose uptake.

A client has a new insulin pump. Which is the nurse's priority instruction in teaching the client? a. "Test your urine daily for ketones." b. "Use only buffered insulin." c. "Keep the insulin frozen until you need it." d. "Change the needle every 3 days."

d. "Change the needle every 3 days." Rationale: Having the same needle remain in place through the skin for longer than 3 days drastically increases the risk for infection in or through the delivery system. Having an insulin pump does not require the client to test for ketones in the urine. Insulin should not be frozen. Insulin is not buffered.

a patient with kidney disease have stidors, tetany and spasm. what is priority

administer renagel

this complication involves leakage of vesicant IV solution

extravasation

A client uses a metered-dose inhaler (MDI) to aid in management of his asthma. Which action by the client indicates to the nurse that he needs further instruction regarding its use? Select all that apply. 1. Activation of the MDI is not coordinated with inspiration. 2. The client inspires rapidly when using the MDI. 3. The client holds his breath for 3 seconds after inhaling with the MDI. 4. The client shakes the MDI after use. 5. The client performs puffs in rapid succession.

1, 2, 3, 4, 5. Utilization of an MDI requires coordination between activation and inspiration; deep breaths to ensure that medication is distributed into the lungs, holding the breath for 10 seconds or as long as possible to disperse the medication into the lungs, shaking up the medication in the MDI before use, and a sufficient amount of time between puffs to provide an adequate amount of inhalation medication.

The nurse is teaching the client how to use a metered-dose inhaler (MDI) to administer a corticosteroid. Which of the following client actions indicates that he is using the MDI correctly? Select all that apply. 1. The inhaler is held upright. 2. The head is tilted down while inhaling the medicine. 3. The client waits 5 minutes between puffs. 4. The mouth is rinsed with water following administration. 5. The client lies supine for 15 minutes following administration.

1, 4. The client should shake the inhaler and hold it upright when administering the drug. The head should be tilted back slightly. The client should wait about 1 to 2 minutes between puffs. The mouth should be rinsed following the use of a corticosteroid MDI to decrease the likelihood of developing an oral infection. The client does not need to lie supine; instead, the client will likely to be able to breathe more freely if sitting upright.

A client is prescribed metaproterenol (Alupent) via a metered-dose inhaler, two puffs every 4 hours. The nurse instructs the client to report adverse effects. Which of the following are potential adverse effects of metaproterenol? 1. Irregular heartbeat. 2. Constipation. 3. Pedal edema. 4. Decreased pulse rate.

1. Irregular heartbeats should be reported promptly to the care provider. Metaproterenol (Alupent) may cause irregular heartbeat, tachycardia, or anginal pain because of its adrenergic effect on beta-adrenergic receptors in the heart. It is not recommended for use in clients with known cardiac disorders. Metaproterenol does not cause constipation, pedal edema, or bradycardia.

Bed rest is prescribed for a client with pneumonia during the acute phase of the illness. The nurse should determine the effectiveness of bed rest by assessing the client's: 1. Decreased cellular demand for oxygen. 2. Reduced episodes of coughing. 3. Diminished pain when breathing deeply. 4. Ability to expectorate secretions more easily.

1. Exudate in the alveoli interferes with ventilation and the diffusion of gases in clients with pneumonia. During the acute phase of the illness, it is essential to reduce the body's need for oxygen at the cellular level; bed rest is the most effective method for doing so. Bed rest does not decrease coughing or promote clearance of secretions, and it does not reduce pain when taking deep breaths.

Which of the following is an expected outcome of pursed-lip breathing for clients with emphysema? 1. To promote oxygen intake. 2. To strengthen the diaphragm. 3. To strengthen the intercostal muscles. 4. To promote carbon dioxide elimination.

4. Pursed-lip breathing prolongs exhalation and prevents air trapping in the alveoli, thereby promoting carbon dioxide elimination. By prolonging exhalation and helping the client relax, pursed-lip breathing helps the client learn to control the rate and depth of respiration. Pursed-lip breathing does not promote the intake of oxygen, strengthen the diaphragm, or strengthen intercostal muscles.

A nurse notes that a client has kyphosis and generalized muscle atrophy. Which of the following problems is a priority when the nurse develops a nursing plan of care? 1. Infection. 2. Confusion. 3. Ineffective coughing and deep breathing. 4. Difficulty chewing solid foods.

3. In kyphosis, the thoracic spine bends forward with convexity of the curve in a posterior direction, making effective coughing and deep breathing difficult. Although the client may develop other problems because respiratory status deteriorates when pulmonary secretions are not adequately cleared from airways, ineffective coughing and deep breathing should receive priority attention.

The nurse administers theophylline (Theo-Dur) to a client. To evaluate the effectiveness of this medication, which of the following drug actions should the nurse anticipate? 1. Suppression of the client's respiratory infection. 2. Decrease in bronchial secretions. 3. Relaxation of bronchial smooth muscle. 4. Thinning of tenacious, purulent sputum.

3. Theophylline (Theo-Dur) is a bronchodilator that is administered to relax airways and decrease dyspnea. Theophylline is not used to treat infections and does not decrease or thin secretions.

A client who has been taking flunisolide (AeroBid), two inhalations a day, for treatment of asthma.has painful, white patches in his mouth. Which response by the nurse would be most appropriate? 1. "This is an anticipated adverse effect of your medication. It should go away in a couple of weeks." 2. "You are using your inhaler too much and it has irritated your mouth." 3. "You have developed a fungal infection from your medication. It will need to be treated with an antifungal agent." 4. "Be sure to brush your teeth and floss daily. Good oral hygiene will treat this problem."

3. Use of oral inhalant corticosteroids such as flunisolide (AeroBid) can lead to the development of oral thrush, a fungal infection. Once developed, thrush must be treated by antifungal therapy; it will not resolve on its own. Fungal infections can develop even without overuse of the corticosteroid inhaler. Although good oral hygiene can help prevent development of a fungal infection, it cannot be used alone to treat the problem.

The hospitalized client with a urethral retention catheter has cystitis. Which is the priority nursing diagnosis for this client? A. Risk for Infection B. Disturbed Body Image C. Risk for Impaired Skin Integrity D. Risk for Urge Urinary Incontinence

A -Risk for Infection -The most common cause of sepsis in hospitalized clients is a urinary tract infection. Ascending infections from cystitis with an indwelling catheter is a major source of such infections. Although the other diagnoses are important, they would not have life-threatening implications for the client.

A client with acute kidney injury had normal assessments 1 hour ago. Now the nurse finds that the client's respiration rate is 44 breaths/min and the client is restless. Which assessment does the nurse perform? a. Obtain an oxygen saturation level. b. Send blood for a creatinine level. c. Assess the client for dehydration. d. Perform a bedside blood glucose.

ANS: A A complication of acute kidney injury is pulmonary edema. Manifestations of this include tachypnea; frothy, blood-tinged sputum; and tachycardia, anxiety, and crackles. The nurse needs to obtain an oxygen saturation, listen to the client's lungs, and notify the health care provider, so that treatment can be started. The other interventions are not helpful.

The nurse is obtaining the health history of a client who has iron deficiency anemia. Which factor in this client's history does the nurse correlate with this diagnosis? a. Eating a meat-free diet b. Family history of sickle cell disease c. History of leukemia d. History of bleeding ulcer

ANS: A A diet high in protein and iron helps keep the client's levels of iron within normal limits. Meat is a good source of protein and iron. A bleeding ulcer could cause anemia but would not cause iron deficiency. Sickle cell disease causes sickle cell anemia. Leukemia causes a decrease in white blood cells.

24. A client is receiving high-dose chemotherapy for multiple myeloma. Which intervention is most important for the nurse to implement to prevent complications during chemotherapy? a. Ensure that the client's fluid intake is 3000 to 5000 mL/day. b. Monitor telemetry every hour during therapy. c. Apply pressure to all injection sites for 5 minutes. d. Assist the client in all ambulatory activities.

ANS: A This client is at high risk for tumor lysis syndrome. Tumor lysis syndrome is the precipitation of intracellular products released when tumor cells are destroyed rapidly. These products, particularly purines, can increase uric acid crystal precipitation in the kidney tubules and may cause acute tubular necrosis. In addition, serum potassium levels can become high. Maintaining adequate hydration and urine output is essential in preventing complications.

The nurse is assessing a client whose warfarin (Coumadin) therapy was discontinued 3 weeks ago. Which laboratory test result indicates that the client's warfarin therapy is no longer therapeutic? a. International normalized ratio (INR), 0.9 b. Reticulocyte count, 1% c. Serum ferritin level, 350 ng/mL d. Total white blood cell (WBC) count, 9000/mm3

ANS: A Warfarin therapy increases the INR. Normal INR ranges between 0.7 and 1.8. Therapeutic warfarin levels, depending on the indication of the disorder, should maintain the INR between 1.5 and 3.0. When the effects of warfarin are no longer present, the INR returns to normal levels. Warfarin therapy does not affect white blood cell count, serum ferritin level, or reticulocyte count. pg 866

The nurse monitors for which clinical manifestations in a client with nephrotic syndrome? (Select all that apply.) a. Proteinuria, >3.5 g/24 hr b. Hypoalbuminemia c. Dehydration d. Lipiduria e. Dysuria f. Costovertebral angle (CVA) tenderness

ANS: A, B, D Nephrotic syndrome is caused by glomerular damage and is characterized by proteinuria (protein level higher than 3.5 g/24 hr), hypoalbuminemia, edema, and lipiduria. CVA tenderness is present with inflammatory changes in the kidney. Dysuria is present with cystitis.

A client asks the nurse, "What are the advantages of peritoneal dialysis over hemodialysis?" Which response by the nurse is accurate? (Select all that apply.) a. "It will give you greater freedom in your scheduling." b. "You have less chance of getting an infection." c. "You need to do it only three times a week." d. "You do not need a machine to do it." e. "You will have fewer dietary restrictions."

ANS: A, D, E Although peritoneal dialysis is slower than hemodialysis, it does not require a specially trained registered nurse and can be done at home, allowing for greater flexibility in scheduling. Peritoneal dialysis is ambulatory, and a machine is not needed. Nursing implications for hemodialysis include vascular access care and diet restrictions, whereas peritoneal dialysis allows for a more flexible diet (abdominal catheter care is still necessary).

33. The nurse is caring for a client who has a sealed radiation implant for cervical cancer. Which activities by the nurse are appropriate? (Select all that apply.) a. Inform the supervisor of the nurse's positive pregnancy test. b. Obtain the dosimeter badge from the nurse going off shift. c. Keep the client's door open for frequent observation. d. Dispose of dirty linen in a red "biohazard" bag. e. Wear a lead apron while providing client care.

ANS: A, E Pregnant nurses should never care for clients with sealed implants of radioactive material, so if the nurse suspects she is pregnant, she should inform her supervisor and request a different assignment. Nurses should wear lead aprons while providing care, ensuring that the apron always faces the client. Each nurse should have his or her own dosimeter film badge. The client's door should be kept closed whenever possible and dirty linens kept in the client's room until the radiation source is removed.

32. The nurse is planning care for a client with hypercalcemia secondary to bone metastasis. Which interventions are included in the plan? (Select all that apply.) a. Increase oral fluids. b. Place an oral airway at the bedside. c. Monitor for Chvostek's sign. d. Implement seizure precautions. e. Assess for hyperactive reflexes. f. Observe for muscle weakness.

ANS: A, F Early manifestations of hypercalcemia include fatigue, loss of appetite, nausea, vomiting, constipation, and polyuria (increased urine output). More serious problems include severe muscle weakness, loss of deep tendon reflexes, paralytic ileus, dehydration, and electrocardiographic changes. An oral airway is not needed. Chvostek's sign is an assessment for hypocalcemia. Seizures and hyperactive reflexes do not occur with hypercalcemia.

Which ethnic groups should the nurse screen specifically for hypocalcemia? (Select all that apply.) a. Whites b. Blacks c. Asians d. Hispanics e. American Indians

ANS: B, C, E Lactose intolerance can lead to hypocalcemia because people avoid milk and dairy products to control their symptoms. Although anyone can have lactose intolerance, the incidence is between 75% and 90% among Asians, blacks, and American Indians.

In interviewing a client with a family history of polycystic kidney disease (PKD), the nurse assesses for which clinical manifestations most carefully? (Select all that apply.) a. Nocturia b. Flank pain c. Diarrhea d. Dysuria e. Bloody urine f. Increased abdominal girth

ANS: B, E, F Flank pain and abdominal girth size are related to distention, and bloody urine is seen with tissue damage secondary to the PKD. The client may also have constipation.

31. In planning a teaching session for a client undergoing photodynamic therapy for lung cancer, the nurse includes which statements? (Select all that apply.) a. "This is a palliative treatment that should decrease your pain." b. "Avoid exposure to the sun for 1 to 3 months after the treatment." c. "Do not eat or drink anything before your treatments." d. "Do not remove skin markings between treatments." e. "You need to wear sunglasses to protect your eyes after treatments." f. "Make sure you keep your curtains closed at home afterward."

ANS: B, E, F Phototherapy causes general sensitivity to light for up to 12 weeks. During this time, the client is at high risk for light sensitivity and eye pain. After the procedure, the client is taught to decrease exposure to sunlight (to the point of being homebound).

17. The student nurse overhears several staff members referring to a client who is receiving chemotherapy as having "chemo brain." The student asks the instructor what that means. Which response by the instructor is best? a. "That is an awful thing to say and the staff should not call a client by that name." b. "It refers to the client's brain as being irreversibly damaged by the chemotherapy." c. "The client has reduced cognitive function that may last for several years." d. "The client has delirium related to the toxic effects of the chemotherapy."

ANS: C "Chemo brain" refers to the changes in concentration, memory, and learning that sometimes accompany chemotherapy. It usually is not present at 3 years after chemotherapy has been completed, so clients should be reassured that this is a temporary condition. Although the staff should be more sensitive, simply criticizing them does not help the student understand the situation.

20. A nurse manager on an oncology nursing unit notes an increased incidence of infection and serious consequences for clients on the unit. Which action by the nursing manager is most beneficial in this situation? a. Review asepsis policies at a mandatory in-service for staff. b. Spot-check all staff for good handwashing practices. c. Develop standard protocols to identify and treat clients with infection. d. Institute protective precautions for all clients receiving chemotherapy.

ANS: C Treatment delays have a serious negative impact on neutropenic clients with infection. Nursing units should have standardized protocols to obtain cultures and diagnostic tests, and to start antibiotics as soon as a client is suspected of having an infection. In-services and spot-checking for good handwashing practice are good ideas as part of a comprehensive infection control practice but are not as important as standard protocols that ensure rapid diagnosis and treatment. Not all clients on chemotherapy will need protective precautions.

A client who has a chronic vitamin B12 deficiency is admitted to the hospital. When obtaining the client's health history, which priority question does the nurse ask this client? a. "Are you having any pain?" b. "Are you having blood in your stools?" c. "Do you notice any changes in your memory?" d. "Do you bruise easily?"

ANS: C Vitamin B12 deficiency impairs cerebral, olfactory, spinal cord, and peripheral nerve function. Severe chronic deficiency may cause permanent neurologic degeneration. The other options are not symptoms of vitamin B12 deficiency.

The nurse is caring for a client with chronic kidney disease who has developed uremia. Which assessment finding does the nurse correlate with this problem? a. Decreased breath sounds b. Foul-smelling urine c. Heart rate of 50 beats/min d. Respiratory rate of 40 breaths/min

ANS: D A client with uremia will also have metabolic acidosis. With severe metabolic acidosis, the client will develop hyperventilation, or Kussmaul respirations, as the body attempts to compensate for the falling pH. The other manifestations would not be associated with acidosis.

a client with diabetes has a blood sugar of 300 and a sodium level of 133 what nursing intervention is indicated to manage sodium with this client

monitor sodium level because it will return to normal with lowering BS

(Select All) What interventions does the nurse recommend for a client who is to be discharged home following total hip replacement surgery?

Elevated toilet seat Walker TED hose

The nurse is caring for a client who has dysphagia caused by systemic sclerosis. What is the best intervention for the nurse to implement for this client?

Encourage frequent, high protein, easy to swallow foods.

nurse action is inappropriate if a client has a K of 8

Give insulin and Iv fluids contain glucose

A client returns to the medical-surgical unit after a total hip replacement with a large wedge-shaped pillow between his legs. The client's daughter asks the nurse why the pillow is in place. What is the nurse's best response?

It will keep the new hip from becoming dislocated

The nurse is caring for an older adult client who has fallen and fractured her hop. The client will have hip replacement surgery followed by extensive rehabilitation. The client confides in the nurse, "I feel like I don't have any control over anything anymore now that I am old." What is the nurse's best response?

It's important to control what you can right now, like making out your menu every day and working with the therapists.

The nurse is teaching a client how to reduce the pain that she often experiences with fibromyalgia. Which statement does the nurse include in the teaching?

Make sure that you get enough sleep every night

The nurse is caring for a client who had a bone marrow aspiration. The client begins to bleed from the aspiration site. Which action does the nurse perform? a. Apply external pressure to the site. b. Elevate the extremities. c. Cover the site with a dressing. d. Immobilize the leg.

NS: A All these options could be done after a bone marrow aspiration and biopsy. However, the most important action when bleeding occurs is to apply external pressure to the site until hemostasis is ensured. The other measures could then be carried out.

A client had a total knee replacement this morning and has a continuous passive motion machine. What activity related to the CPM does the RN delegate to the unlicensed assistive personnel?

Placing controls out of the reach of confused clients

A client with diagnosed osteoarthritis comes to the clinic reporting a low-grade fever, fatigue, and bilateral joint pain. What action by the nurse is most appropriate?

Prepare the client for a laboratory draw for rheumatoid factor

(Select All) The nurse is caring for a hospitalized client who has AIDS and is severly immune compromised. Which interventions are used to help prevent infection in this client?

Provide and incentive spirometer to encourage coughing and deep breathing by the client. Keep a blood pressure cuff, thermometer, and stethoscope in the clint's room for his/her use only. Request that the family take home the freesh flowers that are at the client's bedside. Assist the client with meticulous oral care after meals and at bedtime.

common cause of hypophosphatemia

alcoholism

what hormone triggers nephrons to reabsorb sodium and water from urine back to blood

aldosterone

The nurse teaches a client with DM about differentiating between HYPOglycemia & Ketoacidosis. The client demonstrates an understanding of the teaching by stating that a form of glucose should be taken if which symptoms develop? Select all that apply: a. Polyuria b. Shakiness c. Palpitations d. Blurred vision e. Lightheadedness f. Fruity breath odor

b. Shakiness c. Palpitations e. Lightheadedness Rationale: These are signs of HYPOglycemia and would indicate the need for food or glucose. The other options are signs of HYPERglycemia.

In mixing regular and NPH insulin, the nurse completes the following actions. Place these actions in the correct order. (Separate letters by a comma and space as follows: a, b, c, d.) a. Inspect bottles for expiration dates. b. Gently roll bottle of NPH in hands. c. Wash your hands. d. Inject air into the regular insulin. e. Withdraw the NPH insulin. f. Withdraw the regular insulin. g. Inject air into the NPH bottle. h. Clean rubber stoppers with an alcohol swab.

c. Wash your hands. a. Inspect bottles for expiration dates. b. Gently roll bottle of NPH in hands. h. Clean rubber stoppers with an alcohol swab. g. Inject air into the NPH bottle. d. Inject air into the regular insulin. f. Withdraw the regular insulin. e. Withdraw the NPH insulin. Rationale: After washing hands, it is important to inspect bottles and then to roll NPH to mix the insulin. It is important to inject air into the NPH bottle before placing the needle in a regular insulin bottle to avoid mixing of regular and NPH insulin. The shorter-acting insulin is always drawn up first.

food high in potassium

citrus fruits, cantaloup, bananas, tomato sauce

Which client is at greatest risk for undiagnosed diabetes mellitus? a. Young, muscular white man b. Young African-American man c. Middle-aged Asian woman d. Middle-aged American Indian woman

d. Middle-aged American Indian woman Rationale: Diabetes is a particular problem among African Americans, Hispanics, and American Indians. The incidence of diabetes increases in all races and ethnic groups with age. Being both an American Indian and middle-aged places a person at highest risk.

patient presents with JVD, crackles in the lungs and edema. what is the priority intervention

high fowlers

a nurse instructs a client on how to decrease intake of K in diet, nurse tells client which food contains least amount of K

lettuce

nurse is caring for a client with RF. the lab results reveal a mg level of 3.6 which of the following signs would nurse expect to note in the client based on this mg level

loss of deep tendon reflexes

pt as increased deep tendon reflexes and constipation. appropriate nursing intervention

magnesium sulfate IV

a nurse reviews client electrolyte results and notes a potassium level of 5.4 which of the following would the nurse note on cardiac monitor as a result of lab value

narrow peaked T wave

this complication of IV therapy causes numbeness and tingling

nerve damage

pt is on Maalox and has decrease tendon reflexes what is priority

obtain Mg level

client frequently thirsty. nurse evaluates this symptom as which one

possibly too much sodium and too little water in body

nurse caring for patient with leukemia and notes that the client has poor skin tumor and flat neck and veins. the nurse expects to note in this client if hyponatremia is present

postural blood pressure changes

Which of the following is a priority goal for the client with chronic obstructive pulmonary disease (COPD)? 1. Maintaining functional ability. 2. Minimizing chest pain. 3. Increasing carbon dioxide levels in the blood. 4. Treating infectious agents.

1. A priority goal for the client with COPD is to manage the signs and symptoms of the disease process so as to maintain the client's functional ability. Chest pain is not a typical symptom of COPD. The carbon dioxide concentration in the blood is increased to an abnormal level in clients with COPD; it would not be a goal to increase the level further. Preventing infection would be a goal of care for the client with COPD.

Which of the following physical assessment findings are normal for a client with advanced chronic obstructive pulmonary disease (COPD)? 1. Increased anteroposterior chest diameter. 2. Underdeveloped neck muscles. 3. Collapsed neck veins. 4. Increased chest excursions with respiration.

1. Increased anteroposterior chest diameter is characteristic of advanced COPD. Air is trapped in the overextended alveoli, and the ribs are fixed in an inspiratory position. The result is the typical barrel-chested appearance. Overly developed, not underdeveloped, neck muscles are associated with COPD because of their increased use in the work of breathing. Distended, not collapsed, neck veins are associated with COPD as a symptom of the heart failure that the client may experience secondary to the increased workload on the heart to pump blood into the pulmonary vasculature. Diminished, not increased, chest excursion is associated with COPD.

Which of the following health promotion activities should the nurse include in the discharge teaching plan for a client with asthma? 1. Incorporate physical exercise as tolerated into the daily routine. 2. Monitor peak flow numbers after meals and at bedtime. 3. Eliminate stressors in the work and home environment. 4. Use sedatives to ensure uninterrupted sleep at night.

1. Physical exercise is beneficial and should be incorporated as tolerated into the client's schedule. Peak flow numbers should be monitored daily, usually in the morning (before taking medication). Peak flow does not need to be monitored after each meal. Stressors in the client's life should be modified but cannot be totally eliminated. Although adequate sleep is important, it is not recommended that sedatives be routinely taken to induce sleep.

A 79-year-old female client is admitted to the hospital with a diagnosis of bacterial pneumonia. While obtaining the client's health history, the nurse learns that the client has osteoarthritis, follows a vegetarian diet, and is very concerned with cleanliness. Which of the following would most likely be a predisposing factor for the diagnosis of pneumonia? 1. Age. 2. Osteoarthritis. 3. Vegetarian diet. 4. Daily bathing.

1. The client's age is a predisposing factor for pneumonia; pneumonia is more common in elderly or debilitated clients. Other predisposing factors include smoking, upper respiratory tract infections, malnutrition, immunosuppression, and the presence of a chronic illness. Osteoarthritis, a nutritionally sound vegetarian diet, and frequent bathing are not predisposing factors for pneumonia.

The nurse is planning to teach a client with chronic obstructive pulmonary disease how to cough effectively. Which of the following instructions should be included? 1. Take a deep abdominal breath, bend forward, and cough three or four times on exhalation. 2. Lie flat on the back, splint the thorax, take two deep breaths, and cough. 3. Take several rapid, shallow breaths and then cough forcefully. 4. Assume a side-lying position, extend the arm over the head, and alternate deep breathing with coughing.

1. The goal of effective coughing is to conserve energy, facilitate removal of secretions, and minimize airway collapse. The client should assume a sitting position with feet on the floor if possible. The client should bend forward slightly and, using pursed-lip breathing, exhale. After resuming an upright position, the client should use abdominal breathing to slowly and deeply inhale. After repeating this process three or four times, the client should take a deep abdominal breath, bend forward, and cough three or four times upon exhalation (" huff" cough). Lying flat does not enhance lung expansion; sitting upright promotes full expansion of the thorax. Shallow breathing does not facilitate removal of secretions, and forceful coughing promotes collapse of airways. A side-lying position does not allow for adequate chest expansion to promote deep breathing.

Which of the following is an expected outcome for an elderly client following treatment for bacterial pneumonia? 1. A respiratory rate of 25 to 30 breaths/ minute. 2. The ability to perform activities of daily living without dyspnea. 3. A maximum loss of 5 to 10 lb of body weight. 4. Chest pain that is minimized by splinting the rib cage.

2. An expected outcome for a client recovering from pneumonia would be the ability to perform activities of daily living without experiencing dyspnea. A respiratory rate of 25 to 30 breaths/ minute indicates the client is experiencing tachypnea, which would not be expected on recovery. A weight loss of 5 to 10 lb is undesirable; the expected outcome would be to maintain normal weight. A client who is recovering from pneumonia should experience decreased or no chest pain.

When performing postural drainage, which of the following factors promotes the movement of secretions from the lower to the upper respiratory tract? 1. Friction between the cilia. 2. Force of gravity. 3. Sweeping motion of cilia. 4. Involuntary muscle contractions.

2. The principle behind using postural drainage is that gravity will help move secretions from smaller to larger airways. Postural drainage is best used after percussion has loosened secretions. Coughing or suctioning is then used to remove secretions. Movement of cilia is not sufficient to move secretions. Muscle contractions do not move secretions within the lungs.

A client with bacterial pneumonia is to be started on I.V. antibiotics. Which of the following diagnostic tests must be completed before antibiotic therapy begins? 1. Urinalysis. 2. Sputum culture. 3. Chest radiograph. 4. Red blood cell count.

2. A sputum specimen is obtained for culture to determine the causative organism. After the organism is identified, an appropriate antibiotic can be prescribed. Beginning antibiotic therapy before obtaining the sputum specimen may alter the results of the test. Neither a urinalysis, a chest radiograph, nor a red blood cell count needs to be obtained before initiation of antibiotic therapy for pneumonia.

A client with chronic obstructive pulmonary disease (COPD) is experiencing dyspnea and has a low PaO2 level. The nurse plans to administer oxygen as ordered. Which of the following statements is true concerning oxygen administration to a client with COPD? 1. High oxygen concentrations will cause coughing and dyspnea. 2. High oxygen concentrations may inhibit the hypoxic stimulus to breathe. 3. Increased oxygen use will cause the client to become dependent on the oxygen. 4. Administration of oxygen is contraindicated in clients who are using bronchodilators.

2. Clients who have a long history of COPD may retain carbon dioxide (CO2). Gradually the body adjusts to the higher CO2 concentration, and the high levels of CO2 no longer stimulate the respiratory center. The major respiratory stimulant then becomes hypoxemia. Administration of high concentrations of oxygen eliminates this respiratory stimulus and leads to hypoventilation. Oxygen can be drying if it is not humidified, but it does not cause coughing and dyspnea. Increased oxygen use will not create an oxygen dependency; clients should receive oxygen as needed. Oxygen is not contraindicated with the use of bronchodilators.

The nurse reviews an arterial blood gas report for a client with chronic obstructive pulmonary disease (COPD). pH 7.35; PC02 62; PO2 70; HCO3 34 The nurse should: 1. Apply a 100% non-rebreather mask. 2. Assess the vital signs. 3. Reposition the client. 4. Prepare for intubation.

2. Clients with chronic COPD have CO2 retention and the respiratory drive is stimulated when the PO2 decreases. The heart rate, respiratory rate, and blood pressure should be evaluated to determine if the client is hemodynamically stable. Symptoms, such as dyspnea, should also be assessed. Oxygen supplementation, if indicated, should be titrated upward in small increments. There is no indication that the client is experiencing respiratory distress requiring intubation.

When teaching a client with chronic obstructive pulmonary disease to conserve energy, the nurse should teach the client to lift objects: 1. While inhaling through an open mouth. 2. While exhaling through pursed lips. 3. After exhaling but before inhaling. 4. While taking a deep breath and holding it.

2. Exhaling requires less energy than inhaling. Therefore, lifting while exhaling saves energy and reduces perceived dyspnea. Pursing the lips prolongs exhalation and provides the client with more control over breathing. Lifting after exhaling but before inhaling is similar to lifting with the breath held. This should not be recommended because it is similar to the Valsalva maneuver, which can stimulate cardiac arrhythmias.

A client's arterial blood gas values are as follows: pH, 7.31; PaO2, 80 mm Hg; PaCO2, 65 mm Hg; HCO3 −, 36 mEq/ L. The nurse should assess the client for? 1. Cyanosis. 2. Flushed skin. 3. Irritability. 4. Anxiety.

2. The high PaCO2 level causes flushing due to vasodilation. The client also becomes drowsy and lethargic because carbon dioxide has a depressant effect on the central nervous system. Cyanosis is a sign of hypoxia. Irritability and anxiety are not common with a PaCO2 level of 65 mm Hg but are associated with hypoxia.

The nurse should teach the client with asthma that which of the following is one of the most common precipitating factors of an acute asthma attack? 1. Occupational exposure to toxins. 2. Viral respiratory infections. 3. Exposure to cigarette smoke. 4. Exercising in cold temperatures.

2. The most common precipitator of asthma attacks is viral respiratory infection. Clients with asthma should avoid people who have the flu or a cold and should get yearly flu vaccinations. Environmental exposure to toxins or heavy particulate matter can trigger asthma attacks; however, far fewer asthmatics are exposed to such toxins than are exposed to viruses. Cigarette smoke can also trigger asthma attacks, but to a lesser extent than viral respiratory infections. Some asthmatic attacks are triggered by exercising in cold weather.

The cyanosis that accompanies bacterial pneumonia is primarily caused by which of the following? 1. Decreased cardiac output. 2. Pleural effusion. 3. Inadequate peripheral circulation. 4. Decreased oxygenation of the blood.

4. A client with pneumonia has less lung surface available for the diffusion of gases because of the inflammatory pulmonary response that creates lung exudate and results in reduced oxygenation of the blood. The client becomes cyanotic because blood is not adequately oxygenated in the lungs before it enters the peripheral circulation. Decreased cardiac output may be a comorbid condition in some clients with pneumonia; however, it is not the cause of cyanosis. Pleural effusions are a potential complication of pneumonia but are not the primary cause of decreased oxygenation. Inadequate peripheral circulation is also not the cause of the cyanosis that develops with bacterial pneumonia.

When instructing clients on how to decrease the risk of chronic obstructive pulmonary disease (COPD), the nurse should emphasize which of the following? 1. Participate regularly in aerobic exercises. 2. Maintain a high-protein diet. 3. Avoid exposure to people with known respiratory infections. 4. Abstain from cigarette smoking.

4. Cigarette smoking is the primary cause of COPD. Other risk factors include exposure to environmental pollutants and chronic asthma. Participating in an aerobic exercise program, although beneficial, will not decrease the risk of COPD. Insufficient protein intake and exposure to people with respiratory infections do not increase the risk of COPD.

The nurse assesses the respiratory status of a client who is experiencing an exacerbation of chronic obstructive pulmonary disease (COPD) secondary to an upper respiratory tract infection. Which of the following findings would be expected? 1. Normal breath sounds. 2. Prolonged inspiration. 3. Normal chest movement. 4. Coarse crackles and rhonchi.

4. Exacerbations of COPD are commonly caused by respiratory infections. Coarse crackles and rhonchi would be auscultated as air moves through airways obstructed with secretions. In COPD, breath sounds are diminished because of an enlarged anteroposterior diameter of the chest. Expiration, not inspiration, becomes prolonged. Chest movement is decreased as lungs become overdistended.

A 34-year-old female with a history of asthma is admitted to the emergency department. The nurse notes that the client is dyspneic, with a respiratory rate of 35 breaths/ minute, nasal flaring, and use of accessory muscles. Auscultation of the lung fields reveals greatly diminished breath sounds. Based on these findings, which action should the nurse take to initiate care of the client? 1. Initiate oxygen therapy and reassess the client in 10 minutes. 2. Draw blood for an arterial blood gas analysis and send the client for a chest X-ray. 3. Encourage the client to relax and breathe slowly through the mouth. 4. Administer bronchodilators.

4. In an acute asthma attack, diminished or absent breath sounds can be an ominous sign indicating lack of air movement in the lungs and impending respiratory failure. The client requires immediate intervention with inhaled bronchodilators, I.V. corticosteroids and, possibly, I.V. theophylline (Theo-Dur). Administering oxygen and reassessing the client 10 minutes later would delay needed medical intervention, as would drawing blood for an arterial blood gas analysis and obtaining a chest X-ray. It would be futile to encourage the client to relax and breathe slowly without providing the necessary pharmacologic intervention.

Which question does the nurse ask the client who has isotonic dehydration to determine a possible cause? a. "Do you take diuretics, or 'water pills'?" b. "What do you normally eat over a day's time?" c. "How many bowel movements do you have daily?" d. "Have you been diagnosed with diabetes mellitus?"

A -"Do you take diuretics or 'water pills'?" -Misuse or overuse of diuretics is a common cause of isotonic dehydration. The other statements are not indicative of causes of isotonic dehydration.

Which of the following indicates that the client with chronic obstructive pulmonary disease (COPD) who has been discharged to home understands his care plan? 1. The client promises to do pursed-lip breathing at home. 2. The client states actions to reduce pain. 3. The client says that he will use oxygen via a nasal cannula at 5 L/ minute. 4. The client agrees to call the physician if dyspnea on exertion increases.

4. Increasing dyspnea on exertion indicates that the client may be experiencing complications of COPD. Therefore, the nurse should notify the physician. Extracting promises from clients is not an outcome criterion. Pain is not a common symptom of COPD. Clients with COPD use low-flow oxygen supplementation (1 to 2 L/ minute) to avoid suppressing the respiratory drive, which, for these clients, is stimulated by hypoxia. .

Which of the following diets would be most appropriate for a client with chronic obstructive pulmonary disease (COPD)? 1. Low-fat, low-cholesterol diet. 2. Bland, soft diet. 3. Low-sodium diet. 4. High-calorie, high-protein diet.

4. The client should eat high-calorie, high-protein meals to maintain nutritional status and prevent weight loss that results from the increased work of breathing. The client should be encouraged to eat small, frequent meals. A low-fat, low-cholesterol diet is indicated for clients with coronary artery disease. The client with COPD does not necessarily need to follow a sodium-restricted diet, unless otherwise medically indicated. There is no need for the client to eat bland, soft foods.

Which of the following measures would most likely be successful in reducing pleuritic chest pain in a client with pneumonia? 1. Encourage the client to breathe shallowly. 2. Have the client practice abdominal breathing. 3. Offer the client incentive spirometry. 4. Teach the client to splint the rib cage when coughing.

4. The pleuritic pain is triggered by chest movement and is particularly severe during coughing. Splinting the chest wall will help reduce the discomfort of coughing. Deep breathing is essential to prevent further atelectasis. Abdominal breathing is not as effective in decreasing pleuritic chest pain as is splinting of the rib cage. Incentive spirometry facilitates effective deep breathing but does not decrease pleuritic chest pain.

What information is most important to teach the client going home with a PICC line? A. "Flush the IV daily." B. "Avoid heavy lifting with the arm that has the IV." C. "Be sure to place the arm with the IV in a sling during the day." D. "You can use the arm with the IV for most of the activities of daily living."

A -"Avoid carrying your grandchild with the arm that has the IV." -A properly placed PICC (in the antecubital fossa or basilic vein) allows the client considerable freedom of movement. Clients can participate in most activities of daily living. Heavy lifting can dislodge the catheter or occlude the lumen. Although it is important to keep the insertion site and tubing dry, the client can shower. The device only needs to be flushed after medications are administered through it or flushed weekly.

The client scheduled to have intravenous urography is a diabetic and taking the antidiabetic agent metformin. What should the nurse tell this client? A. "Call your diabetes doctor and tell him or her that you are having an intravenous urogram performed using dye." B. "Do not take your metformin the morning of the test because you are not going to be eating anything and could become hypoglycemic." C. "You must start on an antibiotic before this test because your risk of infection is greater as a result of your diabetes." D. "You must take your metformin immediately before the test is performed because the IV f fluid and the dye contain a significant amount of sugar."

A -"Call your diabetes doctor and tell him or her that you are having an intravenous urogram performed using dye." -Metformin can cause lactic acidosis and renal impairment because of an interaction with the dye. This drug must be discontinued for 48 hours before the procedure and not started again after the procedure until urine output is well established.

The client is being admitted with a suspected diagnosis of bladder cancer. Which question will assist in determining risk factors? A. "Do you smoke cigarettes?" B. "Do you use alcohol?" C. "Do you use recreational drugs?" D. "Do you take any prescription drugs?"

A -"Do you smoke cigarettes?" -Smoking is known to be a factor that greatly increase the risk of bladder cancer. Neither alcohol use, prescription drug use (except medications that contain phenacetin), nor recreational drug use are known to increase the risk of developing bladder cancer.

Which intervention is most important for the nurse to teach the client who has lymphedema in her right arm from a mastectomy 1 year ago? A. "Exercise your arm and use it during tasks that occur at the level of your chest or higher." B. "Be sure to use sunscreen or protective clothing to reduce the risk of injuring this arm." C. "Reduce your salt intake to prevent excess water retention." D. "Do not expose the right arm to temperature extremes."

A -"Exercise your arm and use it during tasks that occur at the level of your chest or higher." -Skeletal muscle contractions facilitate flow in lymph channels. Keeping the arm at chest level or higher prevents stasis of lymph fluid from gravitational forces. The other answer options are not choices that would prevent stasis of lymph fluid.

Which statement made by the client with stress incontinence indicates a need for clarification of nutrition therapy? A. "I will limit my total intake of fluids." B. "I will avoid drinking alcoholic beverages." C. "I will avoid drinking coffee and other caffeinated beverages." D. "I will try to reduce my total body weight by at least 10%."

A -"I will limit my total intake of fluids." -Limiting fluids concentrates urine and can irritate tissues, leading to increased incontinence. Alcoholic and caffeinated beverages are bladder stimulants. Obesity increases intra-abdominal pressure, causing incontinence.

The client is beginning to undergo urinary habit training. Which is an effective instruction to give this client's caregiver? A. "Keep a continence record for at least 3 days." B. "Do not resort to running water in an attempt to prompt client." C. "Establish a toileting interval of not less than 4 hours." D. "Avoid leaving the client on the toilet for more than 15 minutes."

A -"Keep a continence record for at least 3 days." -The caregiver should keep a continence record to determine patterns in the client's voiding and incontinence episodes. The caregiver should use the power of suggestion, establish a toileting interval of not less than 2 hours, and avoid leaving the client on the toilet for more than 5 minutes.

The client is a young woman who is being treated with amoxicillin (Amoxil) for a urinary tract infection. Which is the highest priority instruction for the nurse to give the client? A. "Use a second form of birth control while on the drug." B. "You will experience increased menstrual bleeding while on this drug." C. "You may experience an irregular heartbeat while on the drug." D. "Watch for blood in your urine while taking this drug."

A -"Use a second form of birth control while on the drug." -The client should use a second form of birth control because penicillin seems to reduce the effectiveness of estrogen-containing contraceptives. She should not experience increased menstrual bleeding, an irregular heartbeat, or blood in her urine while taking the medication.

Which drug will the nurse administer to the client diagnosed with renal calculi from hyperuricemia? A. Allopurinol (Zyloprim) B. Captopril (Capoten) C. Chlorothiazide (Diuril) D. Phenazopyridine (Pyridium)

A -Allopurinol (Zyloprim) -Allopurinol inhibits the enzyme that converts purine metabolites into uric acid, thereby reducing the amount of uric acid present for precipitation into stones. The other drugs listed would not be effective.

What action should the nurse take to prevent infection in the older adult receiving intravenous therapy?

A -Allowing antiseptic solutions to dry prior to dressing application -The skin of an older adult may be more delicate and compromised. Avoidance of a disruption in skin integrity lessens the chance of an infection occurring with an IV catheter. Allowing antiseptic solutions to dry prior to applying the catheter dressing allows the antiseptic solution to be effective and the dressing to remain securely over the site. Using alcohol pads makes it easier to remove tape and avoid skin tears. The skin should never be shaved before venipuncture because micro-abrasions may occur, which can lead to infection. Excessive friction may damage fragile skin and compromise skin integrity.

Which client will not be able to adhere to bladder training for incontinence? A. An older man who is confused B. An older woman with diabetes C. A middle-aged man with early-stage renal failure D. A middle-aged woman with early menopause

A -An older man who is confused -For a bladder training program to succeed in urge incontinence, the client must be alert, aware, and able to resist the urge to urinate.

The nurse notes that the hand grip of the client with hypokalemia has diminished since the previous assessment 1 hour ago. What is the nurse's priority intervention? a. Assess the client's respiratory rate, rhythm, and depth. b. Measure the client's pulse and blood pressure. c. Document findings and monitor the client. d. Call the health care provider.

A -Assessing the client's rate, rhythm, and depth of respiration -Progressive skeletal muscle weakness is associated with increasing severity of the hypokalemia. The most life-threatening complication of hypokalemia is respiratory insufficiency. It is imperative for the nurse to perform a respiratory assessment first to make sure that the client is not in immediate jeopardy. The nurse would next call the health care provider for orders for potassium replacement.

The nurse is caring for a client receiving an epidural infusion for pain management. Which action has the highest priority? a. Assessing the respiratory rate b. Changing the dressing over the site c. Using various pain management therapies d. Weaning the pain medication

A -Assessing the respiratory rate -Complications from an epidural infusion can be caused by the type of medication being infused or can be related to the catheter. When used for pain management, the client needs to be assessed for level of alertness, respiratory status, and itching. Dressings are not routinely changed because the catheter is only used for short periods. Using other pain management therapies and weaning the pain medication are important, but monitoring the respiratory status has the highest priority in the nursing care of this client

To prevent infection when infusing an intermittent "piggyback" line, which intervention is most important for the nurse to implement?

A -Backpriming the secondary container -The backpriming method allows multiple drugs to be infused through the same secondary set. This method allows the primary and secondary sets to remain connected together as an infusion system and allows the nurse to adhere to the Infusion Nurses Society (INS) standards of practice. The client is at an increased risk for infection whenever the catheter is disconnected from the tubing (distractors B and C). Sterile gloves are not necessary for the IV administration of medication.

Which infusion device will the nurse select for the older adult client with the medical diagnosis of "Dehydration"?

A -Cassette pump -An older adult client who has dehydration will require a large fluid volume that is accurately measured by using a cassette pump during the infusion. Volumetric controllers count drops for administered volume and are inherently inaccurate because of variation in drop size. A syringe pump is accurate but not appropriate for a large volume. Elastomeric balloons are used to deliver intermittent medications.

When an IV pump alarms because of pressure, what action will the nurse take first? a. Check for kinking of the catheter. b. Flush the catheter with a thrombolytic enzyme. c. Get a new infusion pump. d. Remove the IV catheter.

A -Checks for kinking of the catheter -Fluid flow through the infusion system requires that the pressure on the external side be greater than the pressure at the catheter tip. Fluid flow can be slowed for many reasons. A common reason, and easy to correct, is a kinked catheter. If this is not the cause of the pressure alarm, the nurse may have to ascertain if a clot has formed inside the catheter lumen or if the pump is no longer functional

The client is admitted with multiple fractures from a motor vehicle crash (MVC). Which of the client's previous or concurrent health problems is most likely to increase his risk for hypophosphatemia?

A -Chronic alcoholic pancreatitis -Chronic alcoholism leads to malnutrition. Malnutrition is a major contributing factor to the development of hypophosphatemia. None of the other conditions contribute to hypophosphatemia.

The nurse is assessing several clients receiving intravenous medications. Which client situation requires immediate intervention?

A -Completion of an intermittent medication into a Groshong catheter -A Groshong catheter is a peripherally inserted catheter that needs to be flushed with saline after intermittent use. Peripheral IV catheters should be discontinued after 4 days. An order to discontinue the peripheral catheter requires intervention, but flushing of the Groshong catheter is more of an immediate intervention to prevent clotting of the catheter. A nonaccessed implanted port site needs to be assessed, but this is not an immediate intervention.

When assessing the client's peripheral IV site, the nurse observes a streak of red along the vein path and palpates a 4-cm venous cord. What is the most accurate documentation of this finding? a. "Grade 3 phlebitis at IV site" b. "Infection at IV site" c. "Thrombosed area at IV site" d. "Infiltration at IV site"

A -Grade 3 phlebitis at IV site -The presence of a red streak and palpable cord indicates grade 3 phlebitis. There is no information in the description to indicate that infection, infiltration, or thrombosis is present.

When changing the administration set on a central venous catheter, it is most important for the nurse to carry out what intervention?

A -Have the client hold his breath during the disconnection and reconnection. -An air embolus is less likely to form if the exit site is lower than the level of the heart and if the pressure in the thoracic cavity is higher when the disconnection occurs. Having the client perform the Valsalva maneuver and maintain it during disconnection and reconnection helps maintain a higher intrathoracic pressure. The slide clamp on the catheter extension should be kept clamped. The client should be placed in the flat position when changing administration sets.

2. What would be the response if a person's nephrons were not able to filter normally due to scarring of the proximal convoluted tubule leading to inhibition of reabsorption? a. Increased urine output, fluid volume deficit b. Decreased urine output, fluid volume deficit c. Increased urine output, fluid volume overload d. Decreased urine output, fluid volume overload

A -Increased urine output, fluid volume deficit -The nephrons filter about 120 mL/min. Most of this filtrate is reabsorbed in the proximal convoluted tubule. If the tubule were not able to reabsorb the fluid that has been filtered, urine output would greatly increase, leading to rapid and severe dehydration.

The nurse is assessing the laboratory findings of a client with a urinary tract infection. Which finding requires immediate intervention? A. Left shift in the white blood cell (WBC) differential B. Serum white blood cell count of 8000/mm3 C. Presence of red blood cells in the urine D. Presence of white blood cells in the urine

A -Left shift in the white blood cell (WBC) differential -A left shift most commonly occurs with urosepsis, a condition that has a 15% mortality rate. Left shifts rarely occur with uncomplicated cystitis. This is the most life-threatening change in values of the laboratory findings listed.

hich assessment does the nurse use to determine the adequacy of circulation in a client whose blood osmolarity is 250 mOsm/L? a. Measuring urine output b. Measuring abdominal girth c. Monitoring fluid intake d. Comparing radial versus apical pulses

A -Measuring urine output -The blood osmolarity is low. The client could be dehydrated (hypo-osmolar dehydration) or overhydrated, with dilution of blood solute. The most sensitive noninvasive indicator of circulation adequacy is urine output. Measuring abdominal girth, comparing pulses, and monitoring fluid intake would not be accurate assessment techniques for this client.

Which client is at greatest risk for development of a bacterial cystitis? A. Older female client not taking estrogen replacement B. Older male client with mild congestive heart failure C. Middle-aged female client who has never been pregnant D. Middle-aged male client who is taking cyclophosphamide for cancer therapy

A -Older female client not taking estrogen replacement -Females at any age are more susceptible to cystitis than men because of the shorter urethra in women. Postmenopausal women who are not on hormone replacement therapy are at an increased risk for bacterial cystitis because of changes in the cells of the urethra and vagina. The middle-aged female client who has never been pregnant would not have a risk potential as high as the older female client who is using hormone replacement therapy.

Which type of incontinence is most common after a difficult vaginal delivery? A. Stress incontinence B. Urge incontinence C. Reflex incontinence D. Overflow incontinence

A -Stress incontinence -Childbirth is most likely to result in stress incontinence. There is no evidence that childbirth is likely to result in the development of urge, reflex, or overflow incontinence.

The nurse is administering a prescribed fibrinolytic to a client who is having a myocardial infarction (MI). Which adverse effect does the nurse monitor for? a. Bleeding b. Orthostatic hypotension c. Deep vein thrombosis d. Nausea and vomiting

ANS: A A fibrinolytic lyses any clots in the body, thus causing an increased risk for bleeding. Fibrinolytic therapy does not place the client at risk for hypotension, thrombosis, or nausea and vomiting. pg 861

The nurse completes which assessment in a client with acute glomerulonephritis and periorbital edema? a. Auscultating breath sounds b. Checking blood glucose levels c. Measuring deep tendon reflexes d. Testing urine for protein

ANS: A Acute glomerular nephritis can cause sodium and water retention. When clients have edema, they may also have circulatory overload with pulmonary edema. The other assessments would not be related to this client's condition

A client is admitted to the hospital with a serum creatinine level of 2 mg/dL. When taking the client's history, which question does the nurse ask first? a. "Do you take any nonprescription medications?" b. "Does anyone in your family have kidney disease?" c. "Do you have yearly blood work done?" d. "Is your diet low in protein?"

ANS: A Acute renal failure can be caused by certain medications considered to have a nephrotoxic effect, such as NSAIDs and acetaminophen. Asking the client whether he or she takes any nonprescription drugs can help determine which medication(s) might have contributed to the problem. A family history is important but is not as vital as assessing for nephrotoxic agents that the client may have ingested. Yearly blood work might reveal a trend in kidney function, but again would not be as important. A diet low in protein would not be an important factor to assess.

6. A client has bone cancer. What intervention does the nurse implement as a priority for this client? a. Using a lift sheet when repositioning the client b. Positioning the client's heels to keep them from touching the mattress c. Providing small, frequent meals rich in calcium and phosphorus d. Applying pressure for 5 minutes after intramuscular injections

ANS: A Bone metastasis of cancer can cause such bone destruction that grasping or pulling a client can result in a pathologic fracture. Using a lift sheet spreads the client's weight more evenly, preventing excessive force on any one body area. Preventing pressure on the heels will help prevent pressure ulcers; this is a good intervention for all clients but does not take priority over preventing fractures. Adding calcium and phosphorus to meals will not prevent fractures. Applying pressure after IM injections is not related to this client's condition.

Which client is most at risk for developing postrenal kidney failure? a. Client diagnosed with renal calculi b. Client with congestive heart failure c. Client taking NSAIDs for arthritis pain d. Client recovering from glomerulonephritis

ANS: A Causes of postrenal kidney failure include disorders that obstruct the flow of urine, such as renal calculi. Heart failure can lead to prerenal failure, which is due to decreased blood flow to the kidneys. Both NSAIDs and glomerulonephritis can damage the kidney, leading to intrarenal failure.

When evaluating the effects of a low-protein diet in a client with chronic kidney disease, the nurse is most concerned with which result? a. Albumin level of 2 g/dL b. Calcium level of 8.0 mg/dL c. Potassium level of 5.2 mmol/L d. Magnesium level of 3 mEq/L

ANS: A Clients with chronic kidney disease are placed on a low-protein diet. However, decreased serum albumin levels indicate that the protein they are taking in is not enough for their metabolic needs. The electrolyte levels in the other options are not related to protein

The nurse is caring for a client who has a decreased serum iron level. Which intervention does the nurse prioritize for this client? a. Dietary consult b. Family assessment c. Cardiac assessment d. Administration of vitamin K

ANS: A Diets can alter cell quality and affect blood clotting. Diets low in iron can cause anemia and decrease the function of all red blood cells. The question does not say that the hemoglobin is low enough to affect the cardiac function. Family assessment may be important in finding out any genetic or family lifestyle causes of the low serum iron level. However, the first intervention that the nurse can provide is to have the client's dietary habits evaluated and changed so that iron levels can increase. Vitamin K is involved with clotting, not with iron stores.

Which clinical manifestation indicates to the nurse that a client with glomerulonephritis (GN) is responding as expected to the prescribed treatment? a. The client has lost 11 pounds in the past 10 days. b. The client's urine specific gravity is 1.048. c. No blood is observed in the client's urine. d. The client's blood pressure is 152/88 mm Hg.

ANS: A Fluid retention is a major feature of glomerular nephritis. This weight loss represents fluid loss, indicating that the glomeruli are performing the function of filtration. The urine specific gravity is high. Blood is not usually seen in GN, so this finding would be expected. The client's blood pressure is too high; this may indicate kidney damage or fluid overload.

The nurse is assessing a 75-year-old male client. Which blood value indicates that the client is experiencing normal changes associated with aging? a. Hemoglobin, 13.0 g/dL b. Platelet count, 100,000/mm3 c. Prothrombin time (PT), 14 seconds d. White blood cell (WBC) count, 5000/mm3

ANS: A Hemoglobin levels in men and women fall after middle age. Therefore, this client's hemoglobin value would be considered part of the aging process. Platelet counts and blood-clotting times are not age related; the client's platelet count and PT are elevated for some other reason. The WBC count shown is normal. pg 860

A client is taking furosemide (Lasix) and becomes confused. Which potassium level does the nurse correlate with this condition? a. 2.9 mEq/L b. 3.8 mEq/L c. 5.0 mEq/L d. 6.0 mEq/L

ANS: A Hypokalemia decreases cerebral function and is manifested by lethargy, confusion, inability to perform problem-solving tasks, disorientation, and coma. Normal potassium levels are 3.5 to 5.0 mEq/L. At 2.9 mEq/L, potassium is too low, and this could lead to neurologic manifestations.

A client has been treated for hypokalemia. Which clinical manifestation or condition indicates that treatment has been effective? a. Having a bowel movement daily b. Gaining 2 lb during the past week c. Electrocardiogram (ECG) showing inverted T-waves d. Fasting blood glucose level of 106 mg/dL

ANS: A Hypokalemia depresses all excitable tissues, including gastrointestinal smooth muscle. Clients who have hypokalemia have reduced or absent bowel sounds and are constipated. The other answer options are not applicable to hypokalemia

28. The nurse questions which activity for the client with thrombocytopenia? a. Application of warm compresses to bruises b. Cleaning teeth with a soft-bristled brush c. Taking acetaminophen (Tylenol) for pain d. Using stool softeners daily for constipation

ANS: A Ice should be applied to areas of bruising or trauma to decrease bleeding. Warm compresses would lead to vasodilation and potentially to more bleeding. It is important to implement measures to decrease the risk of bleeding. A soft-bristled toothbrush decreases trauma to gums, which could cause bleeding. Straining at the stool could increase risk for rectal bleeding, so stool softeners may be prescribed. Acetaminophen does not affect platelet function and bleeding as do aspirin products.

22. A client with advanced cancer is being treated with intravenous mithramycin (Mithracin). Which clinical manifestation indicates that the treatment is effective? a. Bowel sounds are active in all four quadrants. b. The client's serum sodium level is 138 mEq/L. c. The pulse rate is 68 beats/min and bounding. d. Urine output has increased to 30 mL/hr.

ANS: A Mithramycin is used to treat hypercalcemia, which is seen most often in oncology clients who have bone metastases. Hypercalcemia reduces excitable membrane activity, causing decreased intestinal motility. Return of intestinal motility is an indication that serum calcium levels are decreasing. Mithramycin has no direct effect on serum sodium levels or urine output. The pulse rate most likely would be rapid and irregular with hypercalcemia and would normalize as calcium levels return to normal.

The nurse is assessing a client with numerous areas of bruising. Which question does the nurse ask to determine the cause of this finding? a. "Do you take aspirin?" b. "How often do you exercise?" c. "Are you a vegetarian?" d. "How often do you take Tylenol?"

ANS: A Platelet aggregation is essential for blood clotting. An inability to clot blood when an injury occurs can result in bleeding, which would cause bruising. Aspirin is a drug that interferes with platelet aggregation and has the ability to "plug" an extrinsic event, such as trauma. Vitamin K found in green vegetables enhances clotting factors, which would improve the ability to stop bleeding associated with an extrinsic event. Acetaminophen (Tylenol) and exercise do not inhibit clotting factors.

During a hot summer day, an older adult client tells the clinic nurse, "I am not drinking or voiding that much these days." The nurse notes a heart rate of 100 beats/min and a blood pressure of 90/60 mm Hg. Which action does the nurse take first? a. Give the client something to drink. b. Insert an intravenous catheter. c. Teach the client to drink 2 to 3 liters a day. d. Perform a bladder scan to assess urine volume.

ANS: A Severe blood volume depletion can lead to kidney failure, even in those who have no kidney problem. The client is showing signs of mild volume depletion. The first action the nurse should take is to give the client something to drink. After that, the nurse should teach the client to avoid dehydration by drinking at least 2 to 3 L of fluid daily. The client does not need an IV at this time. Performing a bladder scan will not help prevent or reverse the client's problem.

A client has been missing some scheduled hemodialysis sessions. Which intervention is most important for the nurse to implement? a. Discussing with the client his or her acceptance of the disease b. Discussing with the client the option of peritoneal dialysis c. Rescheduling the sessions to another day or another time d. Stressing to the client the importance of going to the sessions

ANS: A Some people on dialysis retreat into complete or partial denial of the disease and the need for treatment. They may deny the need for dialysis and/or may not adhere to drug therapy and diet restrictions. Providing support as the client struggles to accept the disease is an important step in ensuring compliance with the dialysis regimen. The nurse should explore scheduling options, but missing so many sessions cues the nurse that a bigger problem than just scheduling is involved. The nurse should provide education, but simply stressing the need for dialysis will not help the client accept it. Peritoneal dialysis, with its technical demands on the client and partner, probably is not an option for a client who appears noncompliant with hemodialysis.

A client has end-stage kidney disease (ESKD). The nurse observes tall, peaked T waves on the client's cardiac monitor. Which action by the nurse is best? a. Check the serum potassium level. b. Document the finding in the client's chart. c. Prepare to give sodium bicarbonate. d. Call the health care provider to request an electrocardiogram (ECG).

ANS: A Tall, peaked T waves are a manifestation of hyperkalemia. Thus, the nurse should check the potassium level. Afterward, the nurse should report findings to the provider. The client may need an ECG, but treatment may be based on monitor tracings and potassium levels. Sodium bicarbonate is not warranted. Documentation is important but is not the priority.

16. A client is hospitalized for chemotherapy. The registered nurse intervenes when observing which action by the nursing assistant? a. Allowing the client to rest instead of making him or her perform oral hygiene b. Helping the client wash the groin and axillary areas every 12 hours c. Cutting food and opening food packages when the client's meal tray arrives d. Reminding the client to use the incentive spirometer every hour while awake

ANS: A The biggest dangers to clients on chemotherapy are neutropenia and the risk of serious infection or sepsis. Most infections arise from overgrowth of the client's own normal flora, so personal hygiene is critical. The client must perform hygiene measures on a schedule, even if he or she is very tired. Instead of allowing the client to rest, the nursing assistant should help the client perform oral hygiene and other measures. The other actions would be acceptable.

4. In evaluating dietary teaching for a client with chemotherapy-induced neutropenia, the nurse becomes concerned when the client makes which food choice? a. Fruit salad b. Applesauce c. Steamed broccoli d. Baked potato

ANS: A The client who is neutropenic should be taught to eat a low-bacteria diet. This includes avoiding raw fruits or vegetables and undercooked meat, eggs, or fish.

A client with acute kidney injury is placed on a fluid restriction. To determine whether outcomes related to fluid balance are being met, the nurse assesses for which finding? a. Absence of lung crackles b. Decreased serum creatinine level c. Decreased serum potassium level d. Increased muscle strength

ANS: A The client with chronic kidney disease is expected to achieve and maintain an acceptable fluid balance. Fluid restriction helps with this outcome. Absence of lung crackles can indicate that the client is not fluid overloaded. The other options are not related to fluid balance.

A client's temperature after dialysis is 99° F (37.2° C) and was normal before dialysis. Which is the nurse's best action? a. Continue to monitor the temperature. b. Encourage the client to drink fluids. c. Obtain a white blood cell count. d. Prepare to culture the fistula site.

ANS: A The client's temperature may be elevated because the dialysis machine warms the blood slightly. An excessive temperature elevation from baseline can signal sepsis. The nurse should inform the provider and obtain blood cultures if this happens. The other actions are not needed.

5. What teaching is essential for a client who has received an injection of iodine-131? a. "Do not share a toilet with anyone else or let anyone clean your toilet." b. "You need to save all your urine for the next week." c. "No special precautions are needed because this type of radiation is weak." d. "Avoid all contact with other people until the radiation device is removed."

ANS: A The radiation source is an unsealed isotope that is eliminated from the body in waste products, especially urine and feces. This material is radioactive for about 48 hours after instillation of the isotope. Having the client not share a toilet with other people or allowing anyone to clean the client's toilet for a specific period of time ensures that the isotope has been completely eliminated, and that the client's wastes are no longer radioactive.

The nurse is planning discharge teaching for a client who has a splenectomy. Which statement does the nurse include in this client's teaching plan? a. "Avoid crowds and people who are sick." b. "Do not eat raw fruits or vegetables." c. "Avoid environmental allergens." d. "Do not play contact sports."

ANS: A The spleen is the major site of B-lymphocyte maturation and antibody production. Those who undergo splenectomies for any reason have a decreased antibody-mediated immune response and are particularly susceptible to viral infections. Eating raw fruits and vegetables places the client at risk for bacterial infections. The body responds to environmental allergens with an unspecific inflammatory process. The client is not at risk for bleeding or injury due to contact sports.

25. The nurse teaches a client with superior vena cava syndrome that improvement is characterized by which clinical manifestation? a. The client's hands are less swollen. b. Breath sounds are clear bilaterally. c. The client's back pain is relieved. d. Pedal edema is present.

ANS: A With superior vena cava syndrome, blood flow through the vena cava is compromised as a result of tumor growth. Blood backs up into the periphery, and the client experiences upper body swelling, including the hands and feet. Compression of the superior vena cava has no effect on breath sounds. This would occur when blood is impeded from leaving the lungs, and with disorders that affect the left side of the heart. Back pain is not associated with this disorder.

The nurse is teaching a client who is receiving sodium warfarin (Coumadin). Which topics does the nurse include in the teaching plan? (Select all that apply.) a. Foods high in vitamin K b. Using acetaminophen (Tylenol) for minor pain c. Daily exercise and weight management d. Use of a safety razor and soft toothbrush e. Blood testing regimen

ANS: A, B, D, E The client on warfarin will need to know which foods are high in vitamin K because vitamin K intake must be consistent to avoid interfering with the anticoagulant properties of warfarin. Clients should not take aspirin or NSAIDs for minor pain owing to their anticoagulant properties. Clients must use safety razors and soft toothbrushes to avoid bleeding episodes. The client on warfarin needs regular blood tests for prothrombin time (PT) and international normalized ratio (INR). Daily exercise and weight management are not specifically important to this client.

The nurse is monitoring a client with liver failure. Which assessments does the nurse perform when monitoring for bleeding in this client? (Select all that apply.) a. Gums b. Lung sounds c. Urine d. Stool e. Hair

ANS: A, C, D The liver is the site for production of clotting factors. Without these factors, the client is at risk for bleeding. Common areas of bleeding include the gums and mucous membranes, bladder, and gastrointestinal tract. Lung sounds and hair are part of the assessment but are not essential in the presence of liver failure and hematologic abnormalities.

2. A client is undergoing radiation therapy and asks the nurse about skin care for the exposed area. Which statement by the nurse is most accurate? a. "No products work well to reduce the skin reactions you get from radiation." b. "No one product works best, so you can choose what you would like to use." c. "The only medication that works well for skin reactions is very expensive." d. "No good studies on skin care with radiation have been conducted to date."

ANS: B A recent placebo-controlled study showed that none of three products used to manage radiation-related skin reactions was superior to the others. Researchers concluded that clients should use products that are easy to obtain and use and are within the client's budget. Simply stating that no one product works well does not give the client enough information to make an informed choice. Prescription medications for skin reactions can be expensive, but again this response does not help the client make a decision.

A client is 12 hours post-kidney transplantation. The nurse notes that the client has put out 2000 mL of urine in 10 hours. Which assessment does the nurse carry out first? a. Skin turgor b. Blood pressure c. Serum blood urea nitrogen (BUN) level d. Weight of the client

ANS: B After transplantation, the client may have diuresis. Excessive diuresis might cause hypotension. Hypotension needs to be prevented because it can reduce blood flow and oxygen to the new kidney, threatening graft survival. The other assessments can give information about fluid balance, but hypotension is the main concern here, so the nurse needs to check the client's blood pressure, then notify the provider.

18. A client with prostate cancer is taking estrogen daily to control tumor growth. He reports that his left calf is swollen and painful. Which is the nurse's best action? a. Instruct the client to keep the leg elevated. b. Measure and compare calf circumferences. c. Apply ice to the calf after massaging it. d. Document this expected response.

ANS: B An adverse reaction to hormonal manipulation therapy is the development of thrombus formation. The nurse should measure both calf circumferences and compare them; the side with a thromboembolism will be larger. Elevation may be helpful, but first the nurse needs to assess the situation. Massaging a calf that is swollen and painful is never correct, because this action might break a clot to form an embolus, which could then travel to the lungs.

The nurse is completing the preoperative checklist on a client. The client states, "I take an aspirin every day for my heart." How does the nurse respond? a. "I will call your doctor and request a prescription for pain medication." b. "I need to call the surgeon and reschedule your surgery." c. "I'll give you the prescribed Tylenol to minimize any headache before surgery." d. "I need to administer vitamin K to prevent bleeding during the procedure."

ANS: B Aspirin and other salicylates interfere with platelet aggregation—the first step in the blood-clotting cascade—and decrease the ability of the blood to form a platelet plug. These effects last for longer than 1 week after just one dose of aspirin. The client may need to have the surgery rescheduled. Vitamin K, prescribed pain medication, and Tylenol cannot reduce the anticlotting effects of aspirin.

Which statement by a client with hypertension secondary to renal disease indicates the need for further teaching? a. "I can prevent more damage to my kidneys by managing my blood pressure." b. "If I have increased urination at night, I need to drink less fluid during the day." c. "I need to see the dietitian to discuss limiting my protein intake." d. "It is important that I take my antihypertensive medications as directed."

ANS: B Blood pressure control is needed to slow the progression of renal dysfunction. The client should not restrict fluids to prevent dehydration. Increased nocturnal voiding can be decreased by consuming the same amount of fluid earlier in the day. When dietary protein is restricted, refer the client to the registered dietitian as needed.

19. A client is receiving interleukin-2 (IL-2) for cancer. Which drug is the nurse prepared to administer if needed? a. Lorazepam (Ativan) b. Meperidine (Demerol) c. Furosemide (Lasix) d. Epoetin alfa (Epogen)

ANS: B Clients receiving IL-2 therapy usually experience chills, fever, and rigors during the infusion, especially the first time that they receive the drug. These reactions are a normal response to the administration of biological response modifiers such as IL-2. Clients are treated symptomatically for the discomfort. Demerol is used to treat the chills and rigor. The other medications would not treat a side effect of IL-2 therapy.

A client with polycystic kidney disease (PKD) has received extensive teaching in the clinic. Which statement by the client indicates that an important goal related to nutrition is being met? a. "I take a laxative every night before going to bed." b. "I have a soft bowel movement every morning." c. "Food tastes so much better since I can use salt again." d. "The white bread I am eating does not cause gas."

ANS: B Clients with PKD often have constipation, which can be managed with increased fiber, exercise, and drinking plenty of water. A soft bowel movement on a regular basis indicates that the client is preventing constipation. Laxatives should be used cautiously, and the need for their use indicates that the goal of preventing constipation via nutritional means is not being met. Clients with PKD should be on a restricted salt diet. White bread has a low fiber count and would not be included in a high-fiber diet.

A client is in the emergency department after experiencing kidney trauma. The abdomen is tender and distended, and blood is visible at the urinary meatus. Which action by the nurse is most appropriate? a. Assess vital signs and abdominal pain every 5 to 15 minutes. b. Consult with the provider before inserting a catheter. c. Monitor the client's IV rate and prepare to give blood. d. Assist with obtaining informed consent for surgery if needed.

ANS: B Clients with blood at the urinary meatus should not have a urinary catheter inserted via the urethra before additional diagnostic studies are done. The urethra could be torn. The nurse should question the provider about the need for a catheter; if one is needed, the provider can insert a suprapubic catheter. The other options are appropriate interventions.

The nurse is providing a client with a peritoneal dialysis exchange. The nurse notes the presence of cloudy peritoneal effluent. Which action by the nurse is most appropriate? a. Document the finding in the client's chart. b. Collect a sample to send to the laboratory. c. Reposition the client on the left side. d. Increase the free water content in the next bag. .

ANS: B Cloudy or opaque effluent is an early sign of peritonitis. The nurse should collect and send a sample for culture. Then the nurse should document the finding. The other two options are not appropriate

When providing care for a client receiving peritoneal dialysis, the nurse notices that the effluent is cloudy. Which intervention is most important for the nurse to carry out? a. Irrigate the peritoneal catheter with saline. b. Send a specimen for culture and sensitivity. c. Document the finding in the client's chart. d. Change the dialysate solution and catheter tubing.

ANS: B Cloudy or opaque effluent is the earliest sign of peritonitis. The health care provider should be notified, and a sample of the outflow should be sent for culture and sensitivity. Irrigating the catheter or changing the solution and tubing will not help reveal the cause of the problem so that appropriate treatment can be started. Documentation is important but is not the priority.

A client with polycystic kidney disease and hypertension is prescribed a diuretic for blood pressure control. Which statement by the client indicates the need for further teaching regarding these orders? a. "I will weigh myself every day at the same time." b. "I will drink only 1 liter of fluid each day." c. "I will avoid aspirin and aspirin-containing drugs." d. "I will avoid nonsteroidal anti-inflammatory drugs."

ANS: B Diuretics for blood pressure control can lead to fluid volume depletion and can decrease blood flow to the kidney, further decreasing renal function. The client should be instructed to drink at least 2500 mL/24 hr. NSAIDs should be used cautiously because they can reduce kidney blood flow. Aspirin products increase the risk for bleeding and should be avoided.

On assessment of a client with polycystic kidney disease (PKD), which finding is of greatest concern to the nurse? a. Flank pain b. Periorbital edema c. Bloody and cloudy urine d. Enlarged abdomen

ANS: B Flank pain and a distended or enlarged abdomen occur in PKD because the kidneys enlarge and displace other organs. Urine can be bloody or cloudy owing to cyst rupture or infection. Periorbital edema would not be a finding related to PKD and should be investigated further.

A client with diabetes is hospitalized with recurrent pyelonephritis. The provider orders IV gentamicin (Garamycin) before culture results come back. Which action by the nurse is best? a. Monitor the client's blood sugar before and after each dose. b. Consult with the pharmacist about the antibiotic selection. c. Monitor the client's daily blood urea nitrogen and creatinine levels. d. Check the client's most recent hemoglobin A1c result.

ANS: B Gentamycin is an aminoglycoside antibiotic and has nephrotoxic properties. People with diabetes are always at risk for diabetes-related kidney disease, and such agents should be avoided in this population. The nurse should consult the pharmacist for a list of antibiotics acceptable for empiric therapy for this client. Blood glucose does not need to be monitored more frequently when a client with diabetes receives antibiotics. Checking laboratory work is always an important nursing function, but client safety takes priority.

A client has nephrotic syndrome and a normal glomerular filtration. Which dietary selection shows that the client understands nutritional therapy for this condition? a. Decreased intake of protein b. Increased intake of protein c. Decreased intake of carbohydrates d. Increased intake of carbohydrates

ANS: B In nephrotic syndrome, the renal loss of protein is significant, leading to hypoalbuminemia and edema formation. If glomerular filtration is normal or near-normal, increased protein loss should be matched by increased intake of protein.

21. A client has small cell lung cancer. Which laboratory result requires immediate intervention by the nurse? a. Serum potassium of 5.1 mEq/L b. Serum sodium of 118 mEq/L c. Hematocrit of 45% d. Blood urea nitrogen (BUN) of 10 mg/dL

ANS: B In the syndrome of inappropriate antidiuretic hormone hypersecretion (SIADH), secretion of antidiuretic hormone (ADH) from the posterior pituitary gland is increased, causing the client to reabsorb water from the distal convoluted tubule and collecting duct. As a result, weight increases, and serum sodium and hematocrit levels are diluted. Blood urea nitrogen (BUN) and hematocrit are normal. Potassium is slightly high, but very low sodium places the client at risk for seizures and even death.

A client with chronic hypertension is seen in the clinic. Which assessment indicates that the client's hypertension is not under control? a. Heart rate of 55 beats/min b. Serum creatinine level of 1.9 mg/dL c. Blood glucose level of 128 mg/dL d. Irregular heart sounds

ANS: B Increased blood pressure damages the delicate capillaries in the glomerulus and eventually results in acute kidney injury. An elevated serum creatinine level is a manifestation of this. Heart rate, blood glucose level, and irregular heart sounds are not correlated with acute kidney injury.

12. A client's spouse reports that the last time the client received lorazepam (Ativan) before receiving chemotherapy, the client was extremely drowsy and didn't remember the trip home. Which is the nurse's best action? a. Hold the dose of lorazepam for this round of chemotherapy. b. Explain that this is a normal response to the drug. c. Perform a Mini-Mental State Examination. d. Document the response in the client's chart.

ANS: B Lorazepam, a benzodiazepine, induces sedation and amnesia, in addition to having antiemetic effects. Many clients have little if any memory about events occurring within a few hours after receiving lorazepam. This is an expected side effect and does not denote any permanent reduced cognition in the client. Both the client and the spouse should be aware of this effect so that the client is not at risk for injury. Driving, cooking, or operating mechanical equipment should not be performed until the drug's effects have worn off.

A client is admitted with a 3-day history of vomiting and diarrhea. The client's vital signs are blood pressure, 85/60 mm Hg; and heart rate, 105 beats/min. Which intervention by the nurse takes priority? a. Obtain blood and urine cultures. b. Start an IV of normal saline as ordered. c. Administer antiemetic medications. d. Assess the client's recent travel history.

ANS: B Many types of problems can reduce kidney function. Severe hypotension from shock or dehydration reduces renal blood flow and leads to prerenal acute renal failure (ARF). Volume depletion leading to prerenal azotemia is the most common cause of ARF and usually is reversible with prompt intervention. The nurse should first initiate the ordered IV fluids. Obtaining cultures will help identify a possible cause of the client's symptoms and should be done quickly after the IV has been started. Attending to the client's discomfort would be next. Assessing for travel history, although important, can wait until after the other interventions have been accomplished.

A client who underwent kidney transplantation 7 days ago has developed the following signs: urine output, 50 mL/12 hr; temperature, 102.2° F (39° C); lethargy; serum creatinine, 2.1 mg/dL; blood urea nitrogen (BUN), 54 mg/dL; and potassium, 5.6 mEq/L. Which initial intervention does the nurse anticipate for this client? a. Immediate hemodialysis b. Increased dose of immune suppressive drugs c. Initiation of IV antibiotics after cultures are obtained d. Placement of a catheter for peritoneal dialysis

ANS: B Oliguria, lethargy, elevated temperature, and increases in serum electrolyte levels, BUN, and creatinine, 1 week to 2 years post-transplantation are hallmarks of acute rejection, which can be reversible with increased immune suppressive therapy. The client does not need hemodialysis, peritoneal dialysis, or antibiotics at this point.

7. A client is undergoing radiation therapy and says, "I will be so glad when this is over and I don't have to worry about my skin." What response by the nurse is most appropriate? a. "Unfortunately, your skin will be permanently damaged from the radiation." b. "You need to protect your skin from the sun for at least a year afterward." c. "You can get a prescription for special lotions that reduce the effects of radiation." d. "You're having skin problems? That is unusual; let me take a look at your skin."

ANS: B Skin that has been in the path of external radiation is more susceptible to sun damage and must be protected from the sun for at least a year after completion of radiation therapy. Skin changes due to radiation are common but may not be permanent, depending on the amount of radiation absorbed. No one skin care product has been shown to significantly help radiation-related skin problems.

A client has a serum creatinine level of 2 mg/dL and a urine output of 1000 mL/day. How does the nurse categorize the client's kidney injury? a. Intrarenal b. Nonoliguric c. Prerenal d. Postrenal

ANS: B Some clients have a nonoliguric form of acute renal failure (ARF), in which urine output remains near-normal but creatinine rises. The other categories relate to the cause of acute kidney injury.

The nurse is caring for a client who is receiving heparin therapy. How does the nurse evaluate the therapeutic effect of the therapy? a. Evaluate platelets. b. Monitor the partial thromboplastin time (PTT). c. Assess bleeding time. d. Monitor fibrin degradation products.

ANS: B The PTT assesses the intrinsic clotting cascade. Heparin therapy is monitored by the PTT. Platelets are monitored by the platelet count laboratory value, bleeding time evaluates vascular and platelet activity during hemostasis, and fibrin degradation products help assess for fibrinolysis.

9. A client's radiation implant has become dislodged overnight, and the nurse finds it in the client's bed. What does the nurse do first? a. Assess the client's skin for radiation burns. b. Use tongs to put the implant into the radiation container. c. Notify the safety officer and move the client to a different room. d. Don gloves and attempt to replace the implant.

ANS: B The implant does emit radiation and should be placed into the secure, lead-lined container in the client's room. The nurse does not directly touch this implant but uses long-handled tongs for this purpose. The nurse does not need to assess the client's skin, nor should he or she attempt to replace the source. Moving the client is not necessary, although in keeping with facility policy, the radiation safety officer may need to be notified.

A client has nephrotic syndrome. Which finding shows that therapy is effective? a. Serum albumin level, 2.8 g/dL b. Serum albumin level, 4 g/dL c. Urine protein level, 3.7 g/24 hr d. Potassium, 4.2 mEq/L

ANS: B The main diagnostic findings in nephrotic syndrome are severe proteinuria, low serum albumin, high serum lipids, and fat in the urine. A serum albumin of 4 g/dL is within the normal range, showing that therapy is working. An albumin level of 2.8 g/dL is low, and proteinuria of 3.7 g/24 hr is high, showing that the disease is not yet controlled. Potassium is not affected.

The nurse is teaching a client who has undergone a bone marrow biopsy. Which instruction does the nurse give the client? a. "Wear protective gear when playing contact sports." b. "Monitor the biopsy site for bruising." c. "Remain in bed for at least 12 hours." d. "Use a heating pad for pain at the biopsy site."

ANS: B The most important instruction is to have the client monitor the area for external or internal bleeding. Activities such as contact sports should be avoided, and an ice pack can be used to limit bruising.

The nurse is planning care for a client who has a platelet count of 30,000/mm3. Which intervention does the nurse include in this client's plan of care? a. Oxygen by nasal cannula b. Bleeding Precautions c. Isolation Precautions d. Vital signs every 4 hours

ANS: B The normal platelet count ranges between 150,000 and 400,000/mm3. This client is at extreme risk for bleeding. Although it is necessary to notify the provider, the nurse would first protect the client by instituting Bleeding Precautions. The other interventions are not related to the low platelet count.

The nurse helps to ambulate a client who has anemia. Which clinical manifestation indicates that the client is not tolerating the activity? a. Blood pressure of 120/90 mm Hg b. Heart rate of 110 beats/min c. Pulse oximetry reading of 95% d. Respiratory rate of 20 breaths/min

ANS: B The red blood cells contain thousands of hemoglobin molecules. The most important feature of hemoglobin is its ability to combine loosely with oxygen. A low hemoglobin level can cause decreased oxygenation to the tissues, thus causing a compensatory increase in heart rate. The other options are close to normal range and are not indicative of not tolerating this activity.

8. A client scheduled to undergo radiation therapy for breast cancer asks why 6 weeks of daily treatment is necessary. What is the nurse's best response? a. "Your cancer is widespread and requires more than the usual amount of radiation treatment." b. "Giving larger doses of radiation for a shorter period of time does not produce better effects and has worse side effects." c. "Research has shown that more cancer cells are killed if radiation is given in smaller doses over a longer time period." d. "It is less likely that your hair will fall out or that you will become anemic if radiation is given in this manner."

ANS: C Because of varying responses of all cancer cells within a given tumor, small doses of radiation are given on a daily basis for a set period of time. This method allows multiple opportunities to destroy cancer cells while minimizing damage to normal tissues.

A nurse is caring for several clients at risk for overhydration. The nurse assesses the older client with which finding first? a. Has had diabetes mellitus for 12 years b. Uses sodium-containing antacids frequently c. Just received 3 units of packed red blood cells d. Had abdominal surgery and has a nasogastric tube

ANS: C Blood replacement therapy involves intravenous fluid administration, which inherently increases the risk for overhydration. The fact that the fluid consists of packed red blood cells greatly increases the risk, because this fluid increases the colloidal oncotic pressure of the blood, causing fluid to move from interstitial and intracellular spaces into the plasma volume. An older adult may not have sufficient cardiac or renal reserve to manage this extra fluid.

10. A client is receiving a chemotherapeutic agent intravenously through a peripheral line. What is the nurse's first action when the client reports burning at the site? a. Check for a blood return. b. Slow the rate of infusion. c. Discontinue the infusion. d. Apply a cold compress.

ANS: C Both irritants and vesicants can cause tissue damage. If the nurse suspects extravasation, he or she should immediately stop the infusion. Even if the IV has a good blood return, some of the chemotherapeutic agent can still be leaking into the tissues. Slowing the rate of infusion is not sufficient to prevent further leakage and damage. Applying a cold compress may or may not be the correct action, depending on the specific agent. However, the compress would be applied only after the infusion has been discontinued.

The nurse is preparing a client for a bone biopsy and aspiration. The client asks, "Will this be painful?" How does the nurse respond? a. "The procedure is always done under general anesthesia." b. "The biopsy lasts for only 2 minutes." c. "There is a chance that you may have pain." d. "You can relieve pain with guided imagery."

ANS: C Clients may have pain during this procedure. The type and amount of anesthesia or sedation depend on the physician's preference, the client's preference, and previous experience with bone marrow aspiration. The procedure takes from 5 to 15 minutes. Guided imagery can relieve pain but works well only with some clients.

A client with chronic kidney disease is scheduled to be given the following medications: digoxin (Lanoxin) and epoetin alfa (Epogen). The client reports nausea and vomiting and wishes to wait to take the medications. Which action by the nurse is most appropriate? a. Administer both medications with soda crackers. b. Allow the client to wait an hour before taking the medications. c. Review today's potassium level and notify the health care provider. d. Call the health care provider to get an order for anti-nausea medication.

ANS: C Clients with kidney failure are particularly at risk for digoxin toxicity because the drug is excreted by the kidneys. When caring for clients with chronic kidney disease (CKD) who are receiving digoxin, monitor for signs of toxicity, such as nausea and vomiting. Potassium imbalances can alter digoxin levels as well. The nurse should hold the dose, check the current potassium level, and notify the provider. Giving the digoxin could be dangerous, so the nurse should not administer it with crackers, give it later, or ask for an anti-nausea medication.

29. The nurse prioritizes which intervention in a client with xerostomia secondary to radiation therapy to the neck area? a. Applying lotions and oils to affected areas b. Wearing a hat to decrease heat loss c. Providing oral care after meals and at bedtime d. Monitoring vital signs every 4 hours

ANS: C Head and neck radiation may damage the salivary glands, may cause dry mouth (xerostomia), and may increase the client's lifelong risk for tooth decay. Instruct clients to avoid using lotions or ointments in these areas unless the radiologist prescribes them. Xerostomia is not associated with hair loss, which might require a hat. Monitoring vital signs is important for any client receiving radiation therapy but is not a priority for the client with xerostomia.

A client in the emergency department has potassium of 2.9 mEq/L. For which disease process or condition does the nurse assess the client? a. Diabetes mellitus b. Addison's disease c. Hyperaldosteronism d. Diabetes insipidus

ANS: C Hyperaldosteronism results in increased reabsorption of sodium and water while enhancing excretion of potassium. Therefore, any client with this condition is at high risk for the development of hypokalemia.

A female client is admitted with the medical diagnosis of anemia. The nurse assesses for which potential cause? a. Diet high in meat and fat b. Daily intake of aspirin c. Heavy menses d. Smoking history

ANS: C Iron levels can be low because intake of iron is too low, or because loss of iron through bleeding is excessive. A premenopausal woman may be having unusually heavy menses sufficient to cause excessive loss of blood and iron. Smoking and aspirin do not cause iron deficiency. A diet high in meat provides iron. pg 861

The nurse observes yellow-tinged sclera in a client with dark skin. Based on this assessment finding, what does the nurse do next? a. Assess the client's pulses. b. Examine the soles of the client's feet. c. Inspect the client's hard palate. d. Auscultate the client's lung sounds.

ANS: C Jaundice can best be observed in clients with dark skin by inspecting the oral mucosa, especially the hard palate, for yellow discoloration. Because sclera may have subconjunctival fat deposits that show a yellow hue, and because foot calluses may appear yellow, neither of these areas should be used to assess for jaundice. The client's pulse and lung sounds have no correlation with an assessment of jaundice.

A client with a decreased glomerular filtration rate asks how to prevent further damage to the kidneys. Which is the nurse's best response? a. "The diuretics you are taking will prevent further damage." b. "Kidney damage is inevitable as you age." c. "Avoid taking NSAIDs." d. "You will need to follow a high-protein diet."

ANS: C Kidney failure causes many problems, including decreased glomerular filtration rate. Nephrotoxins can worsen renal failure, especially in someone who already has some loss of kidney function.

A client is taking furosemide (Lasix). To detect a common adverse effect, the nurse obtains which assessment as a priority? a. Breath sounds b. Heart sounds c. Intake and output d. Nutritional patterns

ANS: C Lasix is a diuretic that causes increased urine output. If too much urine output occurs, the client may be at risk for hypovolemia, which is a cause of prerenal kidney failure. A marked change in fluid balance seen in the intake and output measurement can help identify the client who may be at risk for hypovolemia. Heart sounds and breath sounds would be more important to assess if the client was receiving Lasix for fluid overload conditions, such as heart failure. Nutrition assessment is important to ensure that the client gets enough potassium, but dehydration is more common and needs more vigorous assessment.

A client is hospitalized with a urinary tract infection (UTI). Which clinical manifestation alerts the nurse to the possibility of a complication from the UTI? a. Burning on urination b. Cloudy, dark urine c. Fever and chills d. Hematuria

ANS: C Lower urinary tract infections are rarely associated with systemic symptoms of fever and chills. A client with a UTI who develops fever and chills should be assessed for the development of pyelonephritis. The other options can be seen with UTI.

30. Which statement indicates that the client needs more teaching about mucositis? a. "I will rinse my mouth with water after every meal." b. "I will use a soft-bristled toothbrush to prevent trauma." c. "I should use an alcohol-based mouth rinse to kill bacteria." d. "I cannot use floss because it may irritate my gums."

ANS: C Mouthwashes that contain alcohol are drying and can exacerbate mucosal irritation, leading to painful mouth sores. Rinsing the mouth with water or normal saline is indicated. Interventions aimed at decreasing risk for trauma or irritation are matters of priority because of inflammation associated with mucositis.

23. A nurse is reviewing the white blood cell count with differential for a client receiving chemotherapy for cancer. Which finding alerts the nurse to the possibility of sepsis? a. Total white blood cell count is 9000/mm3. b. Lymphocytes outnumber basophils. c. "Bands" outnumber "segs." d. Monocyte count is 1800/mm3.

ANS: C Normally, mature segmented neutrophils ("segs") are the major population of circulating leukocytes, constituting 55% to 70% of the total white blood cell count. Less than 3% to 5% of circulating white blood cells should be the less mature band neutrophils. A left shift occurs when the bone marrow releases more immature neutrophils than mature neutrophils. This condition indicates severe infection with possible sepsis and must be explored further.

3. A client who has just had a mastectomy is crying. When the nurse asks about her crying, the client responds, "I know I shouldn't cry because this surgery may well save my life." What is the nurse's best response? a. "It is all right to cry. Mourning this loss will help make you stronger." b. "I know this is hard, but your chances of survival are better now." c. "I can arrange for someone who had a mastectomy to come visit if you like." d. "How have you coped with difficult situations in the past?"

ANS: C Often, cancer surgery involves the loss of a body part or a decrease in function. Mourning or grieving for a body image alteration is a healthy part of adapting or adjusting to a new image. Visiting with someone who has experienced the same situation as the client is very helpful in showing the client that many aspects of life can be the same afterward. If the opportunity to arrange this type of visit is available, this would be the nurse's best response. The other options do not provide any assistance to the client in coping with her new body image and grieving for her loss.

The nurse is caring for a client who is receiving peritoneal dialysis (PD). Which nursing intervention has the greatest priority when a dialysis exchange is performed? a. Adding potassium and antibiotic to the dialysate bags b. Positioning the client on either side c. Using sterile technique when hooking up dialysate bags d. Warming the dialysate fluid in a microwave oven

ANS: C Peritonitis is the major complication of PD. The most common cause of peritonitis is connection site contamination. To prevent peritonitis, use meticulous sterile technique when caring for the PD catheter and when hooking up or clamping off dialysate bags. This safety precaution is the priority. Never warm dialysate fluid in the microwave. Positioning the client may help with the flow of fluid. Clients may need additives to their dialysate fluid, but potassium and antibiotics are not added together because interactions between them can reduce the effectiveness of the antibiotic.

A client has been diagnosed with acute postrenal kidney injury. Which assessment finding does the nurse assess most carefully for? a. Blood urea nitrogen (BUN), 35 mg/dL b. Creatinine, 2.5 mg/dL c. Feeling of urgency d. Weight gain and edema

ANS: C Postrenal kidney failure is identified by focusing on urinary obstructive problems. Symptoms include changes in the urine stream or difficulty starting urination. All the other distractors can be seen with prerenal and intrarenal kidney injury.

Assessment findings reveal that a client with chronic kidney disease is refusing to take prescribed medications because of the "cost." The client also is having difficulty performing activities of daily living and prefers to sleep most of the day. To which health care team member does the nurse refer the client? a. Home health aide b. Physical therapist c. Psychiatric nurse practitioner d. Physician

ANS: C Professionals from many disciplines are resources for the client with renal failure. A psychiatric evaluation may be needed if depressive symptoms are present. Refusing treatment, having difficulty performing activities of daily living, and excessive sleeping could be signs of depression.

The nurse is caring for a client who is receiving chemotherapy for cancer. Which intervention does the nurse implement for this client? a. Assess the client's fibrinogen level. b. Administer the prescribed iron. c. Maintain strict Standard Precautions. d. Monitor the client's pulse oximetry

ANS: C The client who is receiving chemotherapy drugs that suppress the bone marrow will be at risk for a decreased white blood cell (WBC) count and infection. The nurse will be most therapeutic by adhering to Standard Precautions to prevent infection, such as handwashing. The nurse will not expect the fibrinogen level to be affected by this therapy. Iron is not typically administered with chemotherapy because this is bone marrow suppression, so the administration of epoetin (Epogen) or filgrastim (Neupogen) is most effective. Monitoring the pulse oximetry is part of routine care and probably would not need to be done continuously.

An older client is hospitalized with suspected heart failure. After 2 days of treatment, the client is not improving. Which laboratory value does the nurse report to the provider? a. Potassium, 3.7 mEq/L b. Sodium, 144 mEq/L c. Glomerular filtration rate, 55 mL/min d. Creatinine, 0.9 mg/dL

ANS: C The client's GFR is extremely low; this can correspond with kidney disorders, including acute glomerulonephritis (GN). Because of pulmonary and cardiac congestion that accompanies acute GN, the condition can be mistaken for heart failure, especially in the older adult. The nurse should report this laboratory value so the client can undergo additional diagnostic studies. The other laboratory values are normal.

The nurse is caring for four clients with hematologic-type problems. Which client does the nurse prioritize to see first? a. 18-year-old female with decreased protein levels b. 36-year-old male with increased lymphocytes c. 60-year-old female with decreased erythropoietin d. 82-year-old male with an increased thromboxane level

ANS: C The kidney releases more erythropoietin when tissue oxygenation levels are low. This growth factor then stimulates the bone marrow to increase red blood cell (RBC) production, which improves tissue oxygenation and prevents hypoxia. Hypoxia causes the body to increase its respiratory rate to overcome decreased oxygenation of the tissues. All these clients are important, but the woman with decreased erythropoietin takes priority because of her risk for hypoxia.

The nurse is performing an admission assessment on a 46-year-old client, who states, "I have been drinking a 12-pack of beer every day for the past 20 years." Which laboratory abnormality does the nurse correlate with this history? a. Decreased white blood cell (WBC) count b. Decreased bleeding time c. Elevated prothrombin time (PT) d. Elevated red blood cell (RBC) count

ANS: C The liver is the site for production of prothrombin and most of the blood-clotting factors. If the liver is damaged because of chronic alcoholism, it is unable to produce these clotting factors. Therefore, the PT could become elevated, which would reflect deficiency of some clotting factors. The WBC would not be elevated in this situation because no infection is present. Bleeding time would likely increase. The client's RBC count most likely would not be affected unless the client was bleeding, in which case it would decrease.

A client has a serum creatinine level of 2.5 mg/dL, a serum potassium level of 6 mmol/L, an arterial pH of 7.32, and a urine output of 250 mL/day. Which phase of acute kidney failure is the client experiencing? a. Intrarenal b. Nonoliguric c. Oliguric d. Postrenal

ANS: C The oliguric phase of acute kidney failure is characterized by the accumulation of nitrogenous wastes, resulting in increasing levels of serum creatinine and potassium, bicarbonate deficit, and decreased or no urine output. Intrarenal and postrenal refer to causes of kidney injury. Nonoliguric is not a classification.

13. A client is on chemotherapy and has a platelet count of 25,000. Which intervention is most important to teach this client? a. "Eat a low-bacteria diet." b. "Take your temperature daily." c. "Use a soft-bristled toothbrush." d. "Avoid alcohol-based mouthwashes."

ANS: C This client has thrombocytopenia, which is a common side effect of chemotherapy. This increases the client's risk for prolonged bleeding in response to even minor injury, especially from highly vascular areas such as the gums. The client should be taught to use a soft toothbrush. A low-bacteria diet and daily temperature monitoring would be used in a client who is neutropenic. Alcohol-based mouthwashes will dry mucous membranes.

A client with chronic kidney disease states that he will be going to the dentist for a planned tooth extraction. Which is the nurse's best response? a. "Rinse your mouth with an antiseptic solution after the procedure." b. "Kidney disease is probably what caused your dental decay." c. "You should receive prophylactic antibiotics before any dental procedure." d. "You may take any medication for pain that the dentist prescribes."

ANS: C To prevent sepsis from oral cavity bacteria, the client should be given prophylactic antibiotics before any dental procedure. Rinsing the mouth with antiseptic solution would not be sufficient to prevent infection. Kidney disease may have contributed to the dental decay through loss of calcium from the teeth, but this cannot be confirmed. Clients with kidney disease should not take antibiotics known to be nephrotoxic. Dosage adjustments based on the client's kidney function may be needed.

Which statement made by the client who has kidney stones from secondary hyperoxaluria indicates correct understanding of the role of dietary therapy for this condition? A. "No modifications are needed because this type of stone is not caused by diet." B. "I will avoid dark green leafy vegetables, chocolate, and nuts." C. "I will avoid all dairy products and vitamin D." D. "I will avoid wine, meat, and shellfish."

B -"I will avoid dark green leafy vegetables, chocolate, and nuts." -Secondary hyperoxaluria is caused by an excessive ingestion of foods containing large amounts of oxalate, such as spinach, rhubarb, Swiss chard, collard greens, cocoa, beets, wheat germ, pecans, peanuts, okra, chocolate, and lime peel.

3. Which of the following muscle actions results in voluntary urination? a. Detrusor contraction, external sphincter contraction b. Detrusor contraction, external sphincter relaxation c. Detrusor relaxation, external sphincter contraction d. Detrusor relaxation, external sphincter relaxation

B -Detrusor contraction, external sphincter relaxation -Voiding becomes a voluntary act as a result of learned responses controlled by the cerebral cortex. This causes contraction of the bladder detrusor muscle and simultaneous relaxation of the external urethral sphincter muscle.

Which statement by a client who has undergone kidney transplantation indicates a need for more teaching? a. "I will need to continue to take insulin for my diabetes." b. "I will have to take my cyclosporine for the rest of my life." c. "I will take the antibiotics three times daily until the medication is finished." d. "My new kidney is working fine. I do not need to take medications any longer."

ANS: D A crucial role of the nurse in long-term follow-up of the kidney transplantation client involves maintenance of prescribed drug therapy. Such clients will need to take immune suppressants for the rest of their lives to prevent rejection of the kidney.

The nurse is assessing the following laboratory results of a client before discharge. Which instruction does the nurse include in this client's discharge teaching plan? Test Result Hemoglobin 15 g/dL Hematocrit 45% White blood cell (WBC) count 2000/mm3 Platelet count 250,000/mm3 a. "Avoid contact sports." b. "Do not take any aspirin." c. "Eat a diet high in iron." d. "Perform good hand hygiene."

ANS: D A normal WBC count is 5000 to 10,000/mm3. A white blood cell count of 2000/mm3 is low and makes this client at risk for infection. Good handwashing technique is the best way to prevent the transmission of infection. The other laboratory results are all within normal limits.

The nurse is assessing a client with liver failure. Which assessment is the highest priority for this client? a. Auscultation for bowel sounds b. Assessing for deep vein thrombosis c. Monitoring of blood pressure hourly d. Assessing for signs of bleeding

ANS: D All these options are important in assessment of the client, but the most important action is assessment for signs of bleeding. The liver is the site of production of prothrombin and most of the blood-clotting factors. Clients with liver failure run a high risk of having problems with bleeding.

26. A client has late-stage colon cancer with metastasis to the spine and bones. Which nursing intervention does the nurse add to the care plan to address a priority problem? a. Provide six small meals and snacks daily. b. Offer the client prune juice twice a day. c. Ensure that the client gets adequate rest. d. Give the client pain medications around the clock.

ANS: D Although all interventions might be appropriate, a client with late-stage cancer and bone metastases is at risk for severe pain. Giving the client pain medication around the clock is the best way to manage this type of pain.

A client who has chronic kidney disease is being discharged from the hospital after receiving treatment for a hip fracture. Which information is most important for the nurse to provide to the client before discharge? a. "Increase your intake of foods with protein." b. "Monitor your daily intake and output." c. "Maintain bedrest until the fracture is healed." d. "Take your aluminum hydroxide (Nephrox) with meals."

ANS: D Aluminum hydroxide lowers serum phosphate levels by binding phosphorus present in food. High blood phosphate levels cause hypocalcemia and osteodystrophy; this makes a client prone to fracture. Increasing protein may not be feasible for a client with chronic kidney disease and would not help prevent fracture. Intake and output will not be helpful for orthopedic problems. Bedrest will promote complications.

Which intervention is most important for the nurse to implement in a client after kidney transplant surgery? a. Promote acceptance of new body image. b. Monitor magnesium levels daily. c. Place the client on protective isolation. d. Remove the indwelling (Foley) catheter as soon as possible.

ANS: D Because of increased risk for infection related to immune suppressive drugs given to prevent rejection, the catheter is removed as soon as possible to avoid infection, usually 3 to 7 days after surgery. The client may need assistance with changes in body image, but this is not the priority. The client does not require protective precautions. Laboratory values will be monitored frequently in a post-transplant client, but this is not as important as preventing a complication by removing the catheter.

1. What statement indicates that the client understands teaching about neutropenia? a. "I need to use a soft toothbrush." b. "I have to wear a mask at all times." c. "My grandchildren may get an infection from me." d. "I will call my doctor if I have an increase in temperature."

ANS: D Bone marrow suppression leads to neutropenia and increases the client's risk for infection. Decreased numbers of neutrophils and other white blood cells can minimize the clinical manifestations of infection. For this reason, the client may not develop a high temperature, even with severe infection, and any elevation of temperature should be reported immediately to the health care provider. The client does not need to wear a mask or use a soft toothbrush (although if the client has low platelets, he or she should use a soft toothbrush to avoid causing trauma). The client is not contagious.

14. A client with chemotherapy-induced bone marrow suppression has received filgrastim (Neupogen). Which laboratory finding indicates that this therapy is effective for the client? a. Hematocrit is 28%. b. Hematocrit is 38%. c. Segmented neutrophil count is 2500/mm3. d. Segmented neutrophil count is 3500/mm3.

ANS: D Filgrastim is a single-lineage growth factor that stimulates the maturation and release of only segmented neutrophils. This drug is not given unless the neutrophil count is dangerously low. The near-normal range of neutrophils indicates effective therapy.

A client presents with senile dementia, Alzheimer's type (SDAT) and incontinence. Which therapy will best help this client? A. Bladder training B. Habit training C. Exercise therapy D. Electrical stimulation

B -Habit training -Habit training is the type of bladder training that will be most effective with cognitively impaired clients. Bladder training can only be used with a client who is alert, aware, and able to resist the urge to urinate. Exercise therapy may be too difficult for the cognitively impaired client to grasp, and electrical stimulation will be traumatic for this client.

During hemodialysis, a client with chronic kidney disease develops headache, nausea, vomiting, and restlessness. After notifying the health care provider, which action by the nurse is most appropriate? a. Administer a bolus of dextrose solution. b. Draw blood for sodium and potassium. c. Order a blood urea nitrogen level stat. d. Prepare to administer phenytoin (Dilantin).

ANS: D Headache, nausea, vomiting, and restlessness may be signs of dialysis disequilibrium syndrome. Rapid decreases in fluid and in blood urea nitrogen (BUN) level can cause cerebral edema and increased intracranial pressure (ICP). Early recognition and treatment of this syndrome are essential for preventing a life-threatening situation. Treatment includes administration of anticonvulsants (Dilantin) or barbiturates. Dextrose is not used to treat disequilibrium syndrome, and sodium and potassium levels are not helpful because the symptoms are related to changes in urea levels and increased intracranial pressure. Obtaining the BUN would provide useful information; however, it is more important to treat the problem.

11. A client receiving intravenous chemotherapy asks the nurse the reason for wearing a mask, gloves, and gown while administering drugs to the client. What is the nurse's best response? a. "These coverings protect you from getting an infection from me." b. "I am preventing the spread of infection from you to me or any other client here." c. "The policy is for any nurse giving these drugs to wear a gown, gloves, and mask." d. "The clothing protects me from accidentally absorbing these drugs."

ANS: D Most chemotherapy drugs are absorbed through the skin and mucous membranes. As a result, health care workers who prepare or give these drugs, especially nurses and pharmacists, are at risk for absorbing them. Even at low doses, chronic exposure to chemotherapy drugs can affect health. The Oncology Nursing Society and the Occupational Safety and Health Administration (OSHA) have specific guidelines for using caution and wearing protective clothing whenever preparing, giving, or disposing of chemotherapy drugs.

A client hospitalized for worsening kidney injury suddenly becomes restless and agitated. Assessment reveals tachycardia and crackles bilaterally at the bases of the lungs. Which is the nurse's first intervention? a. Begin ultrafiltration. b. Administer an antianxiety agent. c. Place the client on mechanical ventilation. d. Place the client in high Fowler's position.

ANS: D Restlessness, anxiety, tachycardia, dyspnea, and crackles at the bases of the lungs are early manifestations of pulmonary edema, which is a complication of kidney failure. Initial treatment of pulmonary edema consists of placing the client in high Fowler's position and administering oxygen. Mechanical ventilation and ultrafiltration may be indicated if symptoms become worse. An antianxiety agent would not be helpful. Morphine, however, has both vasoactive and sedating effects.

A client with glomerulonephritis has a glomerular filtration rate (GFR) of 40 mL/min, as measured by a 24-hour creatinine clearance. Which is the nurse's interpretation of this finding? a. Excessive GFR, client at risk for dehydration b. Excessive GFR, client at risk for fluid overload c. Reduced GFR, client at risk for dehydration d. Reduced GFR, client at risk for fluid overload

ANS: D The GFR refers to the initial amount of urine that the kidneys filter from the blood. In the healthy adult, the normal GFR ranges between 100 and 120 mL/min, most of which is reabsorbed in the kidney tubules. A GFR of 40 mL/min is drastically reduced, with the client experiencing fluid retention and risks for hypertension and pulmonary edema as a result of excess vascular fluid.

In assessing a client recently diagnosed with acute glomerulonephritis, the nurse asks which question to determine potential contributing factors? a. "Are you sexually active?" b. "Do you have pain or burning on urination?" c. "Has anyone in your family had chronic kidney problems?" d. "Have you had a cold or sore throat within the last 2 weeks?"

ANS: D The most common cause of acute glomerulonephritis is the presence of a systemic infection (often a skin or respiratory infection) resulting in the formation of antigen-antibody complexes, which precipitate in the kidney tissues. The other questions would not assess for contributing causes.

27. After receiving change-of-shift report, which client does the nurse assess first? a. Client with leukemia who needs an antiemetic before chemotherapy b. Client with breast cancer scheduled for external beam radiation c. Client with xerostomia associated with laryngeal cancer d. Client with neutropenia who has just been admitted with a possible infection

ANS: D The most complex, potentially unstable client is the one with neutropenia with suspected infection. Because the onset of infection is insidious in clients with neutropenia, this client is at risk for sepsis. All other clients are stable.

A client returned to the nursing unit after having a nephrostomy performed. Over the next 6 hours, drainage in the tube has gone from 40 mL/hr to 12 mL over the last hour. Which intervention by the nurse is most appropriate? a. Document the finding in the client's record. b. Evaluate the tube as working in the hand-off report. c. Clamp the tube in preparation for removing it. d. Assess the client's abdomen and vital signs.

ANS: D The nephrostomy tube should continue to have a consistent amount of drainage. If the drainage slows or stops, it may be obstructed. The nurse must notify the provider, but first should carefully assess the client's abdomen for pain and distention and check vital signs so that this information can be reported too. The other interventions are not appropriate.

In assessing a client 6 hours after a radical nephrectomy for renal cell carcinoma, the nurse notes that the client's blood pressure has decreased from 134/90 to 100/56 mm Hg and urine output is 20 mL for this past hour. Which is the nurse's best action? a. Position the client so that the remaining kidney is not dependent. b. Measure the specific gravity of the client's urine. c. Document the findings in the client's record. d. Assess the pulse rate and quality, and then notify the provider.

ANS: D The nurse should fully assess the client for signs of volume depletion and shock, then should notify the provider. The radical nature of the surgery and the proximity of the surgery to the adrenal gland put the client at risk for hemorrhage and adrenal insufficiency. Hypotension is a clinical manifestation associated with both hemorrhage and adrenal insufficiency. Hypotension is particularly dangerous for the remaining kidney, which must receive adequate perfusion to function effectively. Documentation is critical but is not the priority at this time. The other two options would not be helpful interventions.

Which assessment parameter does the nurse monitor in a client with chronic kidney disease to determine fluid and sodium retention status? a. Capillary refill b. Intake and output c. Muscle strength d. Weight and blood pressure

ANS: D Weight and blood pressure are helpful in estimating fluid and sodium retention. Weight and blood pressure rise with excess fluid and sodium. Weight is the most accurate noninvasive assessment for fluid status and therefore sodium status. Capillary refill also gives information on perfusion and oxygenation so is not specific for fluid status. Intake and output are part of the assessment for fluid status but do not account for insensitive water losses. Muscle strength is unrelated.

The nurse is caring for a client who has an elevated white blood cell count. Which intervention does the nurse implement for this client? a. Administer the prescribed Tylenol. b. Hold the client's prescribed steroids. c. Assess the client's respiratory rate. d. Obtain the client's temperature.

ANS: D White blood cells provide immunity and protect against invasion and infection. An elevated white blood cell count could indicate an infectious process, which could cause an elevation in body temperature. Tylenol would treat a fever but not the elevated white blood cell count. Steroids place the client at higher risk for infection but should not be stopped suddenly. The respiratory rate does not need to be assessed in this client.

The nurse is caring for a post operative client on the med surg unit following a total left hip replacement the previous day. During the assessment, the nurse notes that the client's left leg is cool, with weak pedal pulses. What is the nurse's first action?

Assess circulatory status of the right leg.

The client is taking a medication that inhibits aldosterone secretion and release. The nurse assesses for what potential complication? a. Fluid retention b. Hyperkalemia c. Hyponatremia d. Hypervolemia

B - Hyperkalemia -Aldosterone is a naturally occurring hormone of the mineralocorticoid type that increases the reabsorption of water and sodium in the kidney at the same time that it promotes excretion of potassium. Any drug or condition that disrupts aldosterone secretion or release increases the client's risk for excessive water loss and increased potassium reabsorption. The client would not be at risk for overhydration or sodium imbalance.

The RN assigned a new nurse to a client who was receiving chemotherapy through an intravenous extension set attached to a Huber needle. Which information about disconnecting the Huber needle is most important for the RN to provide to this nurse?

B -"Be aware of a rebound effect when discontinuing the system." -Huber needles are used to access implanted ports placed under the skin. Because the dense septum holds tightly to the needle, there can be a rebound when it is pulled from the septum, often resulting in needle stick injury to the nurse. Topical anesthetic cream can be used when accessing the system. Flushing is carried out when the system is accessed and once monthly. Because the implanted port is not being removed, there is no need for a pressure dressing.

Which client statement indicates understanding regarding antibiotic therapy for recurrent urinary tract infections? A. "If my urine becomes lighter and clear, I can stop taking my medicine." B. "Even if I feel completely well, I should take the medication until it is gone." C. "When my urine no longer burns, I will no longer need to take the antibiotics." D. "If my temperature goes above 100° F (37.8° C), I should take twice as much medicine."

B -"Even if I feel completely well, I should take the medication until it is gone." -Antibiotic therapy is most effective, especially for recurrent urinary tract infections, when the client takes the prescribed medication for the entire course and not just when symptoms are present. The other statements demonstrate that additional teaching is needed for the client.

The postmenopausal female client has had two episodes of bacterial urethritis in the last 6 months. She asks her nurse why this is happening to her now. Which is the nurse's best response? A. "Your immune system becomes less effective as you age." B. "Low estrogen levels can make the tissue more susceptible to infection." C. "You should be more careful with your personal hygiene in this area." D. "It is likely that your sexual partner is traumatizing this area."

B -"Low estrogen levels can make the tissue more susceptible to infection." -Low estrogen levels decrease moisture and the type of secretions in the perineal area, predisposing it to the development of infection. The client's immune system, personal hygiene, and sexual practices do not place her at risk for developing urethritis.

Which client statement indicates the need for more teaching regarding identification of the early manifestations of hypokalemia? a. "I have been weighing myself every day." b. "When I am constipated, I drink more fluids." c. "When my muscles feel weak, I eat a banana." d. "I check my pulse each morning and each night."

B -"When I am constipated, I drink more fluids." -The intestinal tract is relatively sensitive to decreasing potassium levels. Common manifestations of hypokalemia are decreased peristalsis and constipation.

A client is being discharged with mild dehydration. Which statement by the client indicates an understanding of measures to prevent mild dehydration from becoming more severe? a. "I will weigh myself at the same time daily wearing the same clothes." b. "When I feel lightheaded, I will drink a full glass of water." c. "I will decrease my fluid intake if my urine output increases." d. "If I forget to take my diuretic, I will take twice the dose next time."

B -"When I feel lightheaded, I will drink a glass of water." -Feeling lightheaded or dizzy is an indication of low blood pressure and poor perfusion. Mild dehydration can cause these problems and increasing fluid intake at the first sign of dehydration may prevent it from becoming worse.

Which personal factor places the client at risk for bladder cancer? A. Has worked in a lumber yard for 10 years B. A 50 pack-year cigarette smoking history C. Numerous episodes of bacterial cystitis D. History of gonorrhea

B -A 50 pack-year cigarette smoking history -The greatest risk factor for bladder cancer is a long history of tobacco use. The other factors would not necessarily contribute to the development of this specific type of cancer.

Which assessment maneuver should the nurse perform first when assessing the renal system at the same time as the abdomen?

B -Abdominal auscultation -Auscultation precedes percussion and palpation because the nurse needs to auscultate for abdominal bruits before palpation or percussion of the abdominal and renal components of a physical assessment.

The home care nurse is about to administer intravenous medication to the client and reads in the chart that the PICC line in the client's left arm has been in place for 4 weeks. The IV is patent, with a good blood return. The site is clean and free from manifestations of infiltration, irritation, and infection. What is the nurse's best action? a. Notify the physician. b. Administer the prescribed medication. c. Discontinue the PICC line. d. Switch the medication to the oral route.

B -Administers the prescribed medication -A PICC line that is functioning well without inflammation or infection may remain in place for months or even years. Because the line shows no signs of complications, it is permissible to administer the IV antibiotic. The physician does not have to be called to have the IV route changed to an oral route.

The client with a renal calculus has just returned from an extracorporeal shock wave lithotripsy procedure and the nurse finds an ecchymotic area on the client's right lower back. Which is the nurse's priority intervention? A. Notifying the physician B. Applying ice to the site C. Placing the client in the prone position D. Documenting the observation as the only action

B -Applying ice to the site -The shock waves can cause bleeding into the tissues through which the waves pass. Application of ice can reduce the extent and discomfort of the bruising.

Which intervention in a client with dehydration-induced confusion is most likely to relieve the confusion? a. Measuring intake and output every four hours b. Applying oxygen by mask or nasal cannula c. Increasing the IV flow rate to 250 mL/hr d. Placing the client in a high Fowler's position

B -Applying oxygen by mask or nasal cannula -Dehydration most frequently leads to poor cerebral perfusion and cerebral hypoxia, causing confusion. Applying oxygen can reduce confusion, even if perfusion is still less than optimum. Increasing the IV flow rate would increase perfusion. However, depending on the degree of dehydration, rehydrating the person too rapidly with IV fluids can lead to cerebral edema.

The nurse observes that the hand grip of the client with hypophosphatemia has diminished in strength since the last assessment 2 hours ago. What is the nurse's primary intervention? a. Document the finding and continue to as-sess. b. Assess respiratory status immediately. c. Request an order for a serum calcium lev-el. d. Administer a rapid bolus of intravenous phosphorus.

B -Assesses respiratory status immediately -Decreased hand grip strength indicates worsening of the hypophosphatemia and general muscle weakness. Muscle weakness can impair respiratory effort and reduce gas exchange to the point that the client becomes hypoxemic. The other interventions are less important.

Which intervention is most important for the nurse to teach the client who is going home but remains at risk for the development of hyperkalemia? a. Weighing self daily at the same time of day b. Assessing radial pulse for a full minute twice a day c. Ensuring an oral intake of a least 3 L of fluids per day d. Restricting sodium as well as potassium intake

B -Assessing radial pulse for a full minute twice each day -As potassium levels rise, dysrhythmias can develop. By being vigilant for changes in pulse rate, rhythm, and quality, the client can seek medical attention before hyperkalemia becomes severe. Taking a daily weight will help determine fluid retention, but this is not an accurate indicator of potassium increase or decrease. Fluid intake should be based on body weight. Sodium restriction may not be necessary

When taking the blood pressure of a very ill client, the nurse observes that the client's hand undergoes flexion contractions. What is the nurse's primary intervention?

B -Deflating the blood pressure cuff and administer oxygen -Hypocalcemia destabilizes excitable membranes and can lead to muscle twitches, spasms, and tetany. This effect of hypocalcemia is enhanced in the presence of tissue hypoxia. The flexion contractions occurring during blood pressure measurement are indicative of hypocalcemia and are referred to as a positive Trousseau's sign. The nurse will first treat the hypoxia.

4. Which of the following conditions are associated with oversecretion of rennin? a. Alzheimer's disease b. Hypertension c. Diabetes mellitus d. Diabetes insipidus

B -Hypertension -Renin is secreted when special cells in the distal convoluted tubule (DCT), called the macula densa, sense changes in blood volume and pressure. When the macula densa cells sense that blood volume is low, blood pressure is low, or blood sodium levels are low, renin is secreted. Renin then converts angiotensinogen into angiotensin I. This leads to a series of reactions that cause the secretion of the hormone aldosterone. This hormone increases kidney reabsorption of sodium and water, increasing blood pressure, blood volume, and blood sodium levels. Inappropriate or excessive renin secretion is a major cause of persistent hypertension.

Which client will the nurse assess first for the development of hypertonic dehydration?

B -The middle-aged man with diabetic ketoacidosis -Hypertonic dehydration occurs when water loss from the extracellular fluid (ECF) is greater than a proportionate electrolyte loss. The remaining ECF is hypertonic, causing fluid to move from the intracellular space to maintain circulating volume. Thus, the symptoms of hypovolemia are not present. Ketoacidosis contributes to excess water loss through the greatly increased respiratory rate and very little electrolyte is lost. In someone who is hemorrhaging, whole blood with fluid and electrolytes is lost, causing isotonic dehydration. The fluid balance problem with heart failure is overhydration. Malnutrition causes the body fluid to be hypotonic from decreased protein and sodium levels.

The client is beginning to undergo urinary bladder training. Which is an effective instruction to give this client? A. "Use the toilet at the first urge, rather than at specific intervals." B. "Try to consciously hold your urine until the scheduled toileting time." C. "Initially try to use the toilet at least every half- hour for 24 hours." D. "The toileting interval can be increased once you have been continent

B-"Try to consciously hold your urine until the scheduled toileting time." -The client should try to hold the urine consciously until the next scheduled toileting time. Toileting should occur at specific intervals during the training. The toileting interval should be no less than every hour. The interval can be increased once the client has been continent for 3 days

Which statement made by the client during nutritional counseling indicates to the nurse that the client with diabetes type 1 correctly understands his or her nutritional needs? A. "If I completely eliminate carbohydrates from my diet, I will not need to take insulin." B. "I will make certain that I eat at least 130 g of carbohydrate each day regardless of my activity level." C. "My intake of protein in terms of grams and calories should be the same as my intake of carbohydrate." D. "My intake of unsaturated fats in terms of grams and calories should be the same as my intake of protein."

B. "I will make certain that I eat at least 130 g of carbohydrate each day regardless of my activity level." Rationale: Carbohydrates are the main fuel for the human cellular engine and the substance most commonly used to make ATP. Clients who have diabetes should never consume less than 130 g of carbohydrate per day (the percentage of total calories needed is determined for each client) . Protein intake should range between 15% and 30% of total caloric intake per day.

The nurse is carign for a client with AIDS who has just been diagnosed with cryptococcal meningitis. Which is the best nursing intervention for this client? a.Initiate respiratory isolation for the next 72 hours. b.Initiate seizure precautions with padded siderails. c.Thicken the client's liquids to honey consistency. d.Administer IV pentamidine isethionate (Pentam)

B. Initiate seizure precautions with padded siderails.

A client verbalizes a fear of contracting HIV because she has a history of intravenous substance abuse. What instructions does the nurse provide to the client to help minimize this risk? a."Boil all needles and syringes for at least 20 minutes before using them again and be sure not to share them." b."Rinse used needles and syringes with water followed by laundry bleach after using them." c."Rinse used needles and syringes with rubbing alcohol before and after using them." d."Run all needles and syringes through the dishwasher with an extra rinse cycle before using them again.

B. Rinse used needles and syringes with water followed by laundry bleach after using them.

The nurse is caring for a newly diagnosed HIV-positive client who will be taking enfuvirtide (Fuzeon). Which precaution is important for the nurse to communicate to this client? a."Stop taking the medication if you develop a fever." b."Rotate the sites where you will be giving the injections." c."Take this medication with a snack or a small meal." d."Do not drive or operate machinery while taking this drug."

B. Rotate the sites where you will be giving the injections.

As a result of the client's admission to the hospital for excessive nausea, vomiting, and a blood pressure of 80/50 mm Hg, the nurse will choose which peripheral catheter gauge to insert into the hand?

C - 20 gage -The nurse selects the access device most appropriate for the designated purpose. In this case, because a large amount of fluid will be needed as a result of excessive fluid loss, the appropriate needle is the 20-gauge catheter IV because this is the most commonly used size in adults and can be used for all fluids. The 22- and 24-gauge catheters will have a slower rate of flow, which may not be desirable with excessive fluid losses and a low blood pressure. The 18-gauge catheter allows rapid flow of IV fluids. However, it requires a large vein and is more prone to irritation to the vein wall.

Which alteration in psychosocial functioning will alert the nurse to the possibility of hypokalemia?

C - Confusion -Hypokalemia decreases cerebral function and is manifested by lethargy, confusion, inability to perform problem-solving tasks, disorientation, and coma.

The client is going home after urography. Which instruction or precaution should the nurse teach this client? A. "Avoid direct contact with the urine for 24 hours until the radioisotope clears." B. "You are likely to experience some dribbling of urine for several weeks after this procedure." C. "Be sure to drink at least 3 L of f luids today to help eliminate the dye faster." D. "Your skin may become slightly yellow-tinged from the dye used in this procedure."

C -"Be sure to drink at least 3 L of fluids today to help eliminate the dye faster." -Dyes used in urography are potentially nephrotoxic.

Which statement made by the older adult client alerts the nurse to assess specifically for fluid and electrolyte imbalances? a. "My skin is always so dry, especially here in the Southwest." b. "I often use a glycerin suppository for constipation." c. "I don't drink liquids after 5 PM so I don't have to get up at night." d. "In addition to coffee, I drink at least one glass of water with each meal."

C -"I don't drink liquids after 5 PM so I don't have to get up at night." -Restricting fluids without a medical reason can lead to dehydration. Many older clients believe that restricting fluids will prevent incontinence and reduce the number of times that they wake up during the night. The increased osmolarity of the urine in response to reducing fluid intake increases the irritation of the bladder and sphincter, increasing the sensation of needing to urinate. The other statements do not indicate practices that could potentially lead to dehydration.

What action is most important for the nurse to teach the client who is at continued risk for mild hypernatremia? A. "Weigh yourself every morning and every night." B. "Check your pulse and rhythm every morning and every night." C. "Read the labels of all packaged foods to determine the sodium content." D. "When you prepare meals, try to bake or grill the food rather than frying it."

C -"Read the labels of all packaged foods to determine the sodium content." -Most prepackaged foods have a high sodium content. Teaching the client how to read the labels and calculate the sodium content of food can help him or her adhere to the prescribed sodium restriction and prevent hypernatremia. Daily weights and checking the pulse are methods of identifying manifestations of hypernatremia but do not prevent it. The addition of substances during cooking increases the sodium content of a meal, not the method of cooking.

During assessment of hydration status, the client tells the nurse that she usually drinks 3 quarts of liquids each day. Which question by the nurse is best? a. "Do you usually drink liquids that are hot or cold?" b. "How much salt do you add to your food?" c. "What kinds of liquids do you usually drink?" d. "Do you drink fluids with meals or between meals?"

C -"What kinds of liquids do you usually drink?" -It is just as important to determine the types of fluids ingested as the amount, because fluids vary widely in their osmolarity. In addition, some liquids, such as those that contain alcohol or caffeine, can contribute to fluid and electrolyte imbalances.

The client is receiving treatment with nitrofurantoin (Furadantin). Which is the highest priority instruction that the nurse can provide to this client regarding accurate administration of the medication?

C -"You should shake the medication well before measuring it out." -The medication is available in a suspension that must be shaken before being measured out. The medication does not have to be mixed before taking, and it will not discolor the urine. The drug is not available in granules that are dissolved.

3. With a renal threshold for glucose of 220 mg/dL, what is the expected response when a client has a blood glucose level of 400 mg/dL? a. 400 mg/dL of excreted glucose in the urine b. 220 mg/dL of excreted glucose in the urine c. 180 mg/dL of glucose is excreted in the urine d. No excreted glucose in the urine

C -180 mg/dL of glucose is excreted in the urine -Blood glucose is freely filtered at the glomerulus. Therefore, if a client has a blood sugar level of 400 mg/dL, the filtrate in the proximal convoluted tubule will have a glucose concentration of 400 mg/dL. With a renal threshold of 220 mg/ dL, a total of 220 mg/dL of the 400 mg/dL will be reabsorbed back into the systemic circulation, and the final urine will have a glucose concentration of 180 mg/dL.

A client has been diagnosed with overhydration and is confused. Which intervention does the nurse include in the client's plan of care to relieve the confusion? a. Measuring intake and output every shift b. Slowing the IV flow rate to 50 mL/hr c. Administering diuretic agents as prescribed d. Placing the client in Trendelenburg position

C -Administering diuretic agents as prescribed -Overhydration most frequently leads to poor neuronal function, causing confusion as a result of electrolyte imbalances (usually sodium dilution). Eliminating the fluid excess is the best way to reduce confusion. The other interventions would not relieve the client's confusion.

The nurse observes skin tenting on the back of the older adult client's hand. Which action by the nurse is most appropriate? a. Notify the physician. b. Examine dependent body areas. c. Assess turgor on the client's forehead. d. Document the finding and continue to monitor.

C -Assessing turgor on the client's forehead -Skin turgor cannot be accurately assessed on an older adult client's hands because of age-related loss of tissue elasticity in this area. Areas that more accurately show skin turgor status on an older client include the skin of the forehead, chest, and abdomen. These should also be assessed, rather than merely examining dependent body areas. Further assessment is needed rather than only documenting, monitoring, and notifying the physician.

To obtain a sterile urine specimen from a client with a Foley catheter, the nurse begins by applying a clamp to the drainage tubing distal to the injection port. What should the nurse do next? A. Clamp another section of the tube to create a fixed sample section for retrieval. B. Insert a 5-mL syringe into the injection port and aspirate the quantity of urine required. C. Clean the injection port cap of the catheter drainage tubing with povidone-iodine solution. D. Request assistance from the health care provider

C -Clean the injection port cap of the catheter drainage tubing with povidone-iodine solution. -It is important to clean the injection port cap of the catheter drainage tubing with an appropriate antiseptic, such as povidone-iodine solution or alcohol. This will help prevent surface contamination prior to injecting the syringe.

The client who has just had an IV started in the right cephalic vein tells the nurse that the wrist and hand below the IV site feel like "pins and needles." What is the nurse's best action? A. Document the response as the only action. B. Discontinue the IV and restart it at another site. C. Check for the presence of a strong blood return. D. Elevate the extremity so that it is above the level of the heart.

C -Discontinues the IV and restart it at another sit. -The sensation that the client has described is related to the IV needle touching the nerve or possibly transecting the nerve. This problem can lead to loss of function and the potential for permanent disability in the distal extremity. It is considered an emergency and the IV must be discontinued. Continuing just to monitor the IV site may lead to loss of function. The presence of blood return does not indicate absence of nerve damage. Elevation of the affected extremity does not ensure that the IV catheter has moved away from the nerve.

When assigning a client having intravenous therapy to a licensed practical nurse (LPN), which instruction is most important for the registered nurse (RN) to provide?

C -Hang a designated IV solution if permitted by the institution. -In many states, licensed practical nurses are limited in their scope of practice related to IV therapy. The RN can delegate and supervise selected nursing tasks, such as hanging an IV, to the LPN, as designated by the health care agency. The RN is accountable for comprehensive client care and outcomes of that care. This accountability would include the client's response to intravenous therapy, changing the central venous catheter dressing, and monitoring the client for side effects of the therapy.

What is the priority teaching intervention for the client with chronic hypophosphatemia? A. Where to find the radial pulse and what qualities to note B. How to intersperse daily activities with periods of rest C. How to select foods high in phosphorus and avoid foods with high concentrations of calcium. D. The importance of weighing himself or herself daily at the same time each day and wearing the same amount of clothing.

C -How to select foods high in phosphorus and low in calcium -Chronic hypophosphatemia can be managed with nutrition therapy. The client needs to increase her or his ingestion of phosphorus and decrease ingestion of calcium because phosphorus and calcium exist in the blood in a balanced inverse relationship.

Which is the result of stimulation of erythropoietin production in the kidney tissue? A. Increased blood f low to the kidney B. Inhibition of vitamin D and loss of bone density C. Increased bone marrow production of red blood cells D. Inhibition of the active transport of sodium, leading to hyponatremia

C -Increased bone marrow production of red blood cells -Erythropoietin is produced in the kidney and released in response to decreased oxygen tension in the renal blood supply. Erythropoietin stimulates red blood cell (RBC) production in the bone marrow.

The nurse is caring for a client with an intraosseous catheter placed in the leg 20 hours ago. Which assessment is of greatest concern? a. Length of time catheter is in place b. Poor vascular access in upper extremities c. Affected leg cool to touch d. Site of intraosseous catheter placement

C -Leg cool to touch -Compartment syndrome is a condition in which increased tissue perfusion in a confined anatomic space causes decreased blood flow to the area. A cool extremity can signal the possibility of this syndrome. All the other distractors are important. However, the possible development of a compartment syndrome is one that requires immediate intervention because the client could require amputation of the limb if the nurse does not pick up this perfusion problem.

Which client does the nurse determine is at highest risk for dehydration? a. Younger adult client on bedrest b. Older adult client receiving hypotonic IV fluid c. Younger adult client receiving hypertonic IV fluid d. Older adult client with cognitive impairment

C -Older adult client receiving blood replacement therapy with 3 units of packed cells -Blood replacement therapy involves intravenous fluid administration, which inherently increases the risk for overhydration. The fact that the fluid consists of packed red blood cells greatly increases the risk because this fluid increases the colloidal oncotic pressure of the blood, causing fluid to move from the interstitial and intracellular spaces into the plasma volume. An older adult may not have sufficient cardiac or renal reserve to manage this extra fluid

What dietary meal selection indicates the client understands how to increase dietary potassium intake? a. Toasted English muffin with butter and blueberry jam, and tea with sugar b. Two scrambled eggs, a slice of white toast, and a cup of strawberries c. Sausage, one slice of whole wheat toast, cup of raisins, and a glass of milk d. Bowl of oatmeal with brown sugar, cup of sliced peaches, and coffee

C -Sausage, one slice of whole wheat toast, 1/2 cup of raisins, and a glass of milk -Meat, dairy products, and dried fruit have high concentrations of potassium. Eggs, breads, cereals, sugar, and some fruit (berries, peaches) are low in potassium.

How is urge incontinence different from stress incontinence?

C -Stress incontinence occurs because of weak pelvic floor muscles. Urge incontinence occurs because of abnormal bladder contractions. -Clients who suffer from stress incontinence have weak pelvic floor muscles or urethral sphincters and cannot tighten their urethra sufficiently to overcome the increased detrusor pressure. Stress incontinence is common after childbirth, when the pelvic muscles are stretched and weakened from pregnancy and delivery. Urge incontinence occurs in people who cannot suppress the contraction signal from the detrusor muscle. Abnormal detrusor contractions may be a result of neurologic abnormalities or may occur with no known abnormality.

1. Confirmed by palpation and x-ray study, the client's right kidney is lower than the left kidney. What is the nurse's interpretation of this finding? a. The client has a problem involving the right kidney. b. The client has a problem involving the left kidney. c. The client has both kidneys in the normal position. d. The client is at increased risk for kidney impairment.

C -The client has both kidneys in the normal position. -Normally, the right kidney is positioned somewhat lower than the left kidney. This anatomic difference in otherwise symmetric organs is caused by liver displacement. The significance of this difference is that the right kidney is easier to palpate in an adult than is the left kidney.

Which item of assessment data obtained by the home care nurse suggests that an older adult client may be dehydrated? a. The client has dry, scaly skin on bilateral upper and lower extremities. b. The client states that he gets up three or more times during the night to urinate. c. The client states that he feels lightheaded when he gets out of bed or stands up. d. The nurse observes tenting on the back of the hand when testing skin turgor.

C -The client states that he feels lightheaded when he gets out of bed or stands up. -Orthostatic or postural hypotension can be caused by or worsened by dehydration. The other statements are not as indicative of the severe degree of dehydration as dizziness on standing.

The client newly diagnosed with type 1 diabetes asks why insulin is given only by injection and not as an oral drug. What is the nurse's best response? A. "Injected insulin works faster than oral drugs to lower blood glucose levels." B. "Oral insulin is so weak that it would require very high dosages to be effective." C. "Insulin is a small protein that is destroyed by stomach acids and intestinal enzymes." D. "Insulin is a "high alert drug" and could more easily be abused if it were available as an oral agent."

C. "Insulin is a small protein that is destroyed by stomach acids and intestinal enzymes." Rationale: Because insulin is a small protein that is easily destroyed by stomach acids and intestinal enzymes, it cannot be used as an oral drug. Most commonly, it is injected subcutaneously.

The client newly diagnosed with diabetes asks why he is always so thirsty. What is the nurse's best response? A. "The extra glucose in the blood increases the blood sodium level, which increases your sense of thirst." B. "Without insulin, glucose is excreted rather than used in the cells. The loss of glucose directly triggers thirst, especially for sugared drinks." C. "The extra glucose in the blood makes the blood thicker, which then triggers thirst so that the water you drink will dilute the blood glucose level." D. "Without insulin, glucose combines with blood cholesterol, which damages the kidneys, making you feel thirsty even when no water has been lost."

C. "The extra glucose in the blood makes the blood thicker, which then triggers thirst so that the water you drink will dilute the blood glucose level." Rationale: The movement of glucose into cells is impaired, and the resulting high blood glucose levels increase the osmolarity of the blood. This increased osmolarity stimulates the osmoreceptors in the hypothalamus, triggering the thirst reflex. In response, the person drinks more water (not sugary fluids or hyperosmotic fluids), which helps dilute blood glucose levels and reduces blood osmolarity.

A client with type 2 diabetes who also has heart failure is prescribed metformin extended-release (Glucophage XR) once daily. On assessment, the nurse finds that the client now has muscle aches, drowsiness, low blood pressure, and a slow, irregular heartbeat. What is the nurse's best action? A. Assess the client's blood glucose level and prepare to administer IV glucose. B. Reassure the client that these symptoms are normal effects of this drug. C. Hold the dose and notify the prescriber immediately. D. Administer the drug at bedtime to prevent falls.

C. Hold the dose and notify the prescriber immediately. Rationale: Muscle aches, drowsiness, low blood pressure, and a slow irregular heartbeat are symptoms of lactic acidosis, an adverse reaction to metformin. The drug should be stopped and the prescriber notified so steps can be taken to reduce the client's acidosis.

The nurse is completing a health history for a client and begins to obtain a sexual history. What is the nurse's best opening question? a."How long have you been sexually active?" b."Are you in a monogamous relationship with your spouse?" c."How do you feel about answering questions about your sexual history?" d."Have you noticed any problems with your ability to have or enjoy sex?

C. How do you feel about answering questions about your sexual history?

The nurse asks a young adult client if she is sexually active. The client asks why the nurse needs to know. That is the nurse's best response? a."I just need to make sure that the information you are providing is reliable." b."I have to fill in answers to all of the questions on the health history form." c."If you are sexually active, we should talk about ways to prevent getting HIV." d."I will have to notify your partner if you have a sexually transmitted disease."

C. If you are sexually active, we hsould talk about ways to prevent getting HIV

Five days after the start of intraperitoneal therapy, the client reports abdominal pain and "feeling warm." The nurse prepares to assess the client further for evidence of which condition? a. Allergic reaction b. Bowel obstruction c. Catheter lumen occlusion d. Infection

D - Infection -Fever, abdominal pain, abdominal rigidity, and rebound tenderness may be present in the client who has peritonitis related to intraperitoneal therapy. Peritonitis is preventable by using strict aseptic technique in the handling of all equipment and infusion supplies. An allergic reaction would occur earlier in the course of treatment. Bowel obstruction and catheter lumen occlusion can occur, but would present clinically in different ways.

What is the most important instruction that the nurse gives to the client who is prescribed oral alendronate?

D -"Do not take this medication with food or any other medication." -Food inhibits the absorption of this medication. This medication has a high potential to interact with other drugs. This medication should not alter the heart rate, cause dependency, or interact with aspirin.

Which is priority discharge teaching for a client who has undergone the removal of a renal calculus? A. "Do not expect to see any blood in your urine." B. "Expect to experience pain in the bladder area." C. "Look for bruising and report it." D. "Drink at least 3 L of fluid daily and monitor urine pH."

D -"Drink at least 3 L of fluid daily and monitor urine pH." -The client should drink at least 3 L of fluid daily and monitor his or her urine pH as directed. He or she should expect to see some blood in the urine postoperatively and some bruising, but should not experience pain. If there is pain, this might signal the development of another stone and should be reported.

The caretaker of a confused client with functional incontinence asks about having an in-dwelling catheter placed. Which is the nurse's best response? A. "You must be very aggravated about this situation. I will call the physician with this request." B. "I will teach you how to insert the catheter, which should be used just at night." C. "We can teach you how to perform intermittent catheterization to drain the bladder." D. "Pads can be worn to prevent smells and leaks. Social services can help you obtain these supplies at a reasonable cost."

D -"Pads can be worn to prevent smells and leaks. Social services can help you obtain these supplies at a reasonable cost." -In-dwelling catheters are used only as a last resort because of the risk for ascending urinary tract infections and sepsis. The use of containment pads should be attempted as a means of controlling wetness first. If the client has skin breakdown, an in-dwelling catheter can be placed temporarily until the area has healed.

The client is scheduled to undergo the surgical creation of an ileal conduit. He expresses his anxiety and fear regarding the procedure. Which is an appropriate response from the nurse?

D -"Would you like to speak with another client who has undergone this procedure?" -The goal for the client who is scheduled to undergo a procedure such as an ileal conduit is to have a positive self-image and a positive attitude about his body image. Medications for anxiety or sleep will not promote this, nor will discussing the procedure once more with his physician. However, discussing the procedure candidly with a former client will foster such feelings, especially when the current client has an opportunity to ask questions and voice concerns to someone with first-hand knowledge of the procedure.

A client has been taught to restrict dietary sodium. Which food selection by the client indicates to the nurse that teaching has been effective? a. Chinese take-out, including steamed rice b. A grilled cheese sandwich with tomato soup c. Slices of ham and cheese on whole grain crackers d. A chicken leg, one slice of bread with butter, and steamed carrots

D -A chicken leg, one slice of whole wheat bread with butter, and 1/2 cup of steamed carrots -The total sodium content of the chicken leg, bread, and carrots is 370 mg. The sodium content of the next lowest sodium-containing meal is 550 mg. The other meal selections are also too high in sodium content for a restricted sodium diet.

Which client is at highest risk for developing a renal calculus? A. An older man with diabetes mellitus B. A young woman who is 6 months pregnant C. A middle-aged woman with mild congestive heart failure D. A young man who had a renal calculus 1 year ago

D -A young man who had a renal calculus 1 year ago -Age and the other conditions listed do not contribute to the formation of renal calculi. The greatest risk factor for calculus formation is a history of a previous stone.

A nurse observes that the client's left flank region is larger than the right flank region. What is the nurse's best action? a. Ask the client if he or she participates in contact sports and has been recently injured. b. Document the finding as the only action on the appropriate flowsheet. c. Apply a heating pad to the left flank after inspecting the site for signs of infection. d. Anticipate further diagnostic testing after sharing informing the physician of this finding.

D -Anticipating further diagnostic testing after informing the physician of this finding -Asymmetry of the flank or a unilateral protrusion may indicate an enlargement of a kidney. The enlargement may be benign or may be associated with a hydronephrosis or mass on the kidney.

Which client is the best candidate to receive hypodermoclysis for IV therapy? a. Client requiring 4000 mL normal saline in 24 hours b. Client with an extensive burn injury c. Client with allergy to hyaluronidase d. Client receiving pain management

D -Client receiving pain management -Subcutaneous therapy (hypodermoclysis) involves the slow infusion of isotonic fluids into the client's subcutaneous tissue. Most often, it is used in hospices for pain management. It should not be used if the fluid replacement needs exceed 3000 mL/day. To be used, the client must have sufficient areas of intact skin. Hyaluronidase is frequently used to help absorb the fluid during the therapy.

Which IV order would the nurse question?

D -Infuse 0.9% normal saline at keep vein open (KVO) rate -To be complete, IV orders for infusion fluids should specify the rate of infusion. This order does not specify the rate of infusion and is not considered complete.

Which client is at greatest risk for dehydration? a. Younger adult client on bedrest b. Older adult client receiving hypotonic IV fluid c. Younger adult client receiving hypertonic IV fluid d. Older adult client with cognitive impairment

D -The older adult client with cognitive impairment -Older adults, because of having less total body water than younger adults, are at greater risk for development of dehydration. Anyone who is cognitively impaired, and either cannot obtain fluids independently or cannot make his or her need for fluids known, is at high risk for dehydration.

Which is an initial priority intervention for the client with stress incontinence? A. Beginning medication teaching B. Having the client sign an informed consent form for surgery C. Assisting the client in finding a supplier of absorbent pads and undergarments D. Instructing the client to maintain a diary that records times of urine leakage, activities, and diet

D -Instructing the client to maintain a diary that records times of urine leakage, activities, and diet -Maintaining a diary detailing times of urine leakage, activities, and foods eaten will aid in the diagnostic process by showing if there is a connection between specific factors that seem to trigger the incontinent episodes. Use of medication, surgical procedures, and absorbent pads or undergarments may be used as part of the physician's treatment plan at some point, but more conservation interventions should be implemented first.

The client reports the regular use of all the following medications. Which one alerts the nurse to the possibility of renal impairment when used consistently? A. Antacids B. Penicillin C. Antihistamine nasal sprays D. Nonsteroidal anti-inflammatory drugs (NSAIDs)

D -Nonsteroidal anti-inflammatory drugs (NSAIDs) -NSAIDs inhibit prostaglandin production and decrease blood flow to the nephrons. They can cause an interstitial nephritis and renal impairmen

The nurse is caring for four clients receiving intravenous therapy. Which client should the nurse assess first? a. Client with a newly inserted peripherally inserted central catheter (PICC) line waiting for x-ray b. Client with a peripheral catheter for intermittent infusions c. Older adult client with a nonaccessed implanted port d. Older adult client with normal saline infusion

D -Older adult client with normal saline infusing -Older adults are more prone to fluid overload and resulting congestive heart failure. Because this client is receiving continuous IV fluid, he or she is at risk for fluid overload and needs to be assessed. All the other clients would need to be assessed for complications of IV catheters. However, they do not need immediate assessment.

The nurse is caring for a client with a radial arterial catheter. Which assessment is of most concern? a. Amount of pressure in fluid container b. Date of catheter tubing change c. Checking for heparin in infusion container d. Presence of an ulnar pulse

D -Presence of an ulnar pulse -An intra-arterial catheter can cause arterial occlusion, which can lead to absent or decreased perfusion to the extremity. Assessing the ulnar pulse could be one way to assess circulation to the arm in which the catheter is located. The nurse would note that there is enough pressure in the fluid container to keep the system flushed, and would also check to see if the catheter tubing needs to be changed. However these are not assessments of most concern. Because of heparin-induced thrombocytopenia, heparin is not used in most institutions for an arterial catheter.

A nurse is caring for several clients with dehydration. The nurse assesses the client with which finding as needing oxygen therapy? a. Tenting of skin on the back of the hand b. Increased urine osmolarity c. Weight loss of 10 pounds d. Pulse rate of 115 beats/min

D -Pulse rate of 115 beats/min -Severe dehydration can decrease circulating volume and decrease cardiac output, placing vital organs at risk for hypoxia, anoxia, and ischemia. Whenever cardiac output is decreased with dehydration, oxygen therapy is indicated.

In examining a peripheral IV site, the nurse observes a red streak along the length of the vein and the vein feels hard and cordlike. What is the nurse's next action?

D -Removing the catheter -The clinical manifestations described are those associated with phlebitis. Phlebitis is an inflammation of the vein. Its presence in a vein being used for IV therapy may be caused by mechanical forces associated with the IV device or by chemical factors related to the composition and osmolarity of the drug solution. The key manifestation is that the symptoms are directly associated with the vein and the catheter must be removed. Warm compresses can be applied for 20 minutes four times daily after the catheter is removed. The site needs to be monitored after the catheter is removed. The arm is not swollen. Therefore, elevation of the extremity is not a correct option.

Prior to the administration of intravenous fluid, it is most important for the nurse to obtain which information from the health care provider's orders? a. Intravenous catheter size b. Osmolarity of the solution c. Vein to be used for therapy d. Specific type of IV fluid

D -Specific type of intravenous fluid -An order for infusion therapy must contain the following to be complete: specific type of fluid, rate of administration, and drugs added to the solution. Osmolarity of the solution is not necessary because it is incorporated into the specific type of fluid. It is the nurse's independent decision about the most appropriate vein to cannulate and the catheter size to use.

The client is receiving an intravenous infusion of 60 mEq of potassium chloride in a 1000-mL solution of dextrose 5% in 0.45% saline. The client states that the area around the IV site burns. What intervention will the nurse perform first? a. Notify the physician. b. Assess for a blood return. c. Document the finding. d. Stop the IV infusion

D -Stops the IV fluid containing potassium -Potassium is a severe tissue irritant. The safest action is to discontinue the solution that contains the potassium. The nurse could run another solution as assessment continues. Assessing for a blood return may or may not be successful. The solution could be diluted (less potassium) and the rate could be slowed once it is determined that the needle is in the vein.

When assessing a client's peripheral IV site, the nurse notices edema and tenderness above the site. What action will the nurse take first? a. Apply cold compresses to the IV site. b. Elevate the extremity on a pillow. c. Flush the catheter. d. Stop the infusion of IV fluids

D -Stops the infusion of IV fluids -Infiltration occurs when the needle dislodges partially or completely from the vein. Signs of infiltration include edema and tenderness above the site. The nurse should stop the infusion and remove the catheter. Cold compresses and elevation of the extremity can be done after the catheter is discontinued to increase client comfort.

Which client is at greatest risk for developing hypercalcemia?

D -The client with hyperparathyroidism -The parathyroid glands secrete parathyroid hormone. The actions of parathyroid hormone include increasing intestinal absorption of calcium, decreasing renal excretion of calcium, and increasing calcium resorption from the bones. All these actions increase the serum calcium level.

Which assessment finding for a client with a PICC line requires immediate attention?

D -Upper extremity swelling -Upper extremity swelling could indicate infiltration, and the PICC line will need to be removed. The initial dressing over the PICC site should be changed within 24 hours. However, this does not require immediate attention but the swelling does. The dwell time for PICC lines can be for months or even years. Securement devices are being used more now to secure the catheter in place and prevent complications such as phlebitis and infiltration.

Which change in renal or urinary functioning as a result of the normal aging process increases the older client's risk for infection? a. Decreased glomerular filtration b. Decreased filtrate reabsorption c. Weakened sphincter muscles d. Urinary retention

D -Urinary retention -Incomplete bladder emptying for whatever reason increases the client's risk for urinary tract infections. This is a result of urine stasis, which provides an excellent culture medium that promotes the growth of microorganisms.

The client newly diagnosed with type 2 diabetes asks how diabetes type 1 and diabetes type 2 are different. What is the nurse's best response? A. "Diabetes type 1 develops in people younger than 40 years and diabetes type 2 develops only in older people." B. "Diabetes type 2 develops in people younger than 40 years and diabetes type 1 develops only in older people." C. "Patients with type 1 diabetes are at higher risk for obesity and heart disease, whereas patients with type 2 diabetes are at higher risk for strokes." D. "Patients with type 1 diabetes produce no insulin and patients with type 2 diabetes produce insulin but their insulin receptors are not very sensitive to it."

D. "Patients with type 1 diabetes produce no insulin and patients with type 2 diabetes produce insulin but their insulin receptors are not very sensitive to it." Rationale: The main problem with type 1 diabetes is that the person can no longer make insulin. Without insulin, the client's blood glucose level becomes very high, but glucose cannot enter many cells. Clients with type 1 diabetes must use insulin daily for the rest of their lives or receive a pancreas transplant. With type 2 diabetes, the person still has beta cells that make some insulin. In fact, some people with type 2 diabetes have normal levels of insulin; however, the insulin receptors are not very sensitive to it. As a result, insulin does not bind as tightly to its receptors as it should, and less glucose moves from the blood into the cells.

A client with a history of rheumatoid arthritis will be starting drug therapy with etanercept (Enbrel). What is most important for the nurse to teach the client

The correct technique for subcutaneous injections

The nurse is caring for a pregnant client who is taking umbra (adalimumab) to control symptoms of rheumatoid arthritis. The client mentions the pain and inconvenience of the subcutaneous injections and asks, "While I'm pregnant, can I take this drug by mouth instead?" What is the nurse's best response?

Humira can be given only by subcutaneous injection.

The nurse provides discharge teaching for a client to prevent a new attack of gout. Which statement by the client indicates that additional teaching is required?

I hate to start limiting my fluid intake so much!

The nurse is teaching a client who has osteoarthritis ways to slow progression of the disease. Which statement indicates that the client understands the nurse's instruction?

I will start swimming twice a week.

The nurse is working with a client who has severe rheumatoid arthritis in their hands. The client states that she is frustrated at mealtime because it is difficult for her to manage cups and silverware. What is the nurse's best response?

Let's see if the occupational therapist can provide you with some utensils that are easier for you to use.

The nurse is caring for a client who has a history of severe rheumatoid arthritis. The client becomes combative and abusive to the staff when she is unable to perform personal care independently. What is the best statement the nurse can make to the client at this time

You seem frustrated. Would you like to try to dress again in a few minutes?

nurse reviews electrolyte values and notes sodium level of 130. the nurse understands that this sodium level would be notes in a client with which condition

_SIADH

a client has the greatest need for K replacement

a client with cardiac disease who is about to receive furosemide with a serum level of 3.5

A client with diabetes is prescribed insulin glargine once daily and regular insulin four times daily. One dose of regular insulin is scheduled at the same time as the glargine. How does the nurse instruct the client to administer the two doses of insulin? a. "Draw up and inject the insulin glargine first, then draw up and inject the regular insulin." b. "Draw up and inject the insulin glargine first, wait 20 minutes, then draw up and inject the regular insulin." c. "First draw up the dose of regular insulin, then draw up the dose of insulin glargine in the same syringe, mix, and inject the two insulins together." d. "First draw up the dose of insulin glargine, then draw up the dose of regular insulin in the same syringe, mix, and inject the two insulins together."

a. "Draw up and inject the insulin glargine first, then draw up and inject the regular insulin." Rationale: Insulin glargine must not be diluted or mixed with any other insulin or solution. Mixing results in an unpredictable alteration in the onset of action and time to peak action. The correct instruction is to draw up and inject first the glargine, then the regular insulin right afterward.

The home health nurse visits a client with a diagnosis of Type 1 DM. The client relates a history of vomiting & diarrhea and tells the nurse that no food has been consumed for the last 24hrs. Which add'l stmt by the client indicates a need for further teaching? a. "I need to stop my insulin" b. "I need to increase my fluid intake" c. "I need to monitor my blood glucose every 3 to 4 hours" d. "I need to call the HCP because of the symptoms"

a. "I need to stop my insulin" Rationale: When a client is unable to eat because of illness they should still take their prescribed insulin or oral meds.

Which statement made by a client getting ready for discharge after pancreas transplantation indicates a need for further teaching about the prescribed drug regimen? a. "If I develop an infection, I should stop taking my corticosteroid." b. "If I have pain over the transplant, I will call the surgeon immediately." c. "I should avoid people who are ill or who have an infection." d. "I should take my cyclosporine exactly the way I was taught."

a. "If I develop an infection, I should stop taking my corticosteroid." Rationale: Immune suppressive agents should not be stopped without the consultation of the transplantation physician, even if an infection is present. Stopping immune suppressive therapy endangers the transplanted organ. The other statements are correct. Pain over the graft site may indicate rejection. Anti-rejection drugs cause immune suppression, and the client should avoid crowds and people who are ill. Changing the routine of anti-rejection medications may cause them to not work optimally.

A client has been taught to inject insulin. Which statement made by the client indicates a need for further teaching? a. "The abdominal site is best because it is closest to the pancreas." b. "I can reach my thigh the best, so I will use different areas of the same thigh." c. "By rotating the sites in one area, my chance of having a reaction is decreased." d. "Changing injection sites from the thigh to the arm will change absorption rates."

a. "The abdominal site is best because it is closest to the pancreas." Rationale: The abdominal site has the fastest rate of absorption because of blood vessels in the area, not because of its proximity to the pancreas. The other statements are accurate assessments of insulin administration.

A client will be undergoing palliative surgery. The client's daughter asks what this means. What is the nurse's best response? a. "The surgery will relieve the symptoms but will not cure your father." b. "There are fewer risks with this type of surgery." c. "There is no guarantee of the outcome of the surgery." d. "The surgery must be performed immediately to save your father's life."

a. "The surgery will relieve the symptoms but will not cure your father."

A client with diabetes is visually impaired and wants to know whether syringes can be prefilled and stored for later use. Which is the nurse's best response? a. "Yes. Prefilled syringes can be stored for 3 weeks in the refrigerator in a vertical position with the needle pointing up." b. "Yes. Prefilled syringes can be stored for up to 3 weeks in the refrigerator, placed in a horizontal position." c. "Insulin reacts with plastic, so prefilled syringes are okay, but they must be made of glass." d. "No. Insulin cannot be stored for any length of time outside of the container."

a. "Yes. Prefilled syringes can be stored for 3 weeks in the refrigerator in a vertical position with the needle pointing up." Rationale: Insulin is relatively stable when stored in a cool, dry place away from light. When refrigerated, prefilled syringes are stable for up to 3 weeks. They should be stored in the vertical position with the needle pointing up to prevent suspended insulin particles from clogging the needle. The other answers are inaccurate.

A client's father has type 1 diabetes mellitus. The client asks if she is in danger of developing the disease as well. Which is the nurse's best response? a. "Your risk of diabetes is higher than that of the general population, but it may not occur." b. "No genetic risk is associated with the development of type 1 diabetes." c. "The risk for becoming diabetic is 50% because of how it is inherited." d. "Female children do not inherit diabetes, but male children will."

a. "Your risk of diabetes is higher than that of the general population, but it may not occur." Rationale: Risk for type 1 diabetes is determined by inheritance of genes coding for HLA-DR and HLA-DQ tissue types. Clients who have one parent with type 1 diabetes are at increased risk for its development. Diabetes (type 1) seems to require interaction between inherited risk and environmental factors, so not everyone with these genes develops diabetes. The other statements are not accurate.

The nurse is performing health screening in a local mall. Which people does the nurse counsel to be tested for diabetes? (Select all that apply.) a. African-American or American Indian b. Person with history of pancreatic trauma c. Woman with a 30-pound weight gain during pregnancy d. Male with a body mass index greater than 25 kg/m2 e. Middle-aged woman with physical inactivity most days of the week f. Young woman who gave birth to a baby weighing more than 9 pounds

a. African-American or American Indian d. Male with a body mass index greater than 25 kg/m2 e. Middle-aged woman with physical inactivity most days of the week f. Young woman who gave birth to a baby weighing more than 9 pounds Rationale: Risk factors for type 2 diabetes include certain ethnic/racial groups (African Americans, American Indians, Hispanics), obesity and physical inactivity, and giving birth to large babies. Pancreatic trauma and a 30-pound gestational weight gain are not risk factors.

The nurse reviews a client's laboratory results before surgery and notes a fasting blood glucose of 120 mg/dL, a prothrombin time (PT) of 25 seconds, and potassium (K+) of 3.8 mEq/L. Which action by the nurse is best? a. Ask the surgeon for additional laboratory studies. b. Administer a potassium supplement of 20 mEq. c. Increase the IV infusion of D5W to 100 mL/hr. d. Record laboratory results on the preoperative assessment.

a. Ask the surgeon for additional laboratory studies.

A client with diabetes has frequent blood glucose readings higher than 300 mg/dL. Which action does the nurse teach the client about self-care? a. Check urine ketones when blood glucose readings are high. b. Increase the insulin dose after two high glucose readings in a row. c. Change the diet to include a 10% increase in protein. d. Work out on the treadmill whenever glucose readings are high.

a. Check urine ketones when blood glucose readings are high. Rationale: Urine should be tested for ketone bodies whenever the client has a blood glucose higher than 300 mg/dL; is acutely ill, under stress, pregnant, or participating in a weight reduction program; or has symptoms of ketoacidosis (nausea, vomiting, and abdominal pain). The client should not change diet and insulin dosages without input from the health care provider. The client should not exercise when blood glucose is higher than 250 mg/dL.

A client has newly diagnosed diabetes. To delay the onset of microvascular and macrovascular complications in this client, the nurse stresses that the client take which action? a. Control hyperglycemia. b. Prevent hypoglycemia. c. Restrict fluid intake. d. Prevent ketosis.

a. Control hyperglycemia Rationale: Hyperglycemia is a critical factor in the pathogenesis of long-term diabetic complications. Maintaining tight glycemic control will help delay the onset of complications. Preventing hypoglycemia and ketosis, although important, is not as important as maintaining daily glycemic control. Restricting fluid intake is not part of the treatment plan for clients with diabetes.

A client with type 1 diabetes has a blood glucose level of 160 mg/dL on arrival at the operating room. Which is the nurse's best action? a. Document the finding in the client's chart. b. Administer a bolus of regular insulin IV. c. Call the physician to cancel the operation. d. Draw blood gases to assess the metabolic state.

a. Document the finding in the client's chart. Rationale: Clients who have type 1 diabetes and are having surgery have been found to have fewer complications, lower rates of infection, and better wound healing if blood glucose levels are maintained at between 140 and 180 mg/dL throughout the perioperative period. The nurse should document the finding and proceed with other operative care. The need for a bolus of insulin, for canceling the operation, or for drawing arterial blood gases (ABGs) is not present.

A client is brought to the emergency department (ED) after a motorcycle accident. The client has suffered a ruptured spleen. What is the immediate priority? a. Emergent surgery to control bleeding b. Aggressive pain control c. Calling the family members d. Assessment of neurologic status

a. Emergent surgery to control bleeding

Which action is most appropriate during a preoperative chart review? a. Ensure that the consent form is signed, dated, and witnessed. b. Call the surgeon if the client has any food allergies. c. Make sure all marks are washed off the surgical site. d. Make sure the client understands the procedure.

a. Ensure that the consent form is signed, dated, and witnessed.

To reduce complications of diabetes, the nurse teaches a client with normal kidney function to modify intake of which nutritional group? a. Fats b. Fiber c. Proteins d. Carbohydrates

a. Fats Rationale: Diabetes causes abnormalities in fat metabolism that lead to hyperlipidemia. The high lipid levels can lead to atherosclerosis and to many pathologic consequences of vascular insufficiency. Specific fat recommendations can be made by the registered dietitian according to individual client factors, but reducing fat intake is healthy for all diabetic people. The client with renal insufficiency may need to limit protein. Fiber should be increased do 25 to 30 g/day, and intake of carbohydrates must be spread out throughout the day.

The home care nurse finds a client who has diabetes awake and alert, but shaky, diaphoretic, and weak. The nurse gives the client cup of orange juice. The client's clinical manifestations have not changed 5 minutes later. Which is the nurse's best next action? a. Give the client another cup of orange juice. b. Call the rescue squad for transportation to the hospital. c. Administer 10 units of regular insulin subcutaneously. d. Administer 1 mg glucagon intramuscularly.

a. Give the client another cup of orange juice. Rationale: This client is experiencing mild hypoglycemia. For mild hypoglycemic manifestations, if the symptoms do not resolve immediately, repeat the treatment. The client does not need glucagon, transportation to the hospital, or insulin.

The nurse is caring for an older adult client with a history of chronic lung disease who will be undergoing surgery the following day. When postoperative care is planned, which potential problem is the highest priority for this client? a. Maintaining oxygenation b. Tolerating activity c. Anxiety and fear d. Hypovolemia

a. Maintaining oxygenation

A client who has been taking pioglitazone (Actos) for 6 months reports to the nurse that his urine has become darker since starting the medication. Which is the nurse's first action? a. Review results of liver enzyme studies. b. Document the report in the client's chart. c. Instruct the client to increase water intake. d. Test a sample of urine for occult blood.

a. Review results of liver enzyme studies. Rationale: Thiazolidinediones (including pioglitazone) can affect liver function; liver function should be assessed at the start of therapy and at regular intervals while the client continues to take these drugs. Dark urine is one indicator of liver impairment because bilirubin is increased in the blood and is excreted in the urine. The nurse should check the client's most recent liver function studies. Documentation should be done after all assessments have been completed. The client does not need to be told to increase water intake, and the nurse does not need to check the urine for occult blood.

The nurse is monitoring a client who was diagnosed with Type 1 DM and is being treated with NPH and regular insulin. Which client complaint(s) would alert the nurse to the presence of a possible hypoglycemic reaction? Select all that apply: a. Tremors b. Anorexia c. Irritability d. Nervousness e. Hot, dry skin f. Muscle cramps

a. Tremors c. Irritability d. Nervousness Rationale: Decreased BG levels produce Autonomic Nervous System symptoms. Classic symptoms are Nervousness, irritability, and tremors.

The nurse administers 6 units of regular insulin and 10 units NPH insulin at 7 AM. At what time does the nurse assess the client for problems related to the NPH insulin? a. 8 AM b. 4 PM c. 8 PM d. 11 PM

b. 4 PM NPH is an intermediate-acting insulin with an onset of 1.5 hours, peak of 4 to 12 hours, and duration of action of 22 hours. Checking the client at 8:00 AM would be too soon; 8:00 PM and 11:00 PM would be too late.

A client with diabetes asks why more than one injection of insulin is required each day. Which is the nurse's best response? a. "You need to start with multiple injections until you become more proficient at self-injection." b. "A single dose of insulin each day would not match your blood insulin levels and your food intake patterns closely enough." c. "A regimen of a single dose of insulin injected each day would require that you could eat no more than one meal each day." d. "A single dose of insulin would be too large to be absorbed predictably, so you would be in danger of unexpected insulin shock."

b. "A single dose of insulin each day would not match your blood insulin levels and your food intake patterns closely enough." Rationale: Even when a single injection of insulin contains a combined dose of different-acting insulins, the timing of the actions and the timing of food intake may not match well enough to prevent wide variations in blood glucose levels.

A client who has type 2 diabetes is prescribed glipizide (Glucotrol). Which precautions does the nurse include in the teaching plan related to this medication? a. "Change positions slowly when you get up." b. "Avoid taking nonsteroidal anti-inflammatory drugs." c. "If you miss a dose of this drug, you can double the next dose." d. "Discontinue the medication if you develop an infection."

b. "Avoid taking nonsteroidal anti-inflammatory drugs." Rationale: Nonsteroidal anti-inflammatory drugs potentiate the hypoglycemic effects of sulfonylurea agents. Glipizide is a sulfonylurea. The other statements are not applicable to glipizide.

Which statement by a client with type 2 diabetes indicates a need for further teaching about diabetic management and follow-up care? a. "I need to have an annual appointment, even if my glucose levels are in good control." b. "Because my diabetes is controlled with diet and exercise, I have to be seen only if I am sick." c. "I can still develop complications, even though I do not have to take insulin at this time." d. "If I have surgery or get very ill, I may have to receive insulin injections for a short time."

b. "Because my diabetes is controlled with diet and exercise, I have to be seen only if I am sick." Rationale: Clients with diabetes need to be seen at least annually to monitor for long-term complications, including visual changes, microalbuminuria, and lipid analysis. The other statements are correct.

A client newly diagnosed with type 2 diabetes tells the nurse that since increasing fiber intake, he is having loose stools, flatulence, and abdominal cramping. Which is the nurse's best response? a. "Decrease your intake of water and other fluids until your stools firm up." b. "Decrease your intake of fiber now and gradually add it back into your diet." c. "You must have allergies to high-fiber foods and will need to avoid them." d. "Taking an antacid 1 hour before or 2 hours after meals will help this problem."

b. "Decrease your intake of fiber now and gradually add it back into your diet." Rationale: Many people experience these side effects when first increasing dietary fiber. Gradually incorporating high-fiber foods into the diet can minimize abdominal cramping, discomfort, loose stools, and flatulence. The client needs increased water intake with fiber. The client does not have allergies, nor should he or she take antacids in the hope that they will reduce the problem.

The nurse is teaching a client about self-monitoring of blood glucose levels. To prevent bloodborne infection, which statement by the nurse is best? a. "Wash your hands after completing the test." b. "Do not share your monitoring equipment." c. "Blot excess blood from the strip." d. "Use gloves during monitoring."

b. "Do not share your monitoring equipment." Rationale: Small particles of blood can adhere to the monitoring device, and infection can be transported from one user to another. Hepatitis B in particular can survive in a dried state for about a week. The client should be taught to avoid sharing any equipment, including the lancet holder. The client should be taught to wash hands before testing. The client would not need to blot excess blood away from the strip or to wear gloves.

The client with type 2 diabetes has recently been changed from the oral antidiabetic agents glyburide (Micronase) and metformin (Glucophage) to glyburide-metformin (Glucovance). The nurse includes which information in the teaching about this medication? a. "Glucovance is more effective than glyburide and metformin." b. "Glucovance contains a combination of glyburide and metformin." c. "Glucovance is a new oral insulin and replaces all other oral antidiabetic agents." d. "Your diabetes is improving and you now need only one drug."

b. "Glucovance contains a combination of glyburide and metformin." Rationale: Glucovance is composed of glyburide and metformin. It is given to enhance the convenience of antidiabetic therapy with glyburide and metformin. The other statements are not accurate.

A client has been taught about lifestyle changes to help manage newly diagnosed diabetes mellitus type 2. Which statement by the client indicates good understanding? a. "Weight gain may lead to type 1 diabetes and I would need insulin." b. "I may not need to take medications if my weight is maintained." c. "I do not have to check my blood glucose if my weight is in the proper range." d. "My vision and foot pain may go away if I lose some weight."

b. "I may not need to take medications if my weight is maintained." Rationale: Type 2 diabetes can be prevented or delayed by weight loss and increased physical activity. Encourage all clients to maintain weight within an appropriate range for height and body build. Once diagnosed with type 2 diabetes, blood glucose monitoring is indicated, regardless of whether the client is taking oral antidiabetic medications. Vision and neurologic changes will not go away with weight control.

The nurse has just completed preoperative teaching with a client who will be having surgery the following day. Which statement by the client indicates that additional teaching is needed? a. "When I brush my teeth before surgery, I will be sure to spit out the water." b. "I will go to the bathroom as soon as I receive all my preoperative medications." c. "I will remember to wear my glasses tomorrow instead of my contact lenses." d. "I won't have to worry about putting my makeup on tomorrow morning."

b. "I will go to the bathroom as soon as I receive all my preoperative medications."

Which statement made by a client with type 2 diabetes taking nateglinide (Starlix) indicates understanding of this therapy? a. "I'll take this medicine with my meals." b. "I'll take this medicine right before I eat." c. "I'll take this medicine just before I go to bed." d. "I'll take this medicine when I wake up in the morning."

b. "I'll take this medicine right before I eat." Rationale: Nateglinide is an insulin secretagogue that is designed to increase meal-related insulin secretion. It should be taken just before a meal. The other options are incorrect.

The nurse is teaching a client with diabetes about exercise. Which statement by the client indicates a need for further teaching? a. "I won't exercise if I find ketones in my urine." b. "If my blood glucose is over 200, I should not exercise." c. "Exercise will help me keep my blood glucose down." d. "My risks for heart disease can be modified with exercise."

b. "If my blood glucose is over 200, I should not exercise." Rationale: Clients should not exercise if their blood glucose is over 250 mg/dL. The other statements are correct and show good understanding.

A client with a 20-year history of diabetes mellitus is reviewing his medications with the nurse. The client holds up the bottle of duloxetine (Cymbalta) and states, "My cousin has depression and is on this drug. Do you think I'm depressed?" What is the nurse's best response? a. "Many people with long-term diabetes become depressed after a while." b. "It's for peripheral neuropathy. Do you have burning pain in your feet or hands?" c. "This antidepressant also has anti-inflammatory properties for diabetic pain." d. "That is possible, but most medications are used for several different things."

b. "It's for peripheral neuropathy. Do you have burning pain in your feet or hands?" Rationale: Damage along nerves causes peripheral neuropathy and leads to burning pain along the nerves. Many drugs, including duloxetine (Cymbalta), can be used to treat peripheral neuropathy. The nurse should assess the client for this condition and then should provide an explanation of why this drug is being used. This medication, although it is used for depression, is not being used for that reason in this case. Cymbalta does not have anti-inflammatory properties. The last option does not provide the client with enough information to be useful.

A client with diabetes asks the nurse why it is necessary to maintain blood glucose levels no lower than about 60 mg/dL. Which is the nurse's best response? a. "Glucose is the only fuel used by the body to produce the energy that it needs." b. "Your brain needs a constant supply of glucose because it cannot store it." c. "Without a minimum level of glucose, your body does not make red blood cells." d. "Glucose in the blood prevents the formation of lactic acid and prevents acidosis."

b. "Your brain needs a constant supply of glucose because it cannot store it." Rationale: Because the brain cannot synthesize or store significant amounts of glucose, a continuous supply from the body's circulation is needed to meet the fuel demands of the central nervous system. The other statements are not accurate.

The nurse correlates which laboratory value with inadequate functioning of a transplanted pancreas? a. Total white blood cell count 5000/mm3 b. 50% decrease in urine amylase level c. Blood urea nitrogen 30 mg/dL d. Elevated bilirubin level

b. 50% decrease in urine amylase level Rationale: Most pancreas transplants are anastomosed to the bladder and drain pancreatic enzymes into the urine. When the pancreas is rejected or is functioning inadequately, the level of pancreatic enzymes in the urine decreases. The other options are not indicative of inadequate pancreatic function.

Twenty minutes after a client has received a preoperative injection of atropine and midazolam (Versed), the client tells the nurse that he must be allergic to the medication because his mouth is dry and his heart seems to be beating faster than normal. What is the nurse's priority action? a. Document the findings. b. Assess the client's pulse and blood pressure. c. Administer diphenhydramine (Benadryl). d. Explain to the client that these symptoms are expected.

b. Assess the client's pulse and blood pressure.

The nurse has been reviewing options for insulin therapy with several clients. For which client does the nurse choose to recommend the pen-type injector insulin delivery system? a. Older adult client who lives at home alone but has periods of confusion b. Client on an intensive regimen with frequent, small insulin doses c. Client from the low-vision clinic who has trouble seeing the syringe d. "Brittle" client who has frequent episodes of hypoglycemia

b. Client on an intensive regimen with frequent, small insulin doses Rationale: The pen-type injector allows greater accuracy with small doses, especially doses lower than 5 units. It is not recommended for those who have visual or neurologic impairments. The client with frequent hypoglycemia would not derive special benefit from using the pen.

A client is brought to the hospital unconscious and needs emergency surgery. The client's only family member cannot come to the hospital before the surgery. Which is the best option for obtaining informed consent for the client's emergent surgery? a. Proceed with surgery and have the family member sign the consent as soon as possible. b. Contact the family member by phone and obtain verbal consent with two witnesses. c. Obtain written consultation with two surgeons that the surgery is needed. d. Have the hospital administrator appoint a temporary legal guardian.

b. Contact the family member by phone and obtain verbal consent with two witnesses.

A client with DM demonstrates acute anxiety when first admitted to the hospital for the tx of HYPERglycemia. What is the most appropriate intervention to decrease the client's anxiety? a. Administer a sedative b. Convey empathy, trust, & respect toward the client c. Ignore the S/S of anxiety so that they will soon disappear d. Make sure that the client knows all the correct medical terms to understand what is happening.

b. Convey empathy, trust, & respect toward the client Rationale: Use Therapeutic communication. Remember that the clients feelings are the priority.

The nurse is preparing a plan of care for a client with DM who has hyperglycemia. The nurse places HIGH PRIORITY on which client problem? a. Lack of knowledge b. Inadequate fluid volume c. Compromised family coping d. Inadequate consumption of nutrients

b. Inadequate fluid volume Rationale: (Use Maslow's) the correct option focuses on the physiologic need as the priority.

The nurse is caring for a critically ill client who has diabetic ketoacidosis (DKA). The nurse finds the following assessment data: blood pressure, 90/62; pulse, 120 beats/min; respirations, 28 breaths/min; urine output, 20 mL/1 hour per catheter; serum potassium, 2.6 mEq/L. The health care provider orders a 40 mEq potassium bolus and an increase in the IV flow rate. Which action by the nurse is most appropriate? a. Give the potassium after increasing the IV flow rate. b. Increase the IV rate; consult the provider about the potassium. c. Increase the IV rate; hold the potassium for now. d. Infuse the potassium first before increasing the IV flow rate.

b. Increase the IV rate; consult the provider about the potassium. Rationale: The client is acutely ill and is severely dehydrated and hypokalemic. The client requires more IV fluids and potassium. However, potassium should not be infused unless the urine output is at least 30 mL/hr. The nurse should first increase the IV rate, then consult with the provider about the potassium. The nurse should not just hold the potassium without consulting the provider because the client's level is dangerously low.

A client undergoing preoperative assessment informs the nurse that he takes medication for high blood pressure and for asthma. What is the nurse's best action? a. Tell the client not to take the medication on the day of surgery. b. Notify the surgeon and the anesthesiologist. c. Document the information in the client's record. d. Tell the client to take medications preoperatively with a sip of water.

b. Notify the surgeon and the anesthesiologist.

The nurse is caring for a client who is 2 days post-op following an abdominal hysterectomy. The client has a history of diabetes mellitus and has been receiving regular insulin accoring to capillary BG testing 4x daily. A carbohydrate-controlled diet has been ordered but the client is complaining of nausea and is not eating. On entering the room, the nurse finds the client to be confused and diaphoretic. Which action is most appropriate at this time? a. Call a code to obtain needed assistance STAT b. Obtain a capillary blood glucose level and perform a focused assessment c. Ask the UAP to stay with the client while obtaining 15 to 30g of a carbohydrate snack for the client to eat. d. Stay with the client and ask the UAP to call the HCP for a prescription for IV 50% dextrose.

b. Obtain a capillary blood glucose level and perform a focused assessment Rationale: Recall that Assessment is the 1st step of the nursing process. Diaphoresis & confusion are signs of moderate hypoglycemia. A likely cause was the administration of insulin without the client eating enough food. Upon assessment, if hypoglycemia is confirmed then the nurse would stay with the client and ask UAP to obtain a carbohydrate snack for the client.

The nurse is teaching a postmenopausal client about the risk of acquiring HIV infection. The client states, "I'm an old woman! I cannot possibly get HIV." What is the nurse's best response? a."Your vaginal walls become thicker after menopause, which increases your risk." b."Women in your age-group are the fastest growing population of AIDS clients today." c."Hormonal fluctuations after menopause make it harder to fight off infection." d."You might be right. How often do you engage in sexual activities?"

b. Women in your age ghroup are teh fastes growing poulation of AIDS clients today

The nurse determines that which arterial blood gas values are consistent with ketoacidosis in the client with diabetes? a. pH 7.38, HCO3- 22 mEq/L, PCO2 38 mm Hg, PO2 98 mm Hg b. pH 7.28, HCO3- 18 mEq/L, PCO2 28 mm Hg, PO2 98 mm Hg c. pH 7.48, HCO3- 28 mEq/L, PCO2 38 mm Hg, PO2 98 mm Hg d. pH 7.28, HCO3- 22 mEq/L, PCO2 58 mm Hg, PO2 88 mm Hg

b. pH 7.28, HCO3- 18 mEq/L, PCO2 28 mm Hg, PO2 98 mm Hg Rationale: When the lungs can no longer offset acidosis, the pH decreases to below normal. The arterial blood gases show primary metabolic acidosis with decreased bicarbonate levels and a compensatory respiratory alkalosis with decreased carbon dioxide levels.

Which statement made by a diabetic client who has a urinary tract infection indicates that teaching was effective regarding antibiotic therapy? a. "If my temperature is normal for 3 days in a row, I can stop taking my medicine." b. "If my temperature goes above 100° F (37.8° C), I should double the dose." c. "Even if I feel completely well, I should take the medication until it is gone." d. "When my urine no longer burns, I will no longer need to take the antibiotics."

c. "Even if I feel completely well, I should take the medication until it is gone." Rationale: Antibiotic therapy is most effective when the client takes the prescribed medication for the entire course, not just when symptoms are present. The other statements are inaccurate.

A client has been newly diagnosed with diabetes mellitus. Which statement made by the client indicates a need for further teaching regarding nutrition therapy? a. "I should be sure to eat moderate to high amounts of fiber." b. "Saturated fats should make up no more than 7% of my total calorie intake." c. "I should try to keep my diet free from carbohydrates." d. "My intake of plain water each day is not restricted."

c. "I should try to keep my diet free from carbohydrates." Rationale: Carbohydrates are an extremely important source of energy. They should compose at least 45% to 65% of the diabetic person's total caloric intake. The client needs to eat at least 130 g of carbohydrates a day. The other statements show good understanding.

The nurse has been teaching a client about a new diagnosis of diabetes mellitus. Which statement by the client indicates a good understanding of self-management? a. "After bathing each day, I will inspect my feet and rub lotion between my toes and on my heels." b. "I can store 3 months' worth of insulin at room temperature as long as the bottles are not open." c. "My medical alert bracelet is important to identify me as having diabetes if I am unconscious." d. "If I travel eastward to see my family, I should plan on using more insulin on the day I travel."

c. "My medical alert bracelet is important to identify me as having diabetes if I am unconscious." Rationale: It is important to encourage clients with diabetes mellitus to wear a medical alert bracelet. This bracelet is helpful if the client becomes hypoglycemic and is unable to provide self-care. Lotion should not be applied between the toes. Insulin in active use can be stored at room temperature for 28 days; otherwise insulin is stored in the refrigerator. Eastbound travel will require a reduction in insulin.

Two months after a simultaneous pancreas-kidney (SPK) transplantation, a client is diagnosed as being in acute rejection. The client states, "I was doing so well with my new organs, and the thought of having to go back to living on hemodialysis and taking insulin is so depressing." Which is the nurse's best response? a. "You should have followed your drug regimen better." b. "You should be glad that at least dialysis treatment is an option for you." c. "One acute rejection episode does not mean that you will lose the new organs." d. "Our center is high on the list for obtaining organs from the national registry."

c. "One acute rejection episode does not mean that you will lose the new organs." Rationale: An episode of acute rejection does not automatically mean that the client will lose the transplant. Pharmacologic manipulation of host immune responses at this time can limit damage to the organ and allow the graft to be maintained. The other statements either belittle the client or downplay his or her concerns.

A client with Type 1 DM calls the nurse to report recurrent episodes of HYPOglycemia with exercising. Which statement by the client indicates an inadequate understanding of the peak action of NPH insulin and exercise? a. "The best time for me to exercise is after I eat" b. "The best time for me to exercise is after breakfast" c. "The best time for me to exercise is mid- to late afternoon? d. "The best time for me to exercise is after my morning snack?

c. "The best time for me to exercise is mid- to late afternoon? Rationale: Recall that NPH insulin peaks at 4-12 hours. A hypoglycemia reaction may occur in response to increased exercise. Clients should AVOID exercise during PEAK TIMES of insulin.

A client with a diagnosis of diabetic ketoacidosis (DKA) is being treated in the emergency department. Which findings would the nurse expect to note as confirming this diagnosis? (Select all the apply) a. Increase in pH b. Comatose state c. Deep, rapid breathing d. Decreased urine output e. Elevated blood glucose level f. Low plasma bicarbonate level

c. Deep, rapid breathing e. Elevated blood glucose level f. Low plasma bicarbonate level Rationale: In DKA, the arterial pH is <7.35, plasma bicarbonate is <15 mEq/L, the BG level is >250 mg/dL, and ketones are present in the blood & urine. Client would experience polyuria, & Kussmaul's respirations. A comatose state may occur but coma would not confirm the diagnosis. Remember that in acidosis, the pH and plasma bicarbonate are both low.

In preparing a staff in-service presentation about diabetes mellitus, the nurse includes which information? a. Diabetes increases the risk for development of epilepsy. b. The cure for diabetes is the administration of insulin. c. Diabetes increases the risk for development of cardiovascular disease. d. Carbohydrate metabolism is altered in diabetes, but protein metabolism is normal.

c. Diabetes increases the risk for development of cardiovascular disease. Rationale: Diabetes mellitus is a major risk factor for morbidity and mortality caused by coronary artery disease, cerebrovascular disease, and peripheral vascular disease. Insulin is a lifelong treatment for some diabetic clients. Because insulin regulates the metabolism of carbohydrates, fats, and protein, abnormalities in insulin production or use alter their metabolism.

During assessment of a client with a 15-year history of diabetes, the nurse notes that the client has decreased tactile sensation in both feet. Which action does the nurse take first? a. Document the finding in the client's chart. b. Test sensory perception in the client's hands. c. Examine the client's feet for signs of injury. d. Notify the health care provider.

c. Examine the client's feet for signs of injury. Rationale: Diabetic neuropathy is common when the disease is of long duration. The client is at great risk for injury in any area with decreased sensation because he or she is less able to feel injurious events. Feet are common locations for neuropathy and injury, so the nurse should inspect them for any signs of injury. After assessing, the nurse should document findings in the client's chart. Testing sensory perception in the hands may or may not be needed. The health care provider can be notified after assessment and documentation have been completed.

A client is admitted to a hospital with a diagnosis if diabetic ketoacidosis (DKA). The initial BG level was 950mg/dL. A continuous intravenous infusion of short-acting insulin is initiated, along with IV rehydration with normal saline. The serum glucose level is now 240 mg/dL. The nurse would NEXT prepare to administer which item? a. Ampule of 50% dextrose b. NPH insulin subcutaneously c. IV fluids containing dextrose d. Phenytoin (Dilantin) for the prevention of seizures

c. IV fluids containing dextrose Rationale: During mgmt of DKA, when the BG level fall to 250-300 mg/dL, the infusion rate is reduced and a dextrose solution is added to maintain a BG level of about 250 mg/dL, or until the client recovers from ketosis.

The nurse is teaching a client with type 2 diabetes about acute complications. Which teaching point by the nurse is most accurate? a. Ketosis is less prevalent among obese adults owing to the protective effects of fat. b. People with type 2 diabetes have normal lipid metabolism, so ketones are not made. c. Insulin produced in type 2 diabetes prevents fat catabolism but not hyperglycemia. d. Oral antidiabetic agents do not promote the breakdown of fat for fuel (lipolysis).

c. Insulin produced in type 2 diabetes prevents fat catabolism but not hyperglycemia. Rationale: Ketosis occurs as a result of fat catabolism when intracellular glucose is unavailable for energy production. The client with type 1 diabetes becomes ketotic because he or she produces no insulin, and blood glucose cannot enter the cells. In type 2 diabetes, natural insulin production continues, although at a greatly reduced level. This level is not sufficient to keep blood glucose levels in the normal range but permits just enough glucose to enter cells for energy production, so that fats are not catabolized for this purpose. The other rationales are incorrect.

A client is brought to the emergency department in an unresponsive state, and a diagnosis of hyperglycemic hyperosmolar nonketotic syndrome is made. The nurse would immediately prepare to initiate which anticipated health care provider's prescription? a. Endotracheal intubation b. 100 units of NPH insulin c. Intravenous infusion of normal saline d. Intravenous infusion of sodium bicarbonate

c. Intravenous infusion of normal saline Rationale: Tx for HHNS is similar to tx for DKA and begins with rehydration. (remember the turtle slide - "HI...E" Hydration, Insulin, Electrolytes)

During the preoperative assessment, the client tells the nurse that he smokes three packs of cigarettes daily. Which action by the nurse is best? a. Call the surgeon to cancel the surgery. b. Have baseline laboratory studies drawn. c. Perform a respiratory assessment. d. Give a nebulizer treatment.

c. Perform a respiratory assessment.

A client with diabetes has a serum creatinine of 1.9 mg/dL. The nurse correlates which urinalysis finding with this client? a. Ketone bodies in the urine during acidosis b. Glucose in the urine during hyperglycemia c. Protein in the urine during a random urinalysis d. White blood cells in the urine during a random urinalysis

c. Protein in the urine during a random urinalysis. Rationale: Urine should not contain protein. The presence of proteinuria in a diabetic client marks the beginning of kidney problems known as diabetic nephropathy, which progresses eventually to end-stage kidney disease. Decline in kidney function is assessed with serum creatinine. This client's creatinine level is high. The other findings would not be correlated with declining kidney function.

A client is being treated for hyperglycemic-hyperosmolar state (HHS). Which clinical manifestation indicates to the nurse that the therapy needs to be adjusted? a. Serum potassium level has increased from 2.8 to 3.2 mEq/L. b. Blood osmolarity has decreased from 350 to 330 mOsm. c. Score on the Glasgow Coma Scale is unchanged from 3 hours ago. d. Urine has remained negative for ketone bodies for the past 3 hours.

c. Score on the Glasgow Coma Scale is unchanged from 3 hours ago. Rationale: A slow but steady improvement in central nervous system (CNS) functioning is the best indicator of therapy effectiveness for HHS. Lack of improvement in level of consciousness may indicate inadequate rates of fluid replacement. The other assessment findings do not indicate adequacy of treatment.

A client voluntarily signed the operative consent form. What is the nurse's next action? a. Teach the client about the surgery. b. Have family members witness the signature. c. Sign under the client's name as a witness. d. Call for the physician to sign the form.

c. Sign under the client's name as a witness.

A client was admitted with diabetic ketoacidosis (DKA). Which manifestations does the nurse monitor the client most closely for? a. Shallow slow respirations and respiratory alkalosis b. Decreased urine output and hyperkalemia c. Tachycardia and orthostatic hypotension d. Peripheral edema and dependent pulmonary crackles

c. Tachycardia and orthostatic hypotension Rationale: DKA leads to dehydration, which is manifested by tachycardia and orthostatic hypotension. Usually clients have Kussmaul respirations, which are fast and deep. Increased urinary output (polyuria) is severe. Because of diuresis and dehydration, peripheral edema and crackles do not occur.

The nurse performs a physical assessment on a client with Type 2 DM. Findings include a Fasting Blood Glucose level 120 mg/dL, temp of 101, pulse of 88, resp of 20, and BP 100/72. Which finding would be of MOST Concern to the nurse? a. Pulse b. Respiration c. Temperature d. Blood pressure

c. Temperature Rationale: An elevated temperature may indicate infection which is a leading cause of hyperglycemic hyperosmolar nonketotic syndrome or DKA.

Three hours after surgery, the nurse notes that the breath of the client with type 1 diabetes has a "fruity" odor. Which is the nurse's best first action? a. Document the finding in the client's chart. b. Increase the IV fluid flow rate. c. Test the serum for ketone bodies. d. Perform pulmonary hygiene.

c. Test the serum for ketone bodies. Rationale: The stress of surgery increases the action of counterregulatory hormones and suppresses the action of insulin, predisposing the client to ketoacidosis and metabolic acidosis. One manifestation of ketoacidosis is a "fruity" odor to the breath. Documentation should occur after all assessments have been completed. The other options are not needed for this problem.

A client with untreated diabetes mellitus has polyuria, is lethargic, and has a blood glucose of 560 mg/dL. The nurse correlates the polyuria with which finding? a. Serum sodium, 163 mEq/L b. Serum creatinine, 1.6 mg/dL c. Presence of urine ketone bodies d. Serum osmolarity, 375 mOsm/kg

d. Serum osmolarity, 375 mOsm/kg Rationale: Hyperglycemia causes hyperosmolarity of extracellular fluid. This leads to polyuria from an osmotic diuresis. The client's serum osmolarity is high. The client's sodium would be expected to be high owing to dehydration. Urine ketone bodies and serum creatinine are not related to the polyuria.

pt has respiratory distress, confusion and dsyrthmias. what electrolyte imbalance

hypokalemia

A young adult client newly diagnosed with type 1 diabetes mellitus has been taught about self-care. Which statement by the client indicates a good understanding of needed eye examinations? a. "At my age, I should continue seeing the ophthalmologist as I usually do." b. "I will see the eye doctor whenever I have a vision problem and yearly after age 40." c. "My vision will change quickly now. I should see the ophthalmologist twice a year." d. "Diabetes can cause blindness, so I should see the ophthalmologist yearly."

d. "Diabetes can cause blindness, so I should see the ophthalmologist yearly." Rationale: Diabetic retinopathy is a leading cause of blindness in North America. All clients with diabetes, regardless of age, should be examined by an ophthalmologist (rather than an optometrist or optician) at diagnosis and at least yearly thereafter.

A client with diabetes has proliferative retinopathy, nephropathy, and peripheral neuropathy. Which statement by the client indicates a good understanding of the disease and exercise? a. "Because I have so many complications, I guess exercise is not a good idea." b. "I have so many complications that I better exercise hard to keep from getting worse." c. "I love to walk outside, but I probably better avoid doing that now." d. "I should look into swimming or water aerobics to get my exercise."

d. "I should look into swimming or water aerobics to get my exercise." Rationale: Exercise is not contraindicated for this client, although modifications based on existing pathology are necessary to prevent further injury. Swimming or water aerobics will give the client exercise without the worry of having the correct shoes or developing a foot injury. The client can walk outside if this is the exercise that he or she prefers. The client should not exercise too vigorously.

The nurse provides instructions to a client newly diagnosed with Type 1 DM. The nurse recognizes accurate understanding of measures to prevent diabetic ketoacidosis when the client makes which statement? a. "I will stop taking my insulin if I'm too sick to eat" b. "I will decrease my insulin dose during times of illness" c. "I will adjust my insulin dose according to the level of glucose in my urine" d. "I will notify my health care provider if my BG level is >250mg/dL"

d. "I will notify my health care provider if my BG level is >250mg/dL" Rationale: (note that options a,b,c are similar or alike) During illness, the client should monitor BG levels and should notify HCP if >250mg/dL.

A client tells the nurse that he has an advance directive with durable power of attorney for health care. The client asks how the advance directive will affect the surgery. What is the nurse's best response? a. "You will not be intubated during general anesthesia for the surgery." b. "There will be no effect on your surgery." c. "The surgical staff will resuscitate only if your heart stops during the operation." d. "If you are unable to make a decision, your designee will be asked."

d. "If you are unable to make a decision, your designee will be asked."

A client is learning to inject insulin. Which action is important for the nurse to teach the client? a. "Do not use needles more than twice before discarding." b. "Massage the site for 1 full minute after injection." c. "Try to make the injection deep enough to enter muscle." d. "Keep the vial you are using in the pantry or the bedroom drawer."

d. "Keep the vial you are using in the pantry or the bedroom drawer." Rationale: Cold insulin directly from the refrigerator is the most common cause of irritation (not infection) at the insulin injection site. Insulin in active use can be stored at room temperature. However, the bathroom is not the best place to store any medication because of increased heat and humidity. Needles should be used only once. Massage will not prevent or treat irritation from cold insulin. Insulin is given by subcutaneous, not intramuscular, injection.

When the nurse brings a client's preoperative medications, the client responds, "I don't need that. I had a good night's sleep last night." What is the nurse's best response? a. "The doctor ordered this medication so you should take it." b. "I will make a note that you refused to take the medication." c. "I will ask your surgeon if you have to take the medication." d. "Let me teach you about your medications for surgery."

d. "Let me teach you about your medications for surgery."

The nurse is teaching a client about sick day management. Which statement by the nurse is most accurate? a. "Continue your prescribed exercise regimen even if you are sick." b. "Avoid eating or drinking to reduce vomiting and diarrhea." c. "Do not use insulin or take your oral antidiabetic agent if you vomit." d. "Monitor your blood glucose levels at least every 4 hours."

d. "Monitor your blood glucose levels at least every 4 hours." Rationale: When ill, the client should monitor his or her blood glucose at least every 4 hours. The other statements are inaccurate.

A client with type 1 diabetes asks whether an occasional glass of wine is allowed in the diet. Which is the nurse's best response? a. "Drinking any wine or alcohol will increase your insulin requirements." b. "Because of poor kidney function, people diagnosed with diabetes should avoid alcohol at all times." c. "You shouldn't drink alcohol because it will make you hungry and overeat." d. "One glass of wine is okay with a meal and is counted as two fat exchanges."

d. "One glass of wine is okay with a meal and is counted as two fat exchanges." Rationale: Under normal circumstances, blood glucose levels will not be affected by moderate use of alcohol when diabetes is well controlled. Because alcohol can induce hypoglycemia, it should be ingested with or shortly after a meal. One alcoholic beverage is substituted for two fat exchanges when caloric intake is calculated. The other statements are incorrect.

A client recently diagnosed with type 1 diabetes tells the nurse, "I will never be able to stick myself with a needle." Which is the nurse's best response? a. "Try not to worry about it. We will give you your injections here in the hospital." b. "Everyone gets used to giving themselves injections. It really does not hurt." c. "I am not sure how your disease can be managed if you refuse to give yourself the shots." d. "Tell me what it is about the injections that is concerning you."

d. "Tell me what it is about the injections that is concerning you." Rationale: Devote as much teaching time as possible to insulin injection and blood glucose monitoring. Clients with newly diagnosed diabetes are often fearful of giving themselves injections. If the client is worried about giving the injections, it is best to try to find out what specifically is causing the concern, so it can be addressed. Giving the injections for the client does not promote self-care ability. Telling the client that others give themselves injections may cause the client to feel badly. Stating that you don't know another way to manage the disease is dismissive of the client's concerns.

confusion, seizures and muscle weakness are signs and symptoms of

hyponatremia

A diabetic client has numbness and reduced sensation. Which intervention does the nurse teach this client to prevent injury? a. "Examine your feet daily using a mirror." b. "Rotate your insulin injection sites." c. "Wear white socks instead of colored socks." d. "Use a bath thermometer to test the water temperature."

d. "Use a bath thermometer to test the water temperature." Rationale: Clients with diminished sensory perception can easily experience a burn injury when bath water is too hot. Instead of checking the temperature of the water by feeling it, they should use a thermometer. Examining the feet daily does not prevent injury, although daily foot examinations are important to find problems so they can be addressed. Rotating insulin and wearing white socks also will not prevent injury.

The nurse is caring for a client admitted to the ER with DKA. In the acute phase, the nurse plans for which priority intervention? a. Correct the acidosis b. Administer 5% dextrose intravenously c. Apply a monitor for an EKG d. Administer short-duration insulin intravenously

d. Administer short-duration insulin intravenously Rationale: Lack of insulin is the primary cause of DKA. Remember that in DKA, the initial tx is short or rapid-acting insulin. Normal saline is administered initially. (Hydrate, Insulin, Electrolytes)

A client has diabetic ketoacidosis and manifests Kussmaul respirations. What action by the nurse takes priority? a. Administration of oxygen by mask or nasal cannula b. Intravenous administration of 10% glucose c. Implementation of seizure precautions d. Administration of intravenous insulin

d. Administration of intravenous insulin Rationale: The rapid, deep respiratory efforts of Kussmaul respiration is the body's attempt to reduce the acids produced by using fat rather than glucose for fuel. The client who is in ketoacidosis and who does not also have a respiratory impairment does not need additional oxygen. Only the administration of insulin will reduce this type of respiration by assisting glucose to move into cells and to be used for fuel instead of fat. Giving the client glucose would be contraindicated. The client does not require Seizure Precautions.

The nurse is conducting preoperative assessments. Which client does the nurse teach about the possibility of developing a venous thromboembolism (VTE)? a. Client with a latex allergy b. Client with body mass index (BMI) of 19 c. Client with an international normalized ratio (INR) of 2.2 d. Client undergoing hip replacement surgery

d. Client undergoing hip replacement surgery

An external insulin pump is prescribed for a client with diabetes mellitus and the client asks the nurse about the functioning of the pump. The nurse bases the response on which information about the pump? a. Is timed to release programmed doses of short-duration or NPH insulin into the bloodstream at specific intervals. b. Continuously infuses small amounts of NPH insulin into the bloodstream while regularly monitoring blood glucose levels. c. I surgically attached to the pancreas and infuses regular insulin into the pancreas, which in turn releases the insulin into the bloodstream. d. Gives a small continuous dose of short-duration insulin subcutaneously, and the client can self-administer a bolus with an add'l dose from the pump b4 each meal.

d. Gives a small continuous dose of short-duration insulin subcutaneously, and the client can self-administer a bolus with an add'l dose from the pump b4 each meal.

A client on an intensified insulin regimen consistently has a fasting blood glucose level between 70 and 80 mg/dL, a postprandial blood glucose level below 200 mg/dL, and a hemoglobin A1c level of 5.5%. Which is the nurse's interpretation of these findings? a. Increased risk for developing ketoacidosis b. Increased risk for developing hyperglycemia c. Signs of insulin resistance d. Good control of blood glucose

d. Good control of blood glucose Rationale: The client is maintaining blood glucose levels within the defined ranges for goals in an intensified regimen. Because the client's glycemic control is good, he or she is not at higher risk for ketoacidosis or hyperglycemia and is not showing signs of insulin resistance.

A client who has used insulin for diabetes control for 20 years has a spongy swelling at the site used most frequently for insulin injection. Which is the nurse's best action? a. Apply ice to this area for 20 minutes. b. Document the finding in the client's chart. c. Assess the client for other signs of cellulitis. d. Instruct the client to use a different site for injection.

d. Instruct the client to use a different site for injection. Rationale: The client has lipohypertrophy as a result of repeated injections at the same site. Avoiding this site for an extended period of time allows dystrophic changes to regress or at least not to become worse. The other actions are not needed.

The nurse is monitoring a client with hypoglycemia. Glucagon provides which function? a. It enhances the activity of insulin, restoring blood glucose levels to normal more quickly after a high-calorie meal. b. It is a storage form of glucose and can be broken down for energy when blood glucose levels are low. c. It converts excess glucose into glycogen, lowering blood glucose levels in times of excess. d. It prevents hypoglycemia by promoting release of glucose from liver storage sites.

d. It prevents hypoglycemia by promoting release of glucose from liver storage sites. Rationale: Glycogen is a counterregulatory hormone secreted by the alpha cells of the pancreas when blood glucose levels are low. The actions of glycogen that raise blood glucose levels include stimulating the liver to break down glycogen (glycogenolysis) and forming new glucose from protein breakdown (gluconeogenesis). The other statements are not accurate descriptions of the actions of glucagon.

The nurse is completing preoperative teaching for a client, and it becomes apparent that the client does not understand the surgery that will be performed. What is the priority action for the nurse? a. Obtain informed consent from the client. b. Continue teaching the client about the surgery. c. Revise the teaching plan for the client. d. Notify the surgeon and document the finding.

d. Notify the surgeon and document the finding.

When examining an adult client's preoperative laboratory results, the nurse notes that the potassium level is 2.9 mEq/mL. What is the nurse's priority action? a. Document the finding. b. Alter the client's diet to include fruit. c. Increase the IV flow rate. d. Notify the surgeon.

d. Notify the surgeon.


Ensembles d'études connexes

Module 3 - Adjustable Rate Mortgages (ARMs)

View Set

Unit 4 Objective Knowledge Check

View Set

CCNA2 Chapter 10 Device Discovery, Management, and Maintenance

View Set

AD RESEARCH FINAL: Descriptive Statistics

View Set

Chapter 7 (Exam 1), Chapter 6 (Exam 1), Chapter 5 (Exam 1), Chapter 4 (Exam 1), Chapter 3 (Exam 1), Chapter 2 (Exam 1), Chapter 1 (Exam 1)

View Set

Ch 1 Introduction to Lifespan Development

View Set

EMT-B Chapter 3: Medical, Legal & Ethical

View Set

Chapter 50 Care of Surgical Patients Elsevier

View Set